■ちょっとした疑問はここに書いてね14■

このエントリーをはてなブックマークに追加
1超回復無視 ◆pW.7d.xs
   ,.―――')      ______________
 γ∞γ~  \    /
 人w"/ 从从) )   /  いちいちスレッド立てないで
  ヽ | | l  l |〃 <  ちょっとした疑問はここに書いてね。
  `wハ~ ーノ)    \                 Part 14
   / \`「        ̄ ̄ ̄ ̄ ̄ ̄ ̄ ̄ ̄ ̄ ̄ ̄ ̄ ̄
前スレ
Part13:http://science.2ch.net/test/read.cgi/sci/1029946423/
【質問する前に】
教科書をよく読もう。
http://www.yahoo.co.jp/ とか http://www.google.com/ を利用し自分で調べること。
宿題を聞くときは、どこまでやってみてどこが分からないのかを書くこと。
丸投げはダメだからね。
(レスする人も丸投げに答えるのは自粛してねっていうかや・め・ろ)

質問に対する返答には、何かしらの返答ちょうだいね。

★過去スレ、関連スレはここ>>2よ。
★それから、書き込む前に>>3の注意事項を読んでね。
★数式の書き方(参考)はこちら>>4
(予備リンク:>>2-10
荒らし厳禁、煽りは黙殺、忘れないうちに定期age。
単発質問スレを発見したらこのスレへの誘導をよろしくね。>ALL
2ご冗談でしょう?名無しさん:02/09/22 02:41 ID:???
3ご冗談でしょう?名無しさん:02/09/22 02:41 ID:???
書き込む際の注意

1.)
板の性格上、UNIX端末からの閲覧も多いと推察されます。
機種依存文字(ローマ数字、丸数字等)は避けて下さい。

2.)
以下のような質問に物理板住人は飽き飽きしているので、たぶん無視されます。
しないで下さい。
「相対性理論は間違っています」「量子力学は間違っています」
「宇宙論は間違っています」「シュレディンガーの猫は変です」
「永久機関を作りました」「タイムマシンについて教えて」
「どうして〜?」関連(→「どのようにして〜?」と質問すること)
「なぜ〜?」関連(物理学の対象ではないため)
それでも何か書きたければ
相対論は:http://science.2ch.net/test/read.cgi/sci/1006249553/
反相対論は:http://science.2ch.net/test/read.cgi/sci/1023215815/
量子力学は:http://science.2ch.net/test/read.cgi/sci/1010632342/
(↑量子力学スレはありもの有効利用ということで。ご協力ください)
「どうして・なぜ」:http://academy.2ch.net/philo/
(哲学板・雑談板のほうがむいている場合が多いです。)
雑談は雑談スレで:http://science.2ch.net/test/read.cgi/sci/973536997/l50

3.)
宿題を聞くときは、どこまでやってみてどこが分からないのかを書くこと。
丸投げはダメよ。丸投げに答えるのもダメよ。
せめてポインタを示す程度に留めましょう。

4.)
厨房を放置できない人も厨房です。
4ご冗談でしょう?名無しさん:02/09/22 02:42 ID:???
【掲示板での数学記号の書き方例(数学板より拝借)】
●スカラー:a,b,c,...,z, A,B,C,...,Z, α,β,γ,...,ω, Α,Β,Γ,...,Ω, ...
●ベクトル:x=[x[1],x[2],...], |x>, x↑, vector(x) (← 混同しない場合はスカラーと同じ記号でいい.通常は(成分を横で書いても)縦ベクトルと
して扱う.)
●行列(1成分表示):A[i,j], I[i,j]=δ_(ij)
●行列(全成分表示):A=[[A[1,1],A[2,1],...],[A[1,2],A[2,2],...],...]=[a1,a2,a3,...], I=[[1,0,0,...],[0,1,0,...],[0,0,1,...],...] (← ここでは列ごとに表示(縦ベク
トルを横に並べる).行ごとに表示しても構わないが,統一して使わないと混同するので注意.)
●関数:f(x), f[x]
●数列:a(n), a[n], a_n
●足し算:a+b
●引き算:a-b
●掛け算:a*b, ab (← 通常"*"を使い,"x"は使わない.)
●割り算・分数:a/b, a/(b+c), a/(bc) (← 通常"/"を使い,"÷"は使わない.)
●複号:a±b=a士b, a干b (← "±"は「きごう」で変換可.他に漢字の"士""干"なども利用できる.)
●平方根:√(a+b)=(a+b)^(1/2) (← "√"は「るーと」で変換可.)
●指数・指数関数:a^b, x^(n+1), exp(x+y)=e^(x+y) (← "^"を使う."exp"はeの指数.)
●対数・対数関数:log_{a}(b), log(x/2)=log_{10}(x/2), ln(x/2)=log_{e}(x/2) (← 底を省略する場合,"log"は常用対数,"ln"は自然対数.)
5ご冗談でしょう?名無しさん:02/09/22 02:43 ID:???
●三角比・三角関数:sin(a), cos(x+y), tan(x/2)
●内積・外積・スカラー3重積:a・b=(a,b), axb=[a,b], a・(bxc)=(axb)・c=[a,b,c]=det([a,b,c])
●行列式・トレース:|A|=det(A), tr(A)
●絶対値:|x|
●ガウス記号:[x] (← 関数の変数表示などと混同しないように注意.)
●共役複素数:z~
●階乗:n!=n*(n-1)*(n-2)*...*2*1, n!!=n*(n-2)*(n-4)*...
●順列・組合せ:P[n,k]=nPk, C[n.k]=nCk, Π[n,k]=nΠk, H[n,k]=nHk (← "Π"は「ぱい」で変換可.)
●微分・偏微分:y', dy/dx, ∂y/∂x (← "∂"は「きごう」で変換可.)
●ベクトル微分:∇f=grad(f), ∇・A=div(A),∇xA=rot(A), (∇^2)f=Δf (← "∇"は「きごう」,"Δ"は「でるた」で変換可.)
●積分:∫[0,1]f(x)dx=F(x)|_[x=0,1], ∫[y=0,x]f(x,y)dy, ∬_[D]f(x,y)dxdy, 点[C]f(r)dl (← "∫"は「いんてぐらる」,"∬"は「きごう」で変換
可.)
●数列和・数列積:Σ[k=1,n]a(k), Π[k=1,n]a(k) (← "Σ"は「しぐま」,"Π"は「ぱい」で変換可.)
●極限:lim[x→∞]f(x) (← "∞"は「むげんだい」で変換可.)
●図形:"△"は「さんかく」,"∠"は「かく」,"⊥"は「すいちょく」で変換可.

●その他
・関数等の変数表示や式の括弧は,括弧()だけでなく[]{}を適当に組み合わせると見やすい場合がある.
・ギリシャ文字はその読み方で変換可.
・上記のほとんどの数学記号や上記以外の数学記号"⇒∀≠≧≒∈±≡∩∽"などは「きごう」で順次変換できる.
6ご冗談でしょう?名無しさん:02/09/22 05:00 ID:OLhdsQ7v

京都大学からW32.Magistr.39921@mm
資源工学専攻物理探査工学研究室http://tansa1.kumst.kyoto-u.ac.jp/
只今までに検出されたウィルス
http://yuri.robot.mach.mie-u.ac.jp/~virusstat/old_log_1.html
時刻 ウィルス名 Message-Id
2002-09-21 22:24:53 W32.Magistr.39921@mm< [email protected] >
2002-09-21 22:24:54 W32.Magistr.39921@mm< [email protected] >
8ご冗談でしょう?名無しさん:02/09/22 11:00 ID:SzJbp/TT
  今だ〜100ゲットー
  ∧_∧
 (・∀・∩)
 (つ  丿
  (  )
  ( )
9ご冗談でしょう?名無しさん:02/09/22 13:55 ID:dEciaN3H
素朴な疑問なんですが
宇宙が縮みはじめると
物理の法則とかが
変わって来たりするのでしょうか?

10荒らそうぜ:02/09/22 13:58 ID:5uFvA7Uz
http://homepage3.nifty.com/komada/
このクソHPを潰そう
112ch The Ripper:02/09/22 14:08 ID:V8KWiOok
疑問・・・特異点についてあるだけの知識をくれ
12ご冗談でしょう?名無しさん:02/09/22 14:47 ID:BO7PT7H4
くだらない質問ですいません。
僕は文系の者で物理の知識は1Bがなんとなく判る程度の者です
(独学ですのでわからないことだらけです)。

たまにサッカーマンガなどで、シュート→キーパーキャッチ→キーパーごと吹っ飛んで
ゴールというシーンがありますが、あれを物理学で考えるとシュートの速度はどのくら
いになるのでしょうか。

運動量とかいろいろ考えたのですがどうもよくわかりません。
必要ない数字もあるかと思いますが調べたところサッカーボールは
重さ410〜450g、直径68〜70cm、空気圧0.6〜1.1、
(プロ)サッカー選手の平均身長約178cm、体重約74kgだそうです。
芝生とスパイクとの平均的な摩擦係数はよくわかりません。

だいたいのマンガではそのままゴールネットまでキーパーごと飛んでいくので
3m程は飛んでいると思います。

ボールとキーパーが衝突する際のボールの地面との角度やキャッチの際の衝撃の
殺し方にもよるかと思いますが最低でもどのくらいなのでしょう。
ややこしいので回転はかかってないってコトで。

物理についてはかなり無知なのですが、0.45kgのもので74kgのものを吹っ飛ばす
ということは本来ならば体を貫通するほどそうとう速いのではないか、と思って
気になって仕方ないんです。

どなたかこの疑問にこたえてもらえないでしょうか。
よろしくお願いします。
13ご冗談でしょう?名無しさん:02/09/22 15:46 ID:EnMT6kFG
>>9
その可能性はあります。
宇宙の大局的な構造が物理法則にどうかかわっているのか、は、今もまだ謎の多いところです。
14ご冗談でしょう?名無しさん:02/09/22 19:33 ID:???
くだらない質問で申し訳ないんですが

タイヤが回ると充電できる電気自動車が
上るときは急で短い坂、下るときは緩やかで長い坂をつかい
上ったり下ったりを繰り返すことで、充電はできるんでしょうか?
ただし、上り方や下り方、回数は問わないものとします。
15ご冗談でしょう?名無しさん:02/09/22 19:35 ID:???
充電はできるけど放電のほうが大きい
16ご冗談でしょう?名無しさん:02/09/22 19:49 ID:IBRihf0N
>>12

大雑把に見積もってみるね

摩擦係数って0.7くらい?
74kg×9.8m/s×0.7×3m=1500Jが、ボールがキーパーにした仕事(つまり失ったエネルギー)

シューターがボールにこれだけのエネルギーを与えるんだから、
0.5×0.5kg×v^2=1500を解くと大体77m/s=時速270キロ超

270キロだからレヴィンシュートでギリギリ小柄なキーパー吹っ飛びます
17ご冗談でしょう?名無しさん:02/09/23 04:18 ID:Occ4bf/A
hν=c/λ
の関係についてなぜこうなるのか分かる方いたら教えてください
hはプランク定数 νは振動数 cは光速 λは波長
18ご冗談でしょう?名無しさん:02/09/23 04:27 ID:???
ν=c/λ
定義より明らか。
1917:02/09/23 04:32 ID:Occ4bf/A
>>18
プランク定数はどこへ行くのですか?
左の単位はJで右がs^-1になるんだがその関係がよく分からないんですが
20ご冗談でしょう?名無しさん:02/09/23 04:39 ID:???
宇宙が気圧0とか絶対零度とか空気が無いってのは全部仮説なんですか?
21ご冗談でしょう?名無しさん:02/09/23 04:40 ID:7CDHLBms
>>19
プランク定数が入るのがそもそも間違い。
あなたの言うように単位が合ってないことからも明らかです。
2217:02/09/23 04:40 ID:Occ4bf/A
>>20
それより私の質問分かりませんか?
2317:02/09/23 04:44 ID:Occ4bf/A
>>21
なぬぅ?じゃあこの教科書は誤植か?出版社に小一時間問い詰めてやる
じゃあΔE=hc/λ ならあってますか?
24ご冗談でしょう?名無しさん:02/09/23 04:48 ID:???
>>23
僞が何なのかにもよりますが、例えば準位間のエネルギー差等であれば
あってます。
2517:02/09/23 04:49 ID:Occ4bf/A
>>23
ありがとうございます
胸のつかえが取れました
2617:02/09/23 04:50 ID:Occ4bf/A
>>24だった…欝だ氏のう
2718=21=24:02/09/23 04:59 ID:???
>>17
今思ったんだけど、「hν=hc/λ」と書こうとして、単に右辺のhが抜けただけ
かもしれません。
2817:02/09/23 06:22 ID:Occ4bf/A
>>27
そんなとこでしょうね
昇華坊の量子なんだけどとにかくほかに誤植が多いんで困ってます
29ご冗談でしょう?名無しさん:02/09/23 09:21 ID:NCA8O28F
くだらない質問で恐縮です。
金属(導体)って誘電率εでいくと
ε→0
ε→∞
どちらの極限として考えるのでしょうか?
30ご冗談でしょう?名無しさん:02/09/23 09:49 ID:6t3aflz6
∞だよ
31ご冗談でしょう?名無しさん:02/09/23 10:26 ID:lGmZST31
E=mc^2とかのmやcって、グラムで、メートル秒なの?
単位がわからん。
32をっさん ◆96jfHsJM :02/09/23 10:33 ID:???
>>31
mは質量よって単位は、[g]とか[kg]とか
cは光速。速さの単位です!!だから[m/s]とか[cm/s]などなどです!!!

ちゅどん
33ご冗談でしょう?名無しさん:02/09/23 10:41 ID:7fImjJ31
>>31 そのために単位系というもんがあるの。
MKS単位系っていうのがよく使われるでしょ。MKSは
メートル、キログラム、セコンドのこと、この単位系を使うと、
エネルギーはJ[ジュール]になるわけ。
34ご冗談でしょう?名無しさん:02/09/23 11:39 ID:hHqhbqoD
磁性体とかの説明で、スピンがあったんですが、これについて教えてください。

原子は核の周りを電子が回っている。で、磁性体というのはこの
スピンが整列してるから、それにより電子の運動が電流と見なせて
磁場が発生してるみたいな感じですよね?(間違ってたらご指摘お願いします)

で、疑問なんですが、電子って土星の輪みたいな周り方じゃなくて、
球全体をくるくる回ってますよね?なのに、スピンの方向がきれいにならぶんですか?
それとも、磁性体はそういうもんなんですか?

いまいち分からないのでよろしくお願いします。
ちなみに、電気系の2年生です。
35ご冗談でしょう?名無しさん:02/09/23 11:46 ID:d0joGuUM
相対性理論の本でよく測地線方程式の導き方みたいなところで
事象aとbを結ぶ測地線をほんのわずかにずらす操作を
x^μ(λ)→x^μ(λ)+δx^μ(λ)
とか書かれてるけど
事象aとbを結ぶ測地線をほんのわずかにずらすって
どういうことですか?
なんでaとbだけずれずにaとbを結ぶ測地線だけがずれるんですか?
36訂正:02/09/23 11:47 ID:d0joGuUM
なんでaとbはずれずにaとbを結ぶ測地線だけがずれるんですか?
37ご冗談でしょう?名無しさん:02/09/23 12:15 ID:zfVYcKzq
>>34

>原子は核の周りを電子が回っている。で、磁性体というのはこの
>スピンが整列してるから、それにより電子の運動が電流と見なせて
>磁場が発生してるみたいな感じですよね?(間違ってたらご指摘お願いします)

これ自体古典力学的な解釈だから厳密には正しくない。
初年度学生は量子しらないからそれで納得するしかないよ。
実際は各電子がスピンっていう磁気そのものをつかさどる量を持ってるよ。
38ご冗談でしょう?名無しさん:02/09/23 12:56 ID:???
>>35
導き方のところ、ということは、まだその段階では測地線かどうかわからない。
ただの仮想的な線。aとbを結ぶ測地線というのは、aとbを結ぶあらゆる線のうち、
作用という量が最小になるような線、として定義される。もしはじめに考えた
仮想的な線が測地線なら、その線からわずかにずれた線で作用を考えても
ほとんど変化しない。逆に、わずかにずれた線で作用を考えてもほとんど
変化しないような線を見付ければ、それが測地線だ、ということになる。

関数の極値を見付けるとき、その点が極値であればわずかにずらした点でも
関数値はほとんど変化しない、逆にわずかにずらした点でも関数値はほとんど
変化しない(要するに微分が0の)点を見付ければそれが極値を与える点だ、
というのと同じ。

aとbをずらさないのは、aとbを結ぶ測地線を探しているから。aとbまで
ずらしてしまったら、aとbを結ぶ測地線でなくなる
39ご冗談でしょう?名無しさん:02/09/23 13:16 ID:d0joGuUM
>>38
レスありがとうございます
40ご冗談でしょう?名無しさん:02/09/23 13:21 ID:d0joGuUM
クリストッフェル記号の添え字の部分がややこしくて分けわかんないです
どうしたらいいですか
41ご冗談でしょう?名無しさん:02/09/23 14:05 ID:???
>>40
慣れるしかないです。
ところでどの計算の部分で悩んでいますか?
42ご冗談でしょう?名無しさん:02/09/23 14:12 ID:dCdHJedQ
量子力学以降、物理描像がハッキリと明確でないものが多数
存在する。というか、星や日常の物体と違い、その対象自体
暗黙の了解で済ませられないほど、アナロジーでうまく説明
できないほどぼんやりとしているから。物理学者は、描像よりも
数学的な性質のほうに信頼をおき、描像はあくまでも補助的な
ものだと思っているためか、描像がどうだということには
あまり興味がないらしい。

さらには、場の理論になると、粒子というのは、常に安定した存在
ではなく、絶えず生成・消滅したり、絶えず別な粒子の吸収・放出を
繰り返しているものという描象になる。

そのとき、エネルギーという量がその混沌とした中での数少ない
不変量の一つとして重要な量となる。

しかし、生成されたり消滅したりと粒子にとって忙しいのが現実
だとしても、エネルギーが低いレベルに留まっている限り、その
生成・消滅や吸収・放出の割合(確率)も少なくなり各粒子も
それなりに安定する。

あと、スピンについても、自転しているという描像かどうか
という定義そのものはない。あくまでも、角運動量と同じ性質がある
ということから、その粒子が全体として(より基本的な粒子で内部は
構成されていても全体を見て)自転しているようなものだろうと
アナロジーで考えているだけである。

描像は、あくまでも数学的な性質から類推することにしましょう
という消極的な態度であるのは事実。そのため、最近の物理学者は
おとなしい。
432チャンネルで超有名:02/09/23 14:15 ID:pkYvzjmF
http://tigers-fan.com/~pppnn

女子中高生とHな出会い
  ロリロリ児童とHな?
  2チャンネルで超有名
44ご冗談でしょう?名無しさん:02/09/23 16:22 ID:???
gradとdivの違いがわかんねーよ。
理系やめたほうがいいかな?
45ご冗談でしょう?名無しさん:02/09/23 16:38 ID:???
たぶんね。
46ご冗談でしょう?名無しさん:02/09/23 16:42 ID:???
>>44
gradはスカラー場に作用して、演算結果はベクトル場になる。
divはベクトル場に作用して、演算結果はスカラー場になる。

電磁気でいきなり出て来て、記号が似てるので戸惑ってるだけでしょ。
ベクトル解析の本でも読みなさい。



4712:02/09/23 16:43 ID:UOv7mZYr
>>16
遅れてすいません。すっきりしました。
そうかエネルギー保存使えばよかったんですね。勉強になります。
実際はマッハ超えて衝撃波出るくらいもっと速いかと思ってました。
ありがとうございました。
48ご冗談でしょう?名無しさん:02/09/23 16:49 ID:/YwyFe8D
http://tv2.2ch.net/test/read.cgi/morningcoffee/1032757269/
↑にあるプランク長ってなんですか?
49危(゚Д゚)機→行列:02/09/23 16:58 ID:b2zdbKRB
http://kitech.mods.jp/imgbbs/img/files/1032767742.png
↑ベータートロンです。
↑のベータートロンで、-電荷をもった質点を加速させる向きに力が
かかっているらしいのですが、どのように考えたらよいのでしょうか?
右ネジの法則で考えると、電流の向きが-電荷の運動方向と同じ=
-電荷は減速する。となってしまい、よくわかりません。
どのように考えたらよいのでしょえか?
よろしくお願いします。
50ご冗談でしょう?名無しさん:02/09/23 17:37 ID:KXBvXjOR
age
51ご冗談でしょう?名無しさん:02/09/23 17:47 ID:79yNsGJ6

  理論研究者って、一般人からみれば金食い虫なんだろうな
52ご冗談でしょう?名無しさん:02/09/23 17:55 ID:???
>>49
>右ネジの法則で考えると、電流の向きが-電荷の運動方向と同じ
電流の向きは負電荷の運動方向の逆と定義されていますが???

電場の向きの間違い? 何の右ネジの法則を考えたのか知らないけど、
負電荷が力を受けることによって発生する電流による磁場は、与えられた
磁場変化を打ち消す向きでなくてはならない、ということからも、
発生する電場の向きがどちらでないといけないかは予測できる。

∇×E=-∂B/∂tを積分すれば、定量的にも発生する電場が求められる。

結論は、その図は正しい
5334:02/09/23 19:12 ID:hHqhbqoD
>>37
そうですか。どうもです。来年量子する予定(選択なので取れたら)
なので、それまではこれでいいっか…
54ご冗談でしょう?名無しさん:02/09/23 20:38 ID:Cg521t8J
すみません。以下の質問の答えをお願いします。

半径20cmの歯数20の歯車A、半径10cmの歯数10の歯車Bがあります。

(1)AとBを互いに接する様に配置し、どちらかを回転力を加える場合、
Aを入力にしてBを出力とした方が出力トルク(モーメント)が大きくなると思うのですが
正しいのでしょうか?

(2)AとBを同心軸上に固定配置し、どちらかに回転力を加える場合、
Aを入力にしてBを出力とした方が出力トルクが大きくなると思うのですが
正しいのでしょうか?

(3)これらの内容について詳しく解説しているWebページがあれば
是非教えてください。
55ご冗談でしょう?名無しさん:02/09/23 20:53 ID:???
(1)BのトルクはAに対し1/2倍
(2)AもBも同じ軸なので軸のトルクは一緒(他の軸とのトルク比は回転を伝える機構による)
56ご冗談でしょう?名無しさん:02/09/23 21:22 ID:Cg521t8J
>>55
(2)のAの円弧に接する方向のトルクではどうでしょうか?
57ご冗談でしょう?名無しさん:02/09/23 21:24 ID:HvtefCSp
就職はあるの?
58ご冗談でしょう?名無しさん:02/09/23 22:17 ID:???
>>56
貴方はトルクを理解していない。
Aの円弧に接する方向のトルクって何?
59ご冗談でしょう?名無しさん:02/09/23 23:53 ID:25ygV2+W
>>41
オイラーラグランジュの式に代入したとき
メトリックテンソルが共変だったり反変だったりしてたり
その添え字がそれぞれ分けわかんない変換をされてたりするところです
60ご冗談でしょう?名無しさん:02/09/24 00:13 ID:???
>>59
まぁ、慣れるしかないわな。
導出をきちんと追えばわけわかんなくはない
それをせずに闇雲に丸暗記しようとするとわけわかんないかも
61ご冗談でしょう?名無しさん:02/09/24 00:15 ID:???
2つのコイルを相互誘導させたとき
誘導された方のコイルによる磁束の変化は
誘導した方のコイルに何も影響を与えないのでしょうか?

よろしくお願いします
62ご冗談でしょう?名無しさん:02/09/24 01:24 ID:???
一方のコイルに電流を流し、他方のコイルに
誘導電圧が発生して電流が流れてそのために
磁束が発生すればその磁束が最初のコイル
の磁束に影響を与えて起電力が変化します。
電磁気学の相互インダクタンスの項に説明があります。
63ご冗談でしょう?名無しさん:02/09/24 02:04 ID:TqJogmDi
ベクトルを座標変換するときは共変ベクトルと反変ベクトル
が存在するらしいけどまったく理解できません
共変ベクトルがスカラーで反変ベクトルがベクトル
とからしいけどそもそもベクトルってのはもともと
スカラーがついてるものではないんですか?

6461:02/09/24 02:11 ID:???
62さん、ありがとうございます。
できればそれが書いてある教科書を教えてもらえませんか?
65ご冗談でしょう?名無しさん:02/09/24 02:14 ID:???
>>63
まったく理解できてないのは3行目以降でよくわかりますた。

基本概念の理解が疎かな段階で先に進んでも、3行目以降のような
意味不明な泥沼が待ち構えているだけです。スカラー、ベクトル、
共変、反変、座標変換...それぞれの意味をまずしっかり掴みましょう。
6662:02/09/24 02:37 ID:???
>>64
たとえば電気工学の学生の皆さんお持ちの電気学会の
「電磁気学」でもなんでも
およそ電磁気学の本ならば必ず載っております。
これはもう保証いたします。そこには、
2つのコイルのインダクタンスをL1,L2として
相互インダクタンスをMとすると云々と書いてあるはずです。
67ご冗談でしょう?名無しさん:02/09/24 03:35 ID:UzecC4so
どなたかcotXについて教えてください
定義とか読みとか
6867:02/09/24 03:45 ID:???
もうしわけないがわかっちゃいました
69ご冗談でしょう?名無しさん:02/09/24 05:23 ID:bwimihCp
>>65
だからそれをいろんな本を読んでも理解できなかったから
ここに質問したわけですよ。
何かヒントになるようなこととか教えていただけないかと。
7069:02/09/24 06:04 ID:bwimihCp
ほんとに誰か教えてください
もう一週間以上も共変ベクトル、反変ベクトルのところで
停滞してます。
だれか、、、、おねがいします、、、、
先に進みたい。。。。。
7165ではないが:02/09/24 06:08 ID:???
ヒントですか?
共変ベクトルは反変ベクトルに作用してスカラーを作るものだと考えてください。
反変ベクトルが縦ベクトルなら共変ベクトルは横ベクトルでしょう。そのとき、
作用にあたるものは内積と考えればいいです(与えられた計量による)。
スカラーは座標変換に対してその値を変えないもののことです。
おおざっぱな言い方をすると、こうです。反変ベクトルが座標変換に伴って
変換を受けると、共変ベクトルは、それと逆の変換を受けなければいけません。
なぜなら、新しい共変ベクトルを新しい反変ベクトルに作用させたときに、
古い座標のときと同じ値をとらなければならないからです(値はスカラーだから)。
理解している方々へ・・・言い方がおかしいところとかつっこまないでね。僕
はちゃんと理解してますよ。できるだけ簡単な言い方をしようとしてこういう
表現になってるわけで。
7269:02/09/24 06:44 ID:bwimihCp
>>71
曲面での接線方向のベクトルが共変ベクトルで
法線方向のベクトルが反変ベクトルって事ですか?
違いますか?
違いますよね。。。
73ご冗談でしょう?名無しさん:02/09/24 07:47 ID:???
トイレに貼ってあった反原発のチラシに「槌田エントロピー論」ちゅーのがあったんですけど、
なんですか?これ。最近の統計の教科書の索引には見当たらないし、
ググってもこの論そのものを説明しているサイトは見つからなかった…。
第五の力や幽子のように微妙に香ばしいんですけども。
74ご冗談でしょう?名無しさん:02/09/24 07:50 ID:???
>>72
縦とか横ってのは、行ベクトルとか列ベクトルとかだよ。
行列の掛け算を思い浮かべるとわかりやすいのかな。
7571:02/09/24 07:53 ID:???
違いますね。。。
一応吉岡書店の「物理学における幾何学的方法」をおすすめしておきます。
これに曲面、多様体のことが直感的な言い方でいろいろ書いてあります。
僕はテンソル、共変・反変ベクトルはこれで理解しました。
それとももう読みましたか?

もう出かけなくちゃいけないのでレスできませんが、理解できるよう
頑張ってください。
7669:02/09/24 08:09 ID:bwimihCp
>>75
ありがとうございます
本屋で探してみます。
77ご冗談でしょう?名無しさん:02/09/24 16:57 ID:???
age
78くだれねえ質問ボイルの法則:02/09/24 16:59 ID:qktl5SoT
問題:水面下30mのところで発生した泡は、
水面に達する直前には体積が何倍になるか。
ただし、大気の圧力を1.0atmとし、
水の温度は一定で、
水深10mごとに1.0atmずつ水圧が増えるものとする。

答え:4×Vo=1×V
      故にV=4Vo
どうして4倍するのかは分かるんですが、
どうしてVに1をかけるんですか?
教えて下さい。
79ご冗談でしょう?名無しさん:02/09/24 17:11 ID:???
それがわかんないなら、どうして4倍するのかも分かってないじゃん。
80ご冗談でしょう?名無しさん:02/09/24 17:31 ID:???
これ、化学の問題じゃないの?
まぁいいけど。

ボイルの法則に条件をそのまま代入すればいいだけでしょ。
81ご冗談でしょう?名無しさん:02/09/24 17:42 ID:???
3を掛けずに4を掛けたのがわかれば
1を掛けた理由もわかるのでは
82くだれねえ質問ボイルの法則:02/09/24 18:05 ID:qktl5SoT
あっ、大気の圧力をかけてんのか。
分かったぞい。
ちなみに物理の問題だぞい。
83ご冗談でしょう?名無しさん:02/09/24 18:16 ID:???
物理でボイルの法則習うんですか。
私は化学で習いました。
確かにどっちで習ってもいいモノですね。
84ご冗談でしょう?名無しさん:02/09/24 18:57 ID:9JCXHcJH
剛体の球が剛体の全く歪みの無い平面の上に乗っている場合
球と面が接している一点には無限の圧力がかかるのですか?
85ご冗談でしょう?名無しさん:02/09/24 19:03 ID:???
>>84
そりゃそうだが深く追求してはいけない。
86ご冗談でしょう?名無しさん:02/09/24 19:23 ID:???
>>84
>>85
何の話?自作自演?
87ご冗談でしょう?名無しさん:02/09/24 19:25 ID:tg3bLZb1
皆さんは毎日何時間くらい勉強してますか?
88ご冗談でしょう?名無しさん:02/09/24 20:37 ID:Wb9WoSTe
 TVで、女性でも車を紐?綱?で引けるっていうのを実際に実験で
出来るというのを先般、見ました。。。どういう道理、理屈で出来るのでしょうか?
教えて下さい、。。気になって夜も眠れません。
89ご冗談でしょう?名無しさん:02/09/24 20:44 ID:???
>>88
足の裏にかかる静止摩擦力が大きければOK。
摩擦力は
垂直抗力×静止摩擦係数
で決まるわけだが、垂直抗力を大きくするために重い物を背負ったりすれば
びっくりするほどの重いものを引っ張ったりできるワケ。
静止摩擦係数を大きくするために滑りにくい靴をはいたりするのも有効。
90ご冗談でしょう?名無しさん:02/09/24 20:54 ID:Wb9WoSTe
早速のレスありがとうございます!
TVで、そんな感じで紹介・説明していたのを見られた方おられます??
いたら、どの放送局のどの番組でしたでしょうか??是非、教えて下さい。
91ご冗談でしょう?名無しさん:02/09/24 20:59 ID:Wb9WoSTe
追伸:2ちゃんねるでも、(でもって言ったら何ですが)こんな親切な方も
おられるんだなあと感動致しました。 89の方
92ご冗談でしょう?名無しさん:02/09/24 21:20 ID:VS5K1Tqe
>>89
でも結局物体の静止摩擦力以上の力がないと駄目だよな?
93ご冗談でしょう?名無しさん:02/09/24 21:33 ID:Wb9WoSTe
 TV見られた方おられますー??
94トヲリスガリ:02/09/24 22:23 ID:7UOX8Oph
>>88
ちうか、そんな事テレビみて感動してないで、そこらの車押して見ろよ。
あんなモン簡単に動くんだよ。アホゥか?
#だからサイドブレーキなんてモンが車についてんだよ。
95コギャルとH:02/09/24 22:28 ID:e/WHD5Z8
http://wqll.jpn.ch

http://sakayasan.net/zjjj1/

  中高生とHな出会い
  即アポ即H出来る
  超最高なH&Hが・・ 
96ご冗談でしょう?名無しさん:02/09/24 22:30 ID:ucttO3Dd
誰か居りますか?
質問があるのですが・・・
97ご冗談でしょう?名無しさん:02/09/24 22:39 ID:qktl5SoT
時刻t(s)における位置x(m)がx=0.50sinπt
で表される単身動がある。
この角振動数wを求めよ。

答えは3,1(rad/s)なんですが
どうやって求めるのか教えてください。
98ご冗談でしょう?名無しさん:02/09/24 22:46 ID:???
>>97
単振動って x = X sin(wt) でしょ?
何か悩むところあるケ?
99ご冗談でしょう?名無しさん:02/09/24 22:54 ID:qktl5SoT
それでも分からないんっすよ。。
馬鹿でスマソ。
w=π
これがなんの関係があるんだか。。
全然分からないんです。
100ご冗談でしょう?名無しさん:02/09/24 23:03 ID:???
πは約3.1でしょ。
101ご冗談でしょう?名無しさん:02/09/24 23:06 ID:qktl5SoT
なっ、なぬ!?
そうだったのか。。
ありがとう!!
明日、中間テストだっていうのにー
ヤバイ!!
102ご冗談でしょう?名無しさん:02/09/24 23:32 ID:21WZFFPY
バスも引っ張っていたよ、女の人が。。。
サイドブレーキしていないにしても、バスを引っ張る理屈を
TVで何と言っていたか思い出せないのです。。
僕は強迫神経症だから、一つの事が気になりだしたら、いてもたってもいられなくなってしまうのです。
 どなたか、TV観てられた方、おられませんか??
103ご冗談でしょう?名無しさん:02/09/24 23:38 ID:d4WmzLBg
小学生並みの質問ですが

二輪車は走ってると安定するけど、止まっていると不安定なのは何故でしょうか?
104ご冗談でしょう?名無しさん:02/09/25 00:03 ID:???
105ご冗談でしょう?名無しさん:02/09/25 00:40 ID:Uk1eiGuL
宇宙は特異点から始まったっていう説がありますよね?
その説が本当だとして、この膨張している宇宙の外側の観測者にはこの宇宙はいつまでも特異点のままで、膨張は永遠に始まらない様に見える。

この考えは正しいですか?
106ご冗談でしょう?名無しさん:02/09/25 01:05 ID:???
>>105
 宇宙の外側のことは誰も知らない。だから答えようが無い。
107ご冗談でしょう?名無しさん:02/09/25 01:15 ID:???
>>105 昔の理論。 特異点はなくてもいい。
108コギャルとH:02/09/25 01:15 ID:1a4i73Up
http://wqll.jpn.ch

http://sakayasan.net/zjjj1/

  中高生とHな出会い
  即アポ即H出来る
  超最高なH&Hが・・ 
10969:02/09/25 01:55 ID:67iVj+Db
>>74
たびたび質問してすいません。
数式的な形では大体わかってきたんですが
イメージとしては共変ベクトルと反変ベクトルがまったく頭に浮かびません
絵的にイメージすることって不可能なんでしょうか?
11061:02/09/25 02:00 ID:???
62さん、まだ見ていらっしゃいますか?

ちょっと聞き方が間違えてたみたいなのでもう一度質問します。

コイルA、コイルBを相互誘導させたとき
コイルA側に流れる電流はどのようにして求めるのでしょうか?

コイルAによってコイルBに起電力発生→
それによってコイルAを貫く磁束も変化→
磁束の変化分だけ起電力発生→
その起電力によって電流変化→最初に戻る

みたいな感じで誘導起電力の無限ループになってしまうんですが、
それでいいのでしょうか?







111ご冗談でしょう?名無しさん:02/09/25 02:03 ID:dmeKMDMe
>>103
ジャイロ効果
自転車が倒れそうになるとその逆の方向に遠心力が働く
112おせーて君:02/09/25 02:48 ID:???
すみません、長生きしたいんで、
加速器の中に入りたいんですが、
どこかいい所、どなたかご存知ですか?
11362じゃないけど:02/09/25 03:18 ID:???
>>110
連立微分方程式を解けばいいだけ。
ってゆーか、あなた、電磁気の本見ました?
114ご冗談でしょう?名無しさん:02/09/25 03:38 ID:???
>>112
真空だぞ
11562:02/09/25 08:47 ID:???

2つのコイルをA,Bとして、自己インダクタンスをそれぞれL1,L2
とし、相互インダクタンスをMとするとします。
Bを閉じておいて(Bの両端を短絡)、Aに電圧e1を加えてi1の電流を流したとすると
Bにも電流i2が流れます。この状態で下記の式が成立します。
L1*(di1/dt)+M(di2/dt)=e・・・・・(1)
L2*(di2/dt)+M(di1/dt)=0・・・・・(2)

116ご冗談でしょう?名無しさん:02/09/25 11:18 ID:???
>112
加速したらまわりの人の方が長生きするけどそれでいいんか?
117ご冗談でしょう?名無しさん:02/09/25 13:42 ID:???
加速しようが何しようが

長生き

はできないのでは?
118ご冗談でしょう?名無しさん:02/09/25 15:11 ID:???
加速していく……
119ご冗談でしょう?名無しさん:02/09/25 15:55 ID:???
時代の流れ
質問をさせていただきます。
砂川「理論電磁気学」の63ページ(2版)にある分極ベクトルの定義
 P(x)=∫n(x-r)p(r)dr^3/NδV
n:原子の密度分布,N:平均密度,δV:平均を取る巨視的微小体積
p:電気双極子の体積密度
これが「電気双極子の体積密度の平均」をあらわしているというのが
よくわかりません。
 P=∫pdr^3/δV
ではないのでしょうか?
121ご冗談でしょう?名無しさん:02/09/26 04:26 ID:A2CsLJe3
>>103
マジレス。

指の上に棒を立てて倒れないようにバランスとるにはどうする?
指を動かすだろ?
もし、指をじっと止めたままだったら少し傾いたら棒は倒れるだろ?
二輪車が止まっている状態ってのは、この状態だ。
タイヤの接地点が固定されているから、少し傾いたら倒れるしかない。
二輪車が動いていれば、左右に傾いてもハンドルを切って接地点を移動
させてバランスをとることが出来る。

あとは、前輪が前向きについていて、傾いた方向にハンドルが切れやすい
ようになってるし、回転している車輪を右に倒そうとする力がかかると、
車輪は進行方向右を向く。
#回転体の微小な領域について運動量と力積を考えるなりしてしてくれ。
あと、>>111が言ってるジャイロ効果ってのは回転軸が保たれるってことだが、
遠心力とは違うぞ。

さて、ここで問題。
二輪車で安定してまっすぐ走っていて、急に右に曲がるとき
どっちにハンドルを切る?
122ご冗談でしょう?名無しさん:02/09/26 04:35 ID:ZJWp8LAN
クリストッフェル記号の添え字がなぜああなっているのかわかりません
何でああなるんですか?
123つばめさん ◆LvO.T.cY :02/09/26 04:47 ID:???
勝又来未子って、見かけが良いと思いますか?わたしはあまり覚えてませんが。
124つばめさん ◆LvO.T.cY :02/09/26 06:22 ID:nqCCG3ZJ
>>123
この人は東大の物理学科にいたんだが、私と同じ講義を受けたりしたこと
はないので。教養で私が留年した年に物理学科に進学してたことを、
後で知ったということで、一般に仲間意識は物理学科の奴には持っていない
事には矛盾しない。
125ご冗談でしょう?名無しさん:02/09/26 07:36 ID:???
キチガイがとうとう一人で会話を始めたぞ
126ご冗談でしょう?名無しさん:02/09/26 09:28 ID:???
>>122
添え字の位置のことかい?
127ご冗談でしょう?名無しさん:02/09/26 09:48 ID:???
>>123>>124
             _,,,,,.....                ∧∧∧∧∧∧∧∧
        _.. -‐ ' "     ヽ ̄ノ^7__         <            >
    `ー ''"--―――-r⌒``~`゙゙`''ヘ/         <  異議あり !! >
     `ー--――ー--->  〜-、_, ',          <            >
      `ー-- .._ へ/   くてi` 〈           ∨∨∨∨∨∨∨∨
       `ー-_   | ^i        , ノ                    _.. ‐ァ=r‐''⌒゙二ニ二つ
          ヽr''ヘ、_     ,.-=ァ/                _. -‐ '"´  l l    r} } }l
          /   !、   {__//    __      . -‐ ' "´        l ヽ  、 ヽ_ノノ
          ノ       、   ̄ /-‐ ' "´/`゙ ーァ' "´  ‐'"´         ヽ、`ーテヽJ
     _.. -‐''フ|フヽr-‐ ''''フ. ̄「´      /   /                __.. -'-'"
  . ‐ '7    く/|〉-rへ. /   l     l   /            . -‐ '"´
 /  /    / |  | / ` <´        ', /      _.. -- ' "´
./   /    ヽ .| /    / iニニニ}  / 、   _,. -‐'"
    l iニニl ヽ|/   /     ノ        `/


128ご冗談でしょう?名無しさん:02/09/26 12:50 ID:???
>>122
上下の添え字の順番を気にしないこと?
それはクリストッフェル記号はテンソルじゃないから。
129つばめさん ◆LvO.T.cY :02/09/26 14:23 ID:g/mv2xyv
4次元中の3次元球面についての
http://science.2ch.net/test/read.cgi/sci/1029946423/546へのレス
球面上のα=0の点は、Θ、Ψに依存しないので、そこでα曲線は交わる。また、その点で
(Θ、Ψ)一定のα曲線と滑らかに接するものは(π−Θ、Ψ+π)一定のもので、
それら2曲線で閉曲線になる。
同様に、球面上のΘ=0の点は、Ψに依存しないので、そこでΘ曲線は交わる。また、その点で
(α、Ψ)一定のΘ曲線と滑らかに接するものは(α、Ψ+π)一定のもので、
それら2曲線で閉曲線になる。

こうすると、Θ曲線はΨ曲線と2箇所で交わり、α曲線とΘ曲線も2箇所
で交わるが、Θ曲線とα曲線は、Θ=π/2の時しか2箇所で交わらない結果になる。
だから、ある点から直交する3方向にそれぞれ進んだ物体は、
出発点がΘ=π/2の時だけもう一回、向こう側で会う。
でも球面内の点は全て同等なので、Θによらないはずなんですけど。
130つばめさん ◆LvO.T.cY :02/09/26 14:57 ID:g/mv2xyv
>129
なんだ、それでいいのか。
131ご冗談でしょう?名無しさん:02/09/26 16:27 ID:JVSbefzh
>>130
おいおい落ち着けw
132ご冗談でしょう?名無しさん:02/09/26 16:30 ID:???
>>129-130
激しく萎えるような自作自演すんな。ヴァカ
133ご冗談でしょう?名無しさん:02/09/26 16:35 ID:???
>>129-130
わざとやってんの?w
134ご冗談でしょう?名無しさん:02/09/26 17:06 ID:71W0K/IR
共変微分って何ですか?
曲がった空間におけるベクトルの微分って感じですか?
135ご冗談でしょう?名無しさん:02/09/26 17:24 ID:???
>>134
その唐突な疑問はいつ,いかにして生まれたのだろう?
背景が少しはわからないと答えようが無いと思うのだが
136つばめさん ◆LvO.T.cY :02/09/26 17:53 ID:g/mv2xyv
>>129 書き間違いの訂正:
>Θ曲線とα曲線は、Θ=π/2の時しか2箇所で交わらない結果になる。
Ψ曲線とα曲線は、Θ=π/2の時しか2箇所で交わらない結果になる。

それで極座標で表したのだから、3つの座標曲線が大円となって2箇所で直交するのは、
Θ=π/2にさらにα=π/2が必要。
質問にあった2次元球面の場合は、ある点(緯度=0、π以外)から
緯線、経線に沿って移動すると球の向こうで必ず交わるけれど、
3次元ではそういうことが言えるのはΘ=π/2の時のみなようで。
あたり前のような考察はここで終わる。
137くれい:02/09/26 18:08 ID:SRR5ZdNv
レス違いかも知れないけど教えてね 何で光はガラスの中で屈折するの?
密度が違うからとかはなしで。俺は中学生だからわかりやすく


お願い
138ご冗談でしょう?名無しさん:02/09/26 18:12 ID:DXrhvDvW
>>133
>ガラスの中で屈折する

しませんが何か?
139ご冗談でしょう?名無しさん:02/09/26 19:26 ID:???
>>138
> >ガラスの中で屈折する
> しませんが何か?

「何で光はガラスの中で屈折するの?」 の表現でもって、
>>137さんのおっしゃりたいことは、
「空気中からガラスの中へ光が入射したとき、何故 その光は屈折するの?」
ということですよね。
140ご冗談でしょう?名無しさん:02/09/26 20:07 ID:???
>>137
>>139
自作自演でしょ。そーじゃないなら139が答えてやれよ
141ご冗談でしょう?名無しさん:02/09/26 20:27 ID:???
A君が、板に乗って滑らかな水平面上を等速度運動しています。
いま、A君は手に持っていた反発係数0のボールを自由落下させ、板の上に落としました。
こういうとき、A君の乗った板の速度ははじめの速度よりある程度速くなるもんでしょうか?
142ご冗談でしょう?名無しさん:02/09/26 20:41 ID:???
>>141
なぜ速くなると思うの?
143ご冗談でしょう?名無しさん:02/09/26 20:43 ID:???
>>137
これまた状況がわからんが
凸レンズでの光の屈折の作図か?

光はレンズと空気の境界で屈折する。
凸レンズを光が通過するとき、
空気→レンズ中 レンズ中→空気
の2回屈折することとなるが、
書くのが細かいし面倒くさいので
レンズの中ほどで一回だけ屈折するように作図する。
もちろん2回の屈折分だけ1回で屈折するように書くのだ。

って 中学校でこれは習うはずなのだが
144パラリラ:02/09/26 20:58 ID:???
・・・。
145くれい:02/09/26 21:06 ID:SRR5ZdNv
>>137です。ぼくも言葉では表しにくいのですが>>139
「空気中からガラスの中へ光が入射したとき、何故 その光は屈折するの?」
ってことですね。いろいろ調べたのですが、光の二面性ってので屈折すると説明してる
所がありましたがよくわかりません。もっとわかりやすく‘翻訳‘してほしいです
146ご冗談でしょう?名無しさん:02/09/26 23:39 ID:???
>>145
あ〜

多分 理科一分野上の教科書に
「光の速さが変わるから屈折する」
って説明が載ってるはずです
(今年までは)

載ってなかったらその線で説明してある本を探してください

実は すっげー面白い事が起こっているんだけど
そこまではまだまだ準備不足ですので
そいつは後の楽しみにしてね
147ご冗談でしょう?名無しさん:02/09/27 05:15 ID:jWyY5yEL
>>135
相対性理論の本を読んでいて疑問に思いました
誰か答えて
148ご冗談でしょう?名無しさん:02/09/27 05:24 ID:???
149ご冗談でしょう?名無しさん:02/09/27 05:31 ID:jWyY5yEL
>>148
ありがとうございました
やっぱりあってましたね。
150ご冗談でしょう?名無しさん:02/09/27 06:10 ID:???
>>148
Kyoto Super University ですね。
151ご冗談でしょう?名無しさん:02/09/27 07:52 ID:IUb4tR1b
2kgの物質に1kgの力を1秒間与えると
2kgの物質の速度はどのくらいになるんですか?
無重力で、摩擦とかはなしです。
152ご冗談でしょう?名無しさん:02/09/27 08:03 ID:???
>1kgの力を1秒間与えると

1kg重だろ。言葉は性格にね。
153おせーて君:02/09/27 08:24 ID:YNTDq/Rb
みなさまありがとうございました。加速器は諦めました。
少しでも長生きしたいので将来は
ジェット戦闘機乗りになろうと思います。


もうひとつ質問。
手で鏡を持って鏡の方向に向かって光の速さで進んだとしたら、
その鏡に自分の姿は映りますか?
154ご冗談でしょう?名無しさん:02/09/27 08:35 ID:???
>>153
どう加速しようがあんたの主観時間では寿命は同じなのだが?

>手で鏡を持って鏡の方向に向かって光の速さで進んだとしたら、
その前提が間違っている
155ご冗談でしょう?名無しさん:02/09/27 08:39 ID:bOThUTCO
>>153
鏡にぶつかる
156ネタニマジレス?:02/09/27 08:40 ID:???
>>153
>ジェット戦闘機乗りになろうと思います。
たぶんならないほうが長生きできます。

事故る確率増えるし、ストレスもかかるだろうし、
宇宙線被曝量も増えるし
157ご冗談でしょう?名無しさん:02/09/27 08:41 ID:???
>>153
鏡と反対方向に飛んでいったとしたら
鏡の光はドップラー効果で光の波長が無限大になって
真っ暗にみえちゃうんじゃない?
158大鳥居つばめさん@ペルシャ群 ◆LvO.T.cY :02/09/27 10:39 ID:coxtH06e
なぜ、私の親戚は、国立大学を卒業してそのまま普通の企業に勤めれた
のだろうか?(公務員や研究職の人はいません)
国立大学を出るくらいなら、塚田先生いわく文系の職業に向いてるのだろうか?
159大鳥居つばめさん@ペルシャ群 ◆LvO.T.cY :02/09/27 10:56 ID:coxtH06e
学歴で言ったら、一番上が、東北大学化学科。他はほとんど文系。妹は短大。
私が一番、昔から頭が悪く見える。(でも私が一番、小さい頃に音楽の塾とか行ってたのか)
どうしてだろう?
160ご冗談でしょう?名無しさん:02/09/27 11:02 ID:???
>>157
面白いね、でも光源を考えるとどうかな、
光源が人、鏡とともに移動していると
やはり普通に見える。


161ご冗談でしょう?名無しさん:02/09/27 11:04 ID:???
>>158-159
スレ違いもいいとこ。学歴板でも逝け、基地害

ってゆーか、暇を取ったんではないのか?
http://science.2ch.net/test/read.cgi/sci/1030391037/726
もう出てくんな
162ご冗談でしょう?名無しさん:02/09/27 13:13 ID:???
>>153
私は全然わからないですが、おそらく自分の姿は見えないのでは?

ブラックホールに落下していく物体は実際には一瞬で落下したとしても
外から見ている人にとってはとてもゆっくり落下していくように見えるはずだし
それを考えると自分の姿が見えなくなると思う。

でしゃばってスマソm(__)m
163おせーて君:02/09/27 16:26 ID:YNTDq/Rb
>>154
あなたとボクは異なる慣性系にいるようです。
164ご冗談でしょう?名無しさん:02/09/27 16:44 ID:???
>>163
だからどう動こうが他人との相対的な時間差が生まれるだけで
おまえ自身の生きる時間が増えるわけじゃねーよ
165ご冗談でしょう?名無しさん:02/09/27 17:10 ID:IXGADt0H
他大学から京大の院行けるの?
166ご冗談でしょう?名無しさん:02/09/27 18:08 ID:???
>>160
ほんとに?
根拠をおしえて
167ご冗談でしょう?名無しさん:02/09/27 18:33 ID:???
>>166
160は当たり前のことを言ってるだけで
157の設定と全然関係ないので無視しとけ
168ご冗談でしょう?名無しさん:02/09/27 20:21 ID:QPHpUdF7
すっげー基本的なことなんですけど、物理を勉強する場合
最低限必要な知識ってありますか?
169ご冗談でしょう?名無しさん:02/09/27 21:21 ID:???
対象となる物理に合わせたレベルでの最低限の数学知識
170 :02/09/28 00:31 ID:Ovxigd7X
大学院にいきたい大学三年生の理学部のものです。
駅弁なのですが、すいせんでたまたまはいれたので頭はかなりわるいほうなんですが、
大学院にいきたいのです。勉強しようとしたんですがまったくわかりません。
友達はいるんですが、正直友達に教えてもらうより教授などにおしえてもらったほうが
効率がいいとおもうんですが、あまりに基本的なことも理解してないため
教えてくださいと教授に頼むに行く勇気がありません。
基本的なことも自分で勉強してもわからないのです。
教授などにききにいくべきなんでしょうか?
しかし、教授だってこんな僕におしえてくれるものかどうか・・・
どうすればいいでしょうか?よろしくおねがいします。
171ご冗談でしょう?名無しさん:02/09/28 00:56 ID:ueKNc2kN
重複な質問だったら、すみません。
単なる疑問なんですが、

電磁気学の分野で、物理学科の人たちが呼んでいる「電場」を、電気工学科の生徒や先生は「電界」と呼んでいます。同じことなんですが、どうして呼び名が違うのでしょうか?
172ご冗談でしょう?名無しさん:02/09/28 01:15 ID:GG7m3smp
electric field='電場'
今:field='場'
昔:field='界'
173171:02/09/28 01:28 ID:JUkEway/
>>172

なるほど。。じゃぁ、
電場・電界と呼び分けているのは日本だけなのかな。

でも、今でも電気工学科の人たちは電界って言っているのは何故なんだろう。
別に電気工の人たちが悪いって言うような言い回しのつもりじゃないんですけどね、
習慣になっているからなんでしょうかねー。 
電気学会の赤い本の電気磁気学の本や、
昔からの演習書は、まだ「電界」を使っています。
・・・実はワタシは、電気工学科に通っている学生だったりするんです(^^;(爆)
174ご冗談でしょう?名無しさん:02/09/28 01:52 ID:???
高校の教科書とかも「界」じゃない?
175ご冗談でしょう?名無しさん:02/09/28 01:55 ID:JUkEway/
>>174
そういえば、そうですよね!「電界」だった。
静電磁気学の授業で、そういえば「『電界』ともいうし物理学科では『電場』と言う。」って言って初めて電場を聞いて調べてみたらそうだったんでした。

176ご冗談でしょう?名無しさん:02/09/28 02:22 ID:???
>170
死ぬ気で勉強して分からなかったらあきらめろ。
177ご冗談でしょう?名無しさん:02/09/28 02:36 ID:uRYkbEP7
というわけでこれからは
「場の量子論」では無く「界の量子論」ということで。
178ご冗談でしょう?名無しさん:02/09/28 02:42 ID:JUkEway/
高校で「電界」と言っているのを「電場」に改めたりしたり、
そのまま使ってしまったり、結構混乱するから、
最初から「場」なら「場」って、言えばいいのにねぇ・・。
後になって「電界」を「電場」と呼ぶようになったから、
それを「いっせーのせっ」とがらりと帰るのも難しいかもしれないけどね。

でも、なんだか不思議な感じです〜 :-P
179ご冗談でしょう?名無しさん:02/09/28 03:01 ID:qL3BusYs
>178
それは電界は導線周りのさてつの模様のイメ―ジ。
電場は空間の各点に目印を打ったイメ―ジ。
学問的に原始的なイメ―ジは電界の方ジャ無いかな。
180171:02/09/28 03:08 ID:JUkEway/
>>179

お_ぉ! なるほど。 
そういうイメージってあるんですねー。
まだまだおいらも勉強不足だ。
コメントありがとーございます(^^!

もっと電界と電場の違いのことであったら教えて m_0_m
181171:02/09/28 03:12 ID:JUkEway/
>>180
>>179
ん?でも、
そういうイメージの分類をするのは、実は日本人だけなんでしょうかね。"electric field" て英語では一言だけだから、海外ではイメージの分類ってないのかな。。。なんて思ってしまったり。 :-P

しかし、そういうイメージでつかめるという考え方には同意。

182ご冗談でしょう?名無しさん:02/09/28 07:52 ID:g58OkygH
もし地球が今よりかなり速く自転してたとして
北極点でコマを地球の自転と逆の方向に地球の自転と同じ速さで回したら
コマはたちますか?
183ご冗談でしょう?名無しさん:02/09/28 11:13 ID:???
スピンって観測すると↑か↓ですよね?
ってことは観測する前は、それの混合状態。

スピンのSU(2)空間での表現では、
スピンはどんな向きでも向きうるって言ってる
人がいるのですが間違ってますよね?
184大鳥居つばめさん ◆LvO.T.cY :02/09/28 11:39 ID:???
>>183
どんな向きでも可。
状態を決めたとき、スピンが一定の値(↑か↓)を持つ方向が必ず生じる。
185183:02/09/28 11:45 ID:???
スピンが磁場中で回転するって本当ですか?
それは
|ψ>=c1|↑>+c2|↓>
の係数にどのように表れますか?
186ご冗談でしょう?名無しさん:02/09/28 14:00 ID:???
>>182
立ちます

>>185
(磁場の方向によるけど)回転します。
c1(t)=c1(0)exp(-iωt/2)
のような形

187186:02/09/28 14:37 ID:???
>>186では地球の角速度ω、コマの角速度-ω、と思って立つと書いたけど、
>>182は地球の角速度ω、コマの角速度ω-ω=0とも読めるな。だとすると立たない。
188ご冗談でしょう?名無しさん:02/09/28 18:42 ID:dyggyp7s
飛行船や風船が、浮力で上昇する時には、位置エネルギーを獲得すると思うんですが、
そのエネルギーのソースは何でしょうか?
189ご冗談でしょう?名無しさん:02/09/28 18:48 ID:???
>>188
空気の位置エネルギー
190つばめさん ◆LvO.T.cY :02/09/28 19:08 ID:pFZ3jPxc
>>189
風船内が上ることを風船内の薄い空気と外気の交換とみると、
上ったことによって、位置エネルギー(風船内)+位置エネルギー(外気)
が小さくなる。

>>186
ω=eH/2mcだと思う。別に質問者だけに回答するのではないが、
回転するというのは、磁場の方向のスピン成分が1/2になるのに、それに垂直な成分が
平均は0としても幾らか生じることに対応。
191つばめさん ◆LvO.T.cY :02/09/28 19:21 ID:???
>>190
磁場の方向によるってことは、もしかして、違った?
192ご冗談でしょう?名無しさん:02/09/28 19:51 ID:k3P9CAXv
なんでホーキングほどの人がノーベル賞を受賞してないですか。
193つばめさん ◆LvO.T.cY :02/09/28 20:08 ID:???
>>186
磁場の方向によってどのエネルギー準位になるかが決まって、
2つの近接準位のエネルギー差が一定だから、
古典的に回転するということ、だと分かった気が。ここでやめとく。
194188:02/09/28 20:13 ID:dyggyp7s
>>189-190 なるほど〜。ちょっと目鱗です。
要するに、風船が上昇も下降もせずに漂っている時には、風船全体と同じ重量の空気を
押しのけているから。ということですね?

地上に風船がある場合と上空に風船がある場合を比べてみると、
上空に風船が到達した際に、気球が押しのけた空気は、結局
さっきまでいた地上の、気球があった場所を埋めているわけですね。う〜ん。

ところで、更に疑問が沸いたわけですが、上昇中の運動エネルギーのソースは何でしょうか?
195ご冗談でしょう?名無しさん:02/09/28 20:42 ID:???
他にソースがないからやはり空気の位置エネルギー
とならざるを得ない。
196ご冗談でしょう?名無しさん:02/09/28 20:46 ID:???
>>194
空気の位置エネルギー
197ご冗談でしょう?名無しさん:02/09/28 21:42 ID:mHhz0C7O
野球で
同じスピードなのに
軽いボール、重いボールというのがあるけど
なんでなんだろう?


既出ならスイマセン。
198ご冗談でしょう?名無しさん:02/09/28 22:23 ID:???
http://natto.2ch.net/test/read.cgi/sfx/1033058967/l50
上のスレで無重力に関して議論しているやつらに
見せたほうがいいページがあれば教えてくらはい。

簡単に要約すれば、
「宇宙船の中が無重力(無重量)になるのって自由落下状態にあるからだろ?」
「それは潮汐力を無視しているな」
とのことです。
199ご冗談でしょう?名無しさん:02/09/29 01:20 ID:xOS7kQRh
>>197
完璧に芯で捕らえれば、手への衝撃は無くなる。つまり、軽い。
インパクトのとき、バットの芯から外れれば手への衝撃は大きい。つまり重い。
フォークとかナックルとかは落ちる時にデタラメに揺れるため、芯で捕らえるのが難しい。
だからこれらの変化球は「重い球」と呼ばれることがある。
200200!!:02/09/29 04:15 ID:???

201のんちゃん ◆w/wwHt/g :02/09/29 06:57 ID:???
>>192
うちの大学の先生に聞くと、イギリスではホーキングは物理学者というより
数学者とされているようです。ペンローズもそう。で、ノーベル賞には数学
賞はないからもらえないんだそうです。
202ご冗談でしょう?名無しさん:02/09/29 10:57 ID:otsPalSJ
>>201
フィールズ賞じゃいやなのかな。
203のんちゃん ◆w/wwHt/g :02/09/29 11:07 ID:???
>>202
ttp://www.ikeda-e.oku.ed.jp/sansu/child/story/fields/fields.htm
40才までという年齢制限があるようです。フェルマーの最終定理を
証明したワイルズはギリギリでダメだったらしいです。
204197:02/09/29 13:36 ID:CvczO8EK
>>199

なるほど。
そういうことなのか
ありがとうございます。。。
205ご冗談でしょう?名無しさん:02/09/29 15:49 ID:???
ローレンツ変換ってどうして時間の座標をtじゃなくてctにしてるの?
本には座標と同じ長さの次元を持った量として変換すると、変換行列が
無次元となり、綺麗な形式で表現されるからである。
って書かれてるけど、どういうこと?
誰かわかりやすい言葉で説明してくれませんか?
206ご冗談でしょう?名無しさん:02/09/29 18:10 ID:???
そのままじゃん。自分で両方やってみればわかるだろ。
207ご冗談でしょう?名無しさん:02/09/29 22:54 ID:PbbfKJT3
回折格子にレーザーを垂直に入射させて、その結果(干渉像)から回折格子の長さを求める、
ということは出来るんですか?
出来るなら、その方法も教えてください。
208ご冗談でしょう?名無しさん:02/09/29 23:00 ID:???
>>207
出来る。
方法は教科書に載っている。
209ご冗談でしょう?名無しさん:02/09/29 23:04 ID:???
207は宿題の答えを得る機会を失いますた
210207:02/09/29 23:12 ID:PbbfKJT3
>>208
今教科書を見てみたんですが、載っていませんでした・・・
ヒントだけでも良いので、教えてくれませんか?
211ご冗談でしょう?名無しさん:02/09/29 23:13 ID:DmG6sTH5
p=eBrからp=0.3Brはどうやったらでてくるのですか?
ご教授お願いします。
212ご冗談でしょう?名無しさん:02/09/29 23:42 ID:???
おい、お前らここの偏差値70を引き取ってくれ。
物理板出身だっていってるしな。
−−−−−−−−−−−−−−引用−−−−−−−−−−−−−
553 名前:無名武将@お腹せっぷく 投稿日:02/09/29 23:25
蜀物産はつぶした。
こんどは、ここだ!!

物理板なめんなよ、この低脳ども!!
−−−−−−−−−−−−−−−−−−−−−−−−−−−−−
http://hobby.2ch.net/test/read.cgi/warhis/1032422583/
213ご冗談でしょう?名無しさん:02/09/30 00:27 ID:???
>>211
ヒント:単位
214ご冗談でしょう?名無しさん:02/09/30 00:49 ID:lLJyMkA/
>>207
回折は高校でやるんだっけ?やるんだったら高校の教科書。
大学なら学部1年の一般教養の物理実験で回折実験はやるだろうなぁ。それの教科書。
あるいはブラッグ反射。固体物理の基礎的な教科書ならどれにでも載ってるはず。
215おな:02/09/30 02:21 ID:???
一本の棒に伸びていない糸で結びつけられた質量mの2つの球の半径は等しく、2本の異との長さは同じであり
球が静止している時には、2本の糸は平行で2つの球は接している。球1をxy内で糸がたるまないように
横に引く、すると球1は最下点で静止していた球2に速さvで衝突した。衝突は完全弾性衝突である。
球1が球2に与える力積の大きさをPとする。棒は水平線より角度θだけ傾いている
問い:衝突直後の球1の速度の成分、v1x,v1y,をm,v,θで表わせ。
  :衝突の際、球2に糸から加えられた大きさをP,θを用いて表わせ。
  :衝突直後の球2の速さv2をm,p,θを用いて表わせ。

------------------------------------------------
→θ /
/|
/ |
/ |
/  球1    ↑Y
               |
               |
球2

←X  0

216おな:02/09/30 02:22 ID:???
あ、ずれた。
でもだいたい分かってくれると思います。すいません。
217ユマタソ@暫定”管直”人 ◆3F0LolSc :02/09/30 02:58 ID:bbHRz+Tc
>>215
衝突直後の成分だから、
V1x=v-P*cosθ/m, V1y=P*sinθ/m

球2の加えられたものはPsinθ

速さはPcosθ/m


>>207
できると思うが。似たような問題が数年前にあったような気がする
参考文献:全国大学入試問題解答物理(旺文社)
218ご冗談でしょう?名無しさん:02/09/30 10:00 ID:???
>>201-204
数学セミナーの2002年10月号によると、来年からアーベル賞というのが始まるそうです。
フィールズ賞のような年齢制限もなく、毎年行なわれ(フィールズ賞は4年に1度)、
賞金も600万ノルウェークローネ(約1億円)と多く、フィールズ賞よりもノーベル賞に
近い性格のものを目指しているらしい。
219ご冗談でしょう?名無しさん:02/09/30 13:36 ID:???
761 :ユマタソ@暫定”管直”人 ◆3F0LolSc :02/09/30 01:12 ID:4IIlwDq4
年金は年代ごとの問題ではない。赤ん坊から年寄りまでの人間の
問題である。若者が望んでいるのは何か。お年寄りが望んでいるのは何か。
それぞれのプラス面とマイナス面を考慮した上で対策しなければならないのだ。
これは年功序列制度を否定する若者とそうではない年配の労働者との関係と
非常に良く似ている。ケーススタディから得られた私の考えでは、自己選択の
権利を企業が準備してやることである。これで双方の思考の埋め合わせができる。
だが、年金は半ば強制的であり、選択権が無い。
同様に選択権を与えてやるべきである。同じような考え方で、
企業の401kプランが例である。
220ご冗談でしょう?名無しさん:02/09/30 15:15 ID:EqkTPdb1
ふと気になったんだが、こんな実験したらどういう結果になるのか?

ドライアイスを頑丈な箱に詰め、箱を高温の炉に入れる。
ドライアイスが気化しても体積が増えないように常に外部から圧力を掛ける。
俺の予想としては固体or液体の状態で内部の温度が上がると思うんだがどう思う?

221ご冗談でしょう?名無しさん:02/09/30 15:31 ID:???
>>220
超臨界流体って物になるんだとさ
密度は流体なみで分子運動は気体なみの
激しさでね。
222ご冗談でしょう?名無しさん:02/09/30 15:55 ID:8mljghlH
水でもそういうものになりますね。
223ご冗談でしょう?名無しさん:02/09/30 16:31 ID:???
ペットボトルの分解とか。
224ご冗談でしょう?名無しさん:02/09/30 17:02 ID:6f98GmYT
疑問っつーか、FETのいい本探してるのですが、お勧めをのたまってくださいませ。
225ご冗談でしょう?名無しさん:02/09/30 20:59 ID:???
>>220
あんた相図ってものを見たことないのか
226ご冗談でしょう?名無しさん:02/09/30 21:23 ID:???
>>224
もすかすて
電解効果トランジスタのことでつか?
なぜ 工学板じゃなくてここなのでせう?
227ご冗談でしょう?名無しさん:02/09/30 21:31 ID:???
曲面s^2でのクリストッフェル記号
リーマンテンソル、リッチテンソルの求め方を教えてください
228ご冗談でしょう?名無しさん:02/09/30 22:05 ID:???
三次元球面ってこと?それとも一般の曲面?
最近の朝倉の講座「微分幾何」とか「曲面と多様体」
にイパーイ載ってる
229ご冗談でしょう?名無しさん:02/09/30 22:23 ID:???
>>228
3次元球面です
お願い!
おしえて!
230ご冗談でしょう?名無しさん:02/09/30 22:48 ID:???
>>224
ネイチャにFETの記事がでかく載ってるよ。



捏造らしいが。。。
231ご冗談でしょう?名無しさん:02/09/30 23:05 ID:lbGT5z0u
デバイ散乱を上手く説明できる人ってこのスレにいる?
232ご冗談でしょう?名無しさん:02/09/30 23:29 ID:WmeTZVQi
遠心力や慣性重力はなんで起きるんですか?


重力は重力子のキャッチボールでおきるとか聞いたんですが。
233ご冗談でしょう?名無しさん:02/10/01 00:55 ID:xg3Nvlyp
http://sports.2ch.net/test/read.cgi/ski/1032960784/55

ココに書いてる事って本当ですか??
234ご冗談でしょう?名無しさん:02/10/01 02:54 ID:VgUH2Y97
電子レベルでの時間移動は可能なんですか???
また、時間という概念を別の観点から捕えて微積で解析することは
可能なんですか???
235ご冗談でしょう?名無しさん:02/10/01 02:56 ID:???
>>234
>また、時間という概念を別の観点から捕えて微積で解析することは

意味不明。
236ご冗談でしょう?名無しさん:02/10/01 02:58 ID:VgUH2Y97
>>235
ごめんなさい物理苦手なんです
なんとなく疑問に思ったんでふ
237ご冗談でしょう?名無しさん:02/10/01 03:09 ID:N7bb3LRo
摩擦についてなんですが、
硬い物と、柔らかい物の摩擦力の考え方の違いについて教えてください。
238ななし:02/10/01 04:46 ID:sqgDiHFc
 ニュートン力学は、誕生以来、相対論、量子力学で、修正されてきました。

さて、最近雑誌を見ていたら、銀河系の運動を説明するために、ニュートン
の第二法則 「力は加速度に比例する」が、

加速度がある値(10**−10 m/s**2)より小さい場合、第二法
則の修正 「力は加速度の2乗に比例する」ように修正すると銀河の星の運
動をを、上手く説明できるそうです。(暗黒物質がいらない)

 銀河の加速度は日常生活で経験する加速度より桁違いに小さい。天の川銀
河の中心方向に向かって太陽系に働く力は、わずか 10**−10 m/s**2
程度で、地球の中心方向に向かって、スペースシャトルに働く加速度(約9.8m/s**2)
の9800億分の1です。

 修正ニュートン力学では絶対座標系を採用し座標系に作用する全加速度の
合計を加速度と定義する。ていうか地球上の相対座標系の実験では、見つか
っていない。従って地球上では実証実験ができない。ただ、すでに冥王星の
軌道の外にある惑星探査機パイオニアは、不可解な減速を受けており、修正
ニュートン力学で説明できるかも知れない。

 これって正しいの ? この場合どのような、実証実験をすれば、良いでしょうか。

わかる人教えてください。
239おしえて!:02/10/01 09:56 ID:???
宇宙って今、何歳ですか?
240ご冗談でしょう?名無しさん:02/10/01 10:33 ID:???
>>238
トンデモっぽい文章だな…。絶対座標系とか言ってるし。

ニュートン力学は修正などされていない。
量子論や相対論とは別の理論体系。
我々の世界がニュートン力学ではなく量子論、相対論に従うことが分かっただけ。

よく相対論は間違ってるとかいうアフォは
ニュートン力学の仮定と相対論の仮定をごちゃごちゃにしている。
241ご冗談でしょう?名無しさん:02/10/01 10:48 ID:???
>>240
ニュートン自身が絶対座標系(という言葉を使うのかどうかはしらんが)があると
論じていた、って話は読んだことある。バケツの議論とかじゃなかった?
242おしえて!:02/10/01 11:04 ID:???
>>241
たしかバケツに水はってグルグール回したやつでしょ。
でもってエーテルがあるんだなんだとか言ってたやつ。
243ご冗談でしょう?名無しさん:02/10/01 13:11 ID:Buq7jqQV
重力って光より早く伝わるって本当ですか?
244ご冗談でしょう?名無しさん:02/10/01 14:35 ID:???
>>243
それは初耳だ
245つばめさん ◆LvO.T.cY :02/10/01 17:13 ID:PcYCgWNy
690 :大鳥居つばめさん ◆LvO.T.cY :02/09/10 15:32 ID:KxFyDNTK
掃除のアルバイト先のやつが言ってたんですけど、

メモリ48メガってないんですか?
ハードディスク2Gが速いんですか?

そいつは、私が、全体で同じ容量のハードディスクでも、ピックアップの
数で違うのでは?と幾何学的にイメージしながら少し計算して言ったら、
何でそんな疑問が出るかさえ分かりませんでしたが。

691 :大鳥居つばめさん ◆LvO.T.cY :02/09/10 15:33 ID:KxFyDNTK
48=16+32=8+8+16+16

693 :大鳥居つばめさん ◆LvO.T.cY :02/09/10 15:49 ID:KxFyDNTK
>>690
そういう話は私から振ったわけでないのに。
実際には私より物理的にも考えてないくせに、私が考えれないだろうと思っていい加減な断定を。
(もちろん、私が東大卒でなければ、もっと私は、理解ができる頭を持ってると
思われたわけですが)

また、その時、
2^N−1=1+2+4+・・+2^(N-1) とか、
2^Nと2^Mの和が2^Lでないなら、N≠M。
(信号伝達)の基礎のL=Vtの組み合わせ。
などを、考えれないような精神状態でもかろうじてイメージして。

しかもそのぐらいの事で、その後、そいつはコンピューターの話を安易にしなくなりました。
246つばめさん ◆LvO.T.cY :02/10/01 17:18 ID:PcYCgWNy
>>245は前スレのカキコだが、
これは、私が自分で48Mのメモリで使ってるPCがあって、48で使えること
は事実なのに、48は無いよ。って相手がわたしをバカにしてきたという過程です。
他には、カップラーメンで7日のカロリーがあるとか言ってきました。私は、
そのとき一応、それだけの体積中に砂糖などがつまってたらどれくらいのカロリーに
なるか計算して見ましたが。
どうやったら、私が、そこらのレベルが高いとは言えない文系よりも、分数の計算、
温度計を見ること、PCの操作、ができないことがあるだろうか。

以上のように、仕返しは自動的に出ましたが、
掃除の仕事とその場の人の性質から、やったって無駄ですねえ。まわりに
ない良い面も出ないそういう仕事を続けたら鬱になるしかないんじゃないの?
247のんちゃん ◆w/wwHt/g :02/10/01 18:56 ID:???
>>238
その雑誌、日経サイエンスですよね。僕も読みました。

で、正しいかどうかはまだ分からないみたいですね。修正ニュートン力学
が最初に提唱されたのが1983年で、その当時はほとんどの天文学者が批判的
でしたが、たくさんの観測事実が暗黒物質の存在を仮定することなしに説明
できたことから、だんだん賛成する人も増えてきたらしいです。
しかし、銀河団の中心部分では例外が見られるらしく、まだこの理論
は完璧とはいえないようです。
実証実験がなかなかできないのはなぜかというと、地球上、ないしは
太陽近傍では背景加速度(惑星、太陽の重力などによって引き起こされる)
が大きすぎるので、修正ニュートン力学の適用範囲である、a〜10^(-10)m/s^2
という条件が達成されにくいからだそうです。日常生活では相対論的
効果や量子力学的効果がわかりにくいのと同じです。

>>240
トンデモというわけでもないみたいですよ。絶対座標系という言葉も
トンデモたちが使ってるような意味ではありませんし。マックゴーという
メリーランド大学の人が修正ニュートン力学のサイトを開いている
そうなので、一度ごらんになってはいかがでしょう。
http://www.astro.umd.edu/~ssm/mond
ちなみに僕自身はあまり信じていません。
248ご冗談でしょう?名無しさん:02/10/01 19:49 ID:???
>>247

いわゆるMONDですな。
実際は、MONDを信じている人はまだまだ少数派です。

249ご冗談でしょう?名無しさん:02/10/01 20:19 ID:HLld0Aqp
タイムトラベルが起こったとき質量保存則は成り立つんですか?
250ご冗談でしょう?名無しさん:02/10/01 21:07 ID:???
♪タイムトラベルは楽しい♪
♪メトロポリタンミュージアム♪
251ご冗談でしょう?名無しさん:02/10/01 23:03 ID:???
大好きな 絵の中に


          閉  じ  込  め  ら  れ  た 
252238:02/10/02 00:21 ID:f4uZEK0E
雑誌は日経サイエンスです。私はDQNなんで、一寸吃驚してしまいました。

物理スレにも、MONDを意識したスレが在るようなんで、興味を持って質問し
ました。答えてくださったかた、有難うございました。


253歩けみ?:02/10/02 01:11 ID:fbE3Qs7s
放射性物質から出た放射線は他の物質にあたるとその当たった物質も崩壊するんですか?
そうだと放射線がDNAぶっ壊すって話もわかるのですか?
254ご冗談でしょう?名無しさん:02/10/02 01:15 ID:H30/Xlrn
素人質問ですみません。いま現在、元素数はいくつになったのでしょうか?
元素番号112以降は、114、116で止まっている・・・で合っていますか?
255ご冗談でしょう?名無しさん:02/10/02 01:18 ID:???
Physical Review と Physics Letters はどっちの方が格が上なんですか?
他スレで質問したのですが、煽りレスしか来なかったので、マジレスが欲しいのですが...
256ご冗談でしょう?名無しさん:02/10/02 01:18 ID:???
>253
別に放射線に限らず高いエネルギーを受け取ると
結合が切れちゃうと思う。
257歩けみ?:02/10/02 01:22 ID:fbE3Qs7s
>>256
結合というのは陽子、中性子の結合のことですか?
258ご冗談でしょう?名無しさん:02/10/02 01:34 ID:???
>257
よー知らんがエネルギーが十分高ければそうなるだろう。
DNAを壊すだけなら原子間の結合切れば十分。
259ご冗談でしょう?名無しさん:02/10/02 05:09 ID:???
電子機器の電磁波吸収とかで使われてるフェライトって早い話が磁石ですよね?
つまり、普通の磁石でも代用できるということですか?
260ご冗談でしょう?名無しさん:02/10/02 05:52 ID:SgjzEBon
261ご冗談でしょう?名無しさん:02/10/02 06:43 ID:???
>>254
118(ウンウンオクチウム)は捏造だったんだよね。

>>255
分野にもよるんだろうけど。漏れのやってる分野だと、Phys. Rev. Bの論文を
引用していない論文は見たことない。Physics Lettersはたまに見るかな
という程度。だからPhys. Rev. Bの方が重要度は高いと言えるでしょう。
他の分野は全く知らない。スマン。
262ご冗談でしょう?名無しさん:02/10/02 10:30 ID:BExhz4tt
exp(iπ)=cosπ+isinπ=-1 なのに
(exp(iπ2))^(1/2)=(cos2π+isin2π)^(1/2)=1
となってしまうのはどうしてなんでしょう?同じ値にならないとおかしいですよね?
どこかでミスってるんでしょうか?
263つばめさん ◆LvO.T.cY :02/10/02 10:48 ID:dvqb+OU3
>>262
−1を2乗してから、本当は多価関数だけど正の値だけとることに
決めてある√をとるからでしょう。
Zは1価だけど、(Z^2)^(1/2)は多価なわけです。
264ご冗談でしょう?名無しさん:02/10/02 11:02 ID:BExhz4tt
>>263
ありがとうございます。
すると、この場合は(X^a)^b=X^(ab)とする通常の指数法則は成り立たないんですか?
265ご冗談でしょう?名無しさん:02/10/02 11:18 ID:???
>>261
Phys.Rev.[A-E]は分野が固定されているので、かなり専門的な話題も載る。
一方、Phys.Lett.やPhys.Rev.Lett.はあらゆる分野に対して影響が大きい
(とエディター・レフェリーが判断した)話題を載せますから、自分の興味
ある分野の話題は相対的に少ないし、ページ制限もあるので細かいところは
書いてないことが多い(レファレンスたどるの大変)。

分野を固定した場合の重要度はPhys.Rev.A-E、そうでない場合はLetter系
というところでしょうかね。
266ご冗談でしょう?名無しさん:02/10/02 11:20 ID:???
>>249
起こってなくても既に成り立っていませんが、何か?
267つばめさん ◆LvO.T.cY :02/10/02 15:18 ID:U/mVDdhM
>>264
Z^3/4=(Z^1/4)^3=(Z^3)^(1/4)は言えるけど、
今の例では、(Z^1/2)^2=Z≠(Z^2)^(1/2)であるし、
その関係式はいつでも成り立つわけでは無いです。
(Z^1/2の2つの値のうち1つをとることにすれば、成り立つけど)

なお、多価関数(無理関数、周期関数の逆関数)の場合、拡張した定義域(リーマン面)
に対する1価関数にできます。そうしたら線積分の際の計算間違いも少なくなります。
例を挙げずに言葉だけで悪いけど。
268つばめさん ◆LvO.T.cY :02/10/02 15:35 ID:U/mVDdhM
>>267 訂正:
>(Z^1/2の2つの値のうち1つをとることにすれば、成り立つけど)
少し違う。Z^2があるから、Zの値によって主値を場合分けしないと。ということで。
269 ◆P8y0CLuo :02/10/02 19:15 ID:uOtUKNb8
いきなり本題から失礼します。
大学で物理学的定数を求めよう、という実験に誘われました。
なんでも、1ヶ月くらいで測定器具を作って、実際の物理常数(など)を何か
求めてみよう、というゼミみたいな話だそうです。

面白そうなので参加してみようと思っているのですが、出来るだけ安価で、
しかも1ヶ月程度で準備・測定出来る物理常数ってどんなものがあるでしょうか?
前回は重力加速度を、振り子を使って求めたようですが、
もっと規模が大きく、可能なら実験結果が可視的なもの(大学祭に出すため)を考えています。

私は理学系の親しい教官がいないので、ここで質問しました。
私は今のところ、地球物理的な数(太陽質量とかパーセクとか)を考えていますが、
皆さんならどのような対象を選ばれるか教えて貰えますか?
また、その測定方法も簡単に教えて貰えると(例えば三角測量で求まるよ、とか)参考になります。
よろしくお願いします。
270ご冗談でしょう?名無しさん:02/10/02 19:26 ID:???
>>269
光速なんかどう?
271ご冗談でしょう?名無しさん:02/10/02 19:27 ID:???
>>269
光の速さとかどうですか
272ご冗談でしょう?名無しさん:02/10/02 19:31 ID:???
>>光速
回転歯車(フィゾーだった?)
273 ◆P8y0CLuo :02/10/02 19:44 ID:???
>>270-271
をを、なるほど。失念していました。
光速でしたら、高校の時に実験器具の説明も受けた気がします。
>>272さん指摘の通り、確か歯車とハーフミラーを使った実験でしたよね。
確かにかなり格好いいアイデアです。ヒントありがとうございます。

他にも何かありましたら是非教えてください。

#私が新たに思いついたのは、電磁気などの場はコンピュータでデータを視覚的に表示したり出来るので、
#超電導(超伝導?)を作って(簡単に作れるのかどうか知らないですが)
#電磁気関係の何か適当な定数を求めてみる、なんていう下らない物です。
274249:02/10/02 20:09 ID:ATmaYZPX
>>266
核反応を忘れてました。
どうもありがとうございます。

でも他にも何かあるのかな?
小さい空間を考えたらどうなるんでしょう?
275稚拙な質問で申し訳ないが:02/10/02 21:02 ID:yGMX2zF1
金の原子核を衝突させたとき、五兆度の温度に達するために必要な金原子核の速度はいくらなのでしょうか?
光の?%でも良いです。
276ご冗談でしょう?名無しさん:02/10/02 21:18 ID:rQMoUaor
摂氏度とか華氏度とかが入ってくる前に
日本では温度の単位はあったのでしょうか?
277ご冗談でしょう?名無しさん:02/10/02 22:16 ID:H0HTM6Xa
物理ど素人なんですが、最近友達の薦めでエントロピーの本を買いました。
とても面白く読ませてもらったのですが、どうしても熱力学第2法則がふに落ちません。
なんかただ、確率は正しいですよ、って言ってるだけのような気がしてきました。
熱湯と水を混ぜるときに、ある時点での熱湯と水の分子の運動とかが全てわかるとすれば
次に熱湯と水になる時がいつだ、ってのはわかるわけですよね。
エントロピー(確率みたいなもん)が低い方から高い方に行こうとする、とか
自然界の現象の移動方向とか書いてあったのですが、イマイチぴんとこないでいます。

どなたかこのモヤモヤを振り払う例え話かなにかありましたら教えてくださいませ。
(わからないとこがよくわかってないので、説明がわかりにくくてすいません)
278 ◆NNKTuYyY :02/10/02 22:17 ID:???
>>275
どこまで考えて、どこがわからない?(>>1 参照)
279ご冗談でしょう?名無しさん:02/10/02 22:59 ID:???
>>278
おまえもよく読め
280ご冗談でしょう?名無しさん:02/10/02 23:36 ID:???
>>279
教えて貰えないからと言ってその対応は良くない
281269 ◆P8y0CLuo :02/10/03 00:30 ID:0fmGGgxE
さて、ちょっと光速について調べてみました
フィジーの実験は大がかりすぎるので、するならばフーコーの振り子が良さそうですね。
あと、アイデアとしては飛行機に乗って時計の遅れを調べて光速を割り出す、というのも
有りかなと思いましたが、金がかかりすぎるので実現性は無さそうです。
それにしても高校生向けの実験としてはかなり良いですね。

他にも、中学生向けに地球・月・太陽の距離や大きさや重さをはかる実験を考えています
これらは三平方の定理だけで説明できるので、面白いかと思います。

さて、後は大学生向けの電磁気とか大気とかの実験を考えているのですが、
「何らかの(天文学含む)物理定数を求める」というのが前提になるので、なかなか思いつきません。
どなたか、電磁気や大気(流体)関係で、低予算で実際に物理定数が求まるような
実験をご存じの方、もしくはアイデアをお持ちの方がいらっしゃったら是非御教授下さい
282269 ◆P8y0CLuo :02/10/03 00:31 ID:???
>281 修正 ×フーコーの振り子 ○フーコーの光速に関する実験
283名無しさん:02/10/03 01:49 ID:G6O97NI4
学校で教わったのですが、レーザーは必ず直進する(広がらない)という先生と、
若干は広がるという先生がいて、どっちがウソツキかわからずに困っています。
どうなんでしょう?
284ご冗談でしょう?名無しさん:02/10/03 02:05 ID:???
>>283
有限なビーム径なら、必ず広がる。
285255:02/10/03 03:19 ID:???
>>261 >>265
ああ、なるほど。じゃあたとえば Phys. Lett. Bは、素粒子・原子核が
同じ巻に掲載されている点がミソなんですね?
286ご冗談でしょう?名無しさん:02/10/03 03:27 ID:???
>>283
広がるにもう一票。
でも、他人に聞いて納得するんじゃなくて、
ちょっと高いけど、レーザーポインタといって、
黒板の指し棒のかわりにレーザーで指すものが売ってるから、
それを何とか手に入れて二人の先生のまえで
実演してみたら?
287ご冗談でしょう?名無しさん:02/10/03 06:42 ID:/9f7JpIg
>>277 その本に「マックスウェルの悪魔」の喩えは載ってなかったの?
>ある時点での熱湯と水の分子の運動とかが全てわかるとすれば
「観測自体が観測される系に影響を与える」から↑は成立しません。
この例で言えばすべての水分子の運動を観測するために光子なり電子なりを
水分子にぶつける必要がありますね。その結果水分子の運動は変わりますから
「観測が無かったときになるはずだった」結果とは違ってきます。

エントロピー増大で??となるときは「閉じた系の」を忘れてることが多い。
電気冷蔵庫は、エネルギーを投入することで庫内のエントロピーを現象させて
いますが、(冷蔵庫+それのおかれた部屋)のエントロピーは増大しています。
288ご冗談でしょう?名無しさん:02/10/03 08:27 ID:???
>>287
量子力学を持ち出すこともないと思うんだけど。
どうだろ。
289ご冗談でしょう?名無しさん:02/10/03 08:45 ID:???
紫外線を当てると半導体になる絶縁体があるそうですが。
バンド構造が変化してギャップが小さくなるのでしょうか?
キャリアは何でしょうか?
290大鳥居つばめさん ◆dpLvO.T.cY :02/10/03 15:09 ID:vIW9k8VE
>>277
「等温」な状態から「水とお湯に分かれた」状態に変化する確率のほうが、
逆の場合の確率より小さいとして、分かれた状態からの変化を計算すると、
どうしても長時間の後に等温になって、再び分かれるということはない。(H定理を参照)
しかし、力学的には、ポアンカレ周期(膨大な時間後)で再び分かれる。

>>289
分からん。真性半導体になるんでしょうか?不純物ができるんでしょうか?

あげ
291ご冗談でしょう?名無しさん:02/10/03 15:12 ID:???
ホログラフィック原理について誰か簡単に教えてください
比喩的にでもかまいませんです
292264:02/10/03 15:55 ID:Bc0Thz9n
>>267
ありがとうございました。
リーマン面って複素関数で出てきますよね。
じっくり勉強してみようと思います。
293ご冗談でしょう?名無しさん:02/10/03 15:57 ID:Q+QNseSM
>281
平行電線に電流流して生じる力計れば空間の透磁率が出る
平行電極に電圧かけて生じる力計れば誘電率も出そうだけど計るの大変そう
294ご冗談でしょう?名無しさん:02/10/03 19:31 ID:???
>>291
ホログラフィの原理ってこと?
ttp://www2h.biglobe.ne.jp/~mrcisc/japan/column2/kadoma98/98.07.23.htm
とか
295291:02/10/03 20:01 ID:???
>>294
お返事ありがとうございます。でも、そうじゃなくて、
トフーフトのホログラフィック原理なんです。
ホーキングとかベッケンシュタインとかの宇宙論と関係あるらしいのですが・・・
296269 ◆jLP8y0CLuo :02/10/03 20:24 ID:0fmGGgxE
>>293
アイデアありがとうございます。なるほど、透磁率は思いつきませんでした。
空間の透磁率を求める過程では、あまり視覚的に面白い数値データが導き出せなさそうですが、
それよりも本来電線(電極)からどのような電磁場が出来ているのかをシミュレートしてみると
実験用具の側で面白い紹介が出来るかもしれません。
実際に、平行電線から透磁率を出す実験がどのような物かは解りかねますが、
今から調べてみたいと思います。有難うございました。

後は大気関係で1ヶ月位で準備の出来る実験があるといいのですが、
大気関係は実験器具を作るのが大変そうだから無理かな。
297ご冗談でしょう?名無しさん:02/10/03 22:24 ID:rEsgVFTj
金の原子核1個の比熱はいくらでしょうか?
また、原子核同士がぶつかった際に、熱に変換される運動エネルギーの割合はいくらなのでしょうか?
それがわかれば、ぶつかるのは数個の原子核だろうと推測して五兆度から金の原子核が持っていた運動エネルギーが割り出せ、運動エネルギーと原子核の質量でスピードが割り出せると思うのですが。
中学レベルで済みません。
298中卒塗装工:02/10/03 22:42 ID:???
>>291>>295
低次元の場の量子論はより高い次元の古典重力理論と同等であるという考え方。
例えば3次元の理論が2次元の理論で記述されたりするのだが、
それを立体撮影のホログラムと例えているのが語源。

AdS/CFT一致が代表例。
BFSSやIKKTといった行列模型もホログラム原理の例という人もいるようだ。

重力を離れて一般に、パリジ・ウー確率過程量子化やランダムスピン系の様な
古典系のDimensional Reductionが量子系を与えるという話とも強くつながっている
とも思う。

古典重量の解であるブラックホールの(ベッケンシュタイン)エントロピーは、
ブラックホールの体積ではなく地平面の面積に比例しているが、その事から
「ある領域の体積の古典情報は、領域の表面のみの(量子情報)で記述できる」
という着想を得たようだ。

初期のトホーフト gr-qc/9310026 やサスキンド hep-th/9409089 とか
TASIレクチャー hep-th/0002044 あたりを探ってみてはどうだろうか。
299ご冗談でしょう?名無しさん:02/10/03 22:51 ID:???
>>297
 熱ってどんなものかは分かってる? 原子核一個の比熱も出そうと思えば出るけど、
君の問題には役に立たないと思うぞ。
300269 ◆jLP8y0CLuo :02/10/03 23:02 ID:???
ちなみに金の比熱は0.031度/kgだったと思うので、どうしてもというなら計算すれば出ますね。
でも原子核1個に比熱って定義されるんでしょうかね?
301269 ◆jLP8y0CLuo :02/10/03 23:11 ID:???
激しい単位ミス。0.031(cal/mol℃)
302中卒塗装工:02/10/03 23:16 ID:???
>>275>>297
五兆度ってのをT=0.5GeVと読みかえて、金の原子核の衝突とくれば、
これは思いっきりQGP転位を目指したBNLのRHIC実験ですね。

RHICの場合、100AGeV(核子あたりエネルギ)くらいで重心衝突させるので、
核子質量を1GeVとすると γ=100 くらいとなり、
原子核の速度は光速の 約0.99995% となりますね。
303ご冗談でしょう?名無しさん:02/10/03 23:17 ID:???
>>300-301
5兆度まで比熱がその値のままとは思えんが。

>>297(=>>275?)は何の温度が5兆度なのかはっきりさせること。
RHICのAu-Au衝突実験の話か?
304中卒塗装工:02/10/03 23:18 ID:???
>>302 修正 0.99995% → 99.995%
305303:02/10/03 23:19 ID:???
>>302
ケコーン
306ご冗談でしょう?名無しさん:02/10/03 23:38 ID:HXcUA4ag
高校で実験レポート書くんですけど、何かいい実験ありますか?
学校でできて高校の範囲でお願いします。

307ご冗談でしょう?名無しさん:02/10/03 23:47 ID:???
>>306
俺もリアル工房の時はこういった聞き方をしてたっけなぁ?
厨房の時はしてたような気もするなぁ。
308ご冗談でしょう?名無しさん:02/10/03 23:48 ID:???
なぜ物理的な真空のエネルギーが 宇宙全体の総エネルギーに入らないのですか?
309ご冗談でしょう?名無しさん:02/10/04 00:05 ID:???
>>308
入らないの??そうだったっけ・・・?
310ご冗談でしょう?名無しさん:02/10/04 00:08 ID:???
>>308
まず、物理的な真空のエネルギーと宇宙全体の総エネルギーを
定義してもらおうか。
311質問:02/10/04 00:24 ID:F+xhKE8T
バケツに水が入ってます。
ホースの片方を、水の中に入れ、もう片方は、バケツの外にたらす。
水が勝手に流れる原理をなんていう?
312質問:02/10/04 00:26 ID:???
厳密に言えば仕組みがちょっと違うんだろうけど
こんな感じの原理ってなんて名前でしたっけ?
313ご冗談でしょう?名無しさん:02/10/04 00:29 ID:???
314質問:02/10/04 00:31 ID:???
>>313
ありがとおー
なんか色々言葉を入れてググッたんだけどなかなか・・・
どもでした。
3151:02/10/04 00:54 ID:HDlyX7t1
建築で、室内気流をといているものです。
実物と模型実験との力学的な相似則を成り立たせるためは、対象領域が等温の場合、Re数を等しくすればよいといわれます。
これは速度に対する縮率Nu、長さに対する縮率Nlとした場合、
Nu×Nl=1
となることをいいますよね(動粘性係数が等しいとき)。
ということは、実物の対象領域でたとえば吹出し口から10m/sの風が出ているとき、1/10の模型では、100m/sの風を吹かせなければ、Re数が等しくならないことになります。
摩訶不思議だとは思いませんか?
どなたかこの疑問を単純明快にお教えいただけないでしょうか
316ご冗談でしょう?名無しさん:02/10/04 00:58 ID:???
>>315
専門ではないけれど直感的には正しいような気がする。
ビルに風が当たるとき、1/10サイズの模型だと強度も何となく10倍だから
10倍くらいの風を当てるのは結構普通っぽいけどなぁ。
317ご冗談でしょう?名無しさん:02/10/04 04:43 ID:UPUF8L24
Mossbauer(メスバウアー)効果とはどんなものなのでしょうか?
だいたいでいいので教えてください。
あと、どんな本にのっているのかもわかればお願いします。
318  :02/10/04 04:44 ID:???
「ひも理論」ってなんですか?
319ご冗談でしょう?名無しさん:02/10/04 09:03 ID:???
>>317
はんちょー!
320ああああ:02/10/04 09:34 ID:cYc3MKYn
じゃがいもとかの野菜に放射線を当てると芽が出なくなって長持ちしてウヒョー、と
どっかで聞いたような気がしないでもないですが、本当ですか? それと放射能の
汚染を人為的に除去(分解?)することは可能ですか? コスモクリーナーとかの
与太は無しの方向でひとつ。
321つばめさん ◆dpLvO.T.cY :02/10/04 09:56 ID:6pjpi4C0
>>315-316
長さと速度を等倍で変えて、流体の密度も変化させるなら、力学的でなく
幾何学的にも普通っぽい相似が得られるのでは。
322ご冗談でしょう?名無しさん :02/10/04 10:38 ID:0CIzEmw7
>>289
今週号のNatureを読みたまへ。答えは全てそこにある。

>>317
ガンマ線と(主に鉄原子における)原子との相互作用のこと。
材料中の鉄原子の周りの環境が判る。
材料の解析関係の本を見ればどう?
物理板というよりは材料板ネタだ。
323ご冗談でしょう?名無しさん:02/10/04 12:51 ID:b2ItAt6A
>>322
おいおい、思いっきり物理だろ。
324ご冗談でしょう?名無しさん:02/10/04 14:11 ID:zczmKwZN
回帰分析の「y=ax+b」の式で、xが速度のときのaの評価式を
出す方法がわからないのですが、どなたか教えて頂けませんか?
325ご冗談でしょう?成仏さん:02/10/04 14:20 ID:???
上智物理さんはいったいどうなるんですか?
326ご冗談でしょう?名無しさん:02/10/04 14:37 ID:???
>>322
原子核が無反跳でγ線を放出・吸収することだ>メスバウアー効果
鉄に限らないし原子ではなく原子核。
無反跳というのがメスバウアー効果を特徴付けるキーワード。
327理系苦手:02/10/04 14:51 ID:cfPQWmTN
重量の配分がよくわからないのでお助け願います。

長方形(縦50cm×横100cm)のちゃぶ台があります。
※ちゃぶ台の重量は考えないでお願いします。
※ちゃぶ台の足をそれぞれ右前・左前・右奥・左奥という名でお願いします。
そのちゃぶ台の中心にペットボトルの水(4kg)を置きます。
すると台の四足にかかる重量は1kgずつ均等にかかりますよね?

質問1
その水を横方向には動かさず縦方向で手前に25cm動かした時、
ちゃぶ台の足にはそれぞれどれだけの重量がかかってくるのですか?

質問2
ちゃぶ台の右端に5kg・左端に2kgのペットボトルの水を固定します。
そのちゃぶ台の中心にペットボトルの水(4kg)を置いて質問1のようにすると
ちゃぶ台の足にはそれぞれどれだけの重量がかかってくるのですか?

どうかよろしくお願いします。
328知ったか工房:02/10/04 18:34 ID:???
「端に載っている」ということをどう扱うか、が疑問という事か?
329317:02/10/04 20:27 ID:???
319,322,323,326さん
お答え頂きありがとうございます。
なんとなく分かりました。無反跳か・・・
330ご冗談でしょう?名無しさん:02/10/04 21:04 ID:???
>>327
答えを教えて欲しい、というわけではないよね?まさかねぇ。
で、君がその問題のどこがわからないのかがはっきりしないんだけれど。
それと高校生の範囲はスレ違いではないのかな?
331大鳥居つばめさん ◆dpLvO.T.cY :02/10/04 21:47 ID:M3L3ELFl
>>327
台が回転しようとするかを考察する。
台の重さも極限が0になるような感じで一応、考慮する。
332理系苦手:02/10/04 22:58 ID:cfPQWmTN
>>328>>330>>331
すみません無知なのでよくわからないのですが、
4kgの物体が中心にある時と片方へずらした時に4つの足にかかる重量が
どうなるのかが知りたいのです。
それと上記で両端に異なった重量の物体がある時の4つの足にかかる重量が
どうなるのかも知りたいのです。
よろしくお願いします。
333ご冗談でしょう?名無しさん:02/10/04 23:05 ID:???
量子力学の波動関数の波の速さって光速を越えても、問題なし・・・だよね?
334ご冗談でしょう?名無しさん:02/10/04 23:24 ID:???
>>332
どう好意的に解釈しても、自分で調べたとは思えない。とりあえず図書館行って調べなさい
335ご冗談でしょう?名無しさん:02/10/04 23:32 ID:???
基本的な質問すみません。ローレンツ変換について質問です。
http://homepage2.nifty.com/einstein/contents/relativity/contents/relativity2155.html
このページの(5.17)の、t'の式が良くわかりません。
導き方は問題ないのですが、式の中の項「V_x/c^2」は、一体どういう意味なのでしょう?
例えばt=0で、V_xが0でないとすると、t'の値が0ではなくなると思うのですが、
そうすると違う座標系では時間の原点が変わってしまうのでしょうか?

特に急ぎで知りたいわけでもないので、どなたか暇なときにでも教えてください。お願いします。
336理系苦手:02/10/04 23:36 ID:cfPQWmTN
>>334
世界史と地学を専攻していたので物理とかは全然わからないんです。
とりあえず物理の本を読みあさればどこかに書いてあるんですね?
ありがとうございました。
337ご冗談でしょう?名無しさん:02/10/04 23:37 ID:???
同時に、地球の赤道付近の人は極付近の人より、
地球の自転のせいで速いスピードで動いていると思うのですが、
この両者の間で時間の経ち方が変わらない理由も是非教えてください。
338ご冗談でしょう?名無しさん:02/10/04 23:56 ID:???
>>335
 ちゃんと式を追え。そりゃV_xじゃ無くてV*xでしょ。まぁ違う場所なら
O系で同時でもO'系では同時でなくなるわけだけど。

>>337
 時間のたち方が変わらないわけじゃないぞ。なんで変わらないと思うんだ?
 
339ご冗談でしょう?名無しさん:02/10/05 00:01 ID:???
>>338
V*x、お恥ずかしい。ありがとうございます。
時間の経ち方が変わらないと思うのは、世界標準時があって、
私は全世界で秒の単位の長さは同じだと考えているからなのですが。

具体的には、飛行機に時計を持って乗ると、その時計はごくわずかですが遅れますよね?
同様に、時計を北極と赤道で比べると、赤道の時計の針の進みが北極に比べて遅くなるのではないか、
と思うのですが、実際はそんなことが無いので、それは何故だろうという事です
340338:02/10/05 00:15 ID:???
>>339
 人に聞く前に少しは計算とか調べるとかしろや。
 赤道上での速度がどのくらいかは計算できるよな?それで
どのくらい時間の進み方が変わるかも式があるんだから計算
できるな?
 その上で1秒の(現在の)定義をちょっと調べてみろ。
341ご冗談でしょう?名無しさん:02/10/05 00:19 ID:???
>>340
どうしてそんなに高飛車なのですか?
342ご冗談でしょう?名無しさん:02/10/05 00:21 ID:???
>>340
助言ありがとうございます。341は私ではないです(汗
とりあえず計算して調べてみます。また質問に来た場合はよろしくお願いします
343ご冗談でしょう?名無しさん:02/10/05 04:07 ID:???
>>341
相手が自分で何も調べないで憶測だけで物言ってるからだろ?
344ご冗談でしょう?名無しさん:02/10/05 04:56 ID:iInEPln1
>327 高校物理の「力の釣り合い」あたりかな?中学の「てこの原理」でも
考えられると思います。重さは「質点」で考えればいいですね。

Q1.中心から25CM手前は「手前の端」だから、手前の2本の脚に4Kgで、
  各2Kgずつ、奥の2本の脚には各ゼロ。
  左奥(0Kg)        右奥(0Kg)

  左手前(2Kg)  4Kg    右手前(2Kg)

Q2.右端に5Kgは右の2本に各2.5Kg、左端に2Kgは左の2本に各1Kg、
  4Kgが中央にあるときは4本に均等に各1Kg、手前にあるなら手前の
  2本に均等に2Kgづつ、それらの和が各脚にかかります。
  左奥(2Kg)    右奥(3.5Kg) 左奥(1Kg)     右奥(2.5Kg)
2Kg     4Kg   5Kg 2Kg 5Kg
  左手前(2Kg)   右手前(3.5Kg) 左手前(3Kg) 4Kg 右手前(4.5Kg)
 各脚にかかる重さの合計が水の重さの合計11Kgになるか検算してね。
334さん、ごめんなさい。なんか327かわいそうで・・
345ご冗談でしょう?名無しさん:02/10/05 05:24 ID:JnYbOO18
>>337
時間のたち方は変わるよ。
ただ「1/天文学的数字」秒位の差なので貴方のような人には分からないのです。
              
                                 かしこ
346突然ですが:02/10/05 05:28 ID:H6evsvGl
プログラミングを学んでおくことは、理論物理の研究者になる上で役に立ちますか?
347ご冗談でしょう?名無しさん:02/10/05 05:40 ID:iInEPln1
339は時間の進みかたと時計の進みかたをごっちゃに考えてるのかな?
遠方から神の目で赤道と極にある「正確な」時計を見比べたら進みかたが
違うか?という設問ならはっきりすると思うけど。これは実験できて違うよ。

1秒の定義はセシウム原子核をトンカチで殴ったときのクワ〜〜ン音が
にくくなくにしごやという音色であることですね。これが赤道でも極でも
1秒。で、1秒に一回電波パルスを出す時計を赤道と極に置き、
それを例えば月で受信すると、赤道にある時計からのパルス間隔が極からの
よりわずかに長いということがその何とかの式で計算できるはずですね。
そういう考え方の実験がされて検証済みだと思います。
348ご冗談でしょう?名無しさん:02/10/05 05:43 ID:???
>>346
役に立ちません。そんなの勉強するヒマがあるなら・・・。
349346:02/10/05 05:44 ID:H6evsvGl
プログラミングを学んでおくことは、日常生活を送る上で役に立ちますか?
350ご冗談でしょう?名無しさん:02/10/05 05:53 ID:???
>>348
役にたつでしょ。まぁ気合い入れてやる必要もないが。
お遊び程度でやっておけばいいんじゃないの。
数学は気合い入れてやっておきましょう。
351ご冗談でしょう?名無しさん:02/10/05 05:54 ID:???
オブジェクト指向なんかの考え方は役に立つかも・・・。
352ご冗談でしょう?名無しさん:02/10/05 06:04 ID:iInEPln1
>>339 世界標準時ってピュー太の時刻とかのこと?それだったら原子時計で
測ると地球の回転がだんだん遅くなるのに合わせて何年かごとに1秒遅らせる
程度のかなり荒っぽい時計ですよ。
353ご冗談でしょう?名無しさん:02/10/05 06:15 ID:???
昼飯のスパゲティナポリタンを眺めながら、積年の疑問を考えていた。
それは「なぜナポリタンは赤いのだろうか」という問いである。
簡単に見えて、奥の深い問題だ。
「赤いから赤いのだ」などとトートロジーを並べて悦に入る浅薄な人間もいるが、
それは思考停止に他ならず、知性の敗北以外なにものでもない。
「赤方偏移」という現象がある。
宇宙空間において、地球から高速に遠ざかる天体ほどドップラー効果により、
そのスペクトル線が赤色の方に遷移するという現象である。
つまり、本来のナポリタンが何色であろうとも、ナポリタンが我々から
高速で遠ざかっているとすれば、毒々しく赤く見えるはずなのだ。
目の前のナポリタンは高速で動いているか否か?
それはナポリタンの反対側に回ってみることでわかる。
運動の逆方向から観察することで、スペクトルは青方遷移し、
青く見えるはずなのだ。
逆に回ってみたところ、ナポリタンは赤かった。
よってこのナポリタンは高速移動をしていないと言える。
354理系苦手:02/10/05 08:26 ID:???
>>344
おお!答えて下さった方が・・・
これで来週からの仕事が少し進みそうです。
本当にありがとうございました。
355346:02/10/05 13:01 ID:H6evsvGl
>>350
>>351
具体的にどういうところで役立ちますか?
356ご冗談でしょう?名無しさん:02/10/05 15:49 ID:PvyYCMVA
電子土方のアルバイトするとき。
357ご冗談でしょう?名無しさん:02/10/05 20:32 ID:3yvrcWXX
>>353
懐かしい。これ読んだときはかなりワラタ
358ご冗談でしょう?名無しさん:02/10/05 21:38 ID:???
>>355 実験物理のデータ解析とか。
優秀な奴はとりたてて勉強しないでも物理に必要な程度のプログラムは簡単に書くけどね
359ご冗談でしょう?名無しさん:02/10/05 23:06 ID:???
Ta型超新星は、白色矮星の爆発だそうだが、
中性子星のaccretion diskからは超新星は生まれんの?
それもTa型というの?
通常の超新星分類には出てこないようだが。
360ご冗談でしょう?名無しさん:02/10/06 00:07 ID:aBqyMgcK
適当に張ったゴムひもに回転させた円板を正面衝突させると
運動量、力学的エネルギー、角運動量のどれが保存されて
どれが保存されないのですか?
また円板は回転しているので正面衝突した後は回転と逆の方向に
摩擦を受けてその方向に角運動量を獲得するというのは正しいですか?
361ご冗談でしょう?名無しさん:02/10/06 00:45 ID:0BS0fIIQ
>>358
>物理に必要な程度のプログラム
理論物理でプログラミングが必要になることがあるのですか?

363ご冗談でしょう?名無しさん:02/10/06 03:30 ID:???
>>361
最後に数値計算して「実験とこんなに合う」とか主張するからな。
理論屋に計算機は必要。
コンピューターを使わないのは数理物理か。
でも、日本に数理物理の専門家は何人いるんだ?
364物理板って”削除”依頼できないの?:02/10/06 06:47 ID:/MHfDRpJ
こんなにクソスレ立ってて、大掃除しなくていいの?
365361:02/10/06 07:02 ID:0BS0fIIQ
>>363
『最後に数値計算して「実験とこんなに合う」とか主張する』ために
プログラミング技術が必要なのですか?数値計算ぐらいエクセルや市
販のソフトで出来るのでは?

何度もすいません。コンピュータはよく分からないのです。
366ご冗談でしょう?名無しさん:02/10/06 08:40 ID:???
>>365
 エクセルじゃ遅いしメモリ食うし。
 単に式に数値を代入するだけじゃないんだから。
367ご冗談でしょう?名無しさん:02/10/06 08:56 ID:KqC91Vst
先生!ENGLISH板でこんなお願いしてみますた。
{
  スーパーカミオカンデが復活するそうですね。
  ( http://slashdot.jp/ 「スーパーカミオカンデの修復工事終了」 )

  上記ページで「戸塚洋二氏の声明文の一節が非常に印象深いです。」
  ってあるんでこれ
  ( http://www-sk.icrr.u-tokyo.ac.jp/doc/news/appeal.html
  を読んでみたいんですけど、英語力貧弱なので印象深くなる前に
  頭が疲れてしまいます。誰か、何て書いてあるのか教えてくれYO!
}

したらこんなレス。
{
  /.が印象深いと述べてるのは、引用している英文のことじゃん
  検出器は絶対再建するぞぉ、という強い意思を感じるっつーことで。

  戸塚氏の声明じたいは観測再開までの具体的・技術的手順と関係者への挨拶
  が主なので私ら門外漢が読んでもあんまりおもしろくないです。
}

だって。なんか感動する内容期待してたんだけど。漢の熱さみたいなさ。
物理板の諸氏はそこんとこどうよ?
368ご冗談でしょう?名無しさん:02/10/06 09:02 ID:???
>>365
まぁ何をやるかに依存するからねぇ。
Mathematicaとかで十分なケースもあるだろうし、スパコンじゃなきゃ
やってらんないケースもあるだろうし。
ただ、スパコンを使うケースでも、必要なプログラミング技術は
学部の時にやるであろうFORTRANやCの実習レベルに毛が生えた程度だ。
今は優秀なパッケージがネットに転がっているし。
もし、361が学部生で、プログラミングに恐怖心を抱いて理論へ逝くのに
躊躇しているのであれば、心配のしすぎ。
計算物理やっている漏れからすれば、実験装置の扱い方の方が、よっぽどワケワカメ。
369ご冗談でしょう?名無しさん:02/10/06 13:28 ID:lfGSHT7U
オークションで出てましたけど
タイムマシンはありえるんですか?
370ご冗談でしょう?名無しさん:02/10/06 13:30 ID:???
もうすでに発明されてるらしいね。
特許申請済だそうだ。
原理はよくわからんが。
371ご冗談でしょう?名無しさん:02/10/06 13:37 ID:y7VYKT5/
学者のプログラミングテクニックなんてただが知れてる。
腕のいい人も少しはいるんだろうが、
趣味で何年もプログラミングやってる俺から見ると
おぞましいようなプログラムなんてざら。
というかFORTRAN使ってる時点で既に...。

でも、それでも研究はできてるわけで、
そう考えると敷居はそんなに高いとは思わないかな。




実のところ、プロのプログラマでもその程度の腕の人はざらだし(ぉ
372369:02/10/06 13:49 ID:???
>>370
タイムスリップは物理学的に証明できますか?
373361:02/10/06 16:04 ID:0BS0fIIQ
>>366
物理学徒・物理学者はたいていプログラミングはできるのですか?

>>368
>学部の時にやるであろうFORTRANやCの実習レベルに毛が生えた程度だ。
私はまだ教養課程でプログラミングは必修ではないのですが、物理系の専門課程で
に進むと必修でやらされるんでしょうか?

>>371
>というかFORTRAN使ってる時点で既に...。
Javaとかいうのはいいですか?
374ご冗談でしょう?名無しさん:02/10/06 17:22 ID:???
FORTRANはライブラリが十分に枯れてて、
他の言語処理系より確実なのだよ。
過去のコード資産というのは無視できない。

あと、物理の教育過程に多くを期待するのは止めた方が良いだろう。

理論物理でJavaを知ってても直接役に立つ事はほとんどないと思われ。
むしろ、物理以外の所で役に立つ。
375361:02/10/06 17:26 ID:0BS0fIIQ
>むしろ、物理以外の所で役に立つ。
例えば?
376物理板って”削除”依頼できないの?:02/10/06 19:33 ID:UWEO23Sv
377ご冗談でしょう?名無しさん:02/10/06 19:45 ID:fg0BQEBq
>>371

じゃあ何を使えばよいのか言え!。

俺は個人的には自由度の高いCの方が好き、しかし、FORTRANに関しては
プログラミングのしやすさから使うことが多い。それから374の言う
とおりライブラリはかれているので確実性が増すし、研究室固有の財産
がFORTRANコードだったりする。その点でもFORTRANの方が使いやすい。

「反論される」と思うところをあらかじめ「ぶっ潰しておく」と、
確かに物理屋のプログラムはあんまり褒められた物でない様に「見える」。
しかし、それは当たり前。「仕様書どおり」に作るプログラム屋とは違って
物理屋は「物理」に興味がある。そのためにアルゴリズムの研究もしなくて
はならない。しかし、あくまで対象は物理だ。ここでひとつ問題がある。
実際の研究現場では数値計算に何の技法を使うかどうかをケースバイケー
スで決定する必要がある。物理を定性的に見るのか、定量的に見るのかでも
変わる。常識ある人間なら、いきなり定量計算はしない。定性的な振る舞い
を議論し、研究が進んで実際の実験と比較できる段階になった状態で初めて
定量計算になる。それまでは簡単なプログラムで良いし、作法云々よりも
定性的な振る舞いを見るほうが先。最悪なのは、371の馬鹿みたいに、
不必要に技術や作法にこだわると、折角計算したのに、NGの場合そのプログラム
に掛けた「労力が全て無駄」になる事を理解できない奴。

定量計算になれば、1ヶ月とか言う期間で計算をぶっつづける事になるので、
アルゴリズムはもとより、コードレベル、コンパイラレベル全てのレベルで
最適化する必要がある。しかし、財産を増やすためにも不必要に機種依存
性のあるコードは書けない。

結局、371は馬鹿…というより、ど素人

378377:02/10/06 19:51 ID:fg0BQEBq
ちなみに、数値計算の「安定」性を取るか「精度」を取るかによっても
変わる。例えば波動関数の時間発展と言っても、ほぼ同じ計算量でも
精度を追求するなら沢山方法がある。しかし、安定に計算を続けるなら
精度を犠牲にするのはやむを得ない。


379377:02/10/06 20:00 ID:fg0BQEBq
ちなみに絶対に止めてはいけないのなら、絶対に安全なコードを書く。
というより、速度は犠牲にする。そのためにはコンピュータも買う。
OSは安定性を重視する。なければ高くついても買う。

ちなみに俺のプログラムのレベルはかなり低い。友人のプログラマにアルファ
マシンのコンパイラのバグを言ったら、別のコンソールにTCPを使った簡易デバ
ッガを作ってくれた、俺はロウレベルでデバイスを制御するほどの実力は無い。
その意味ではレベルが低いと言われてもしょうがない。

しかし、プログラマとして優秀だからと言って数値計算のプログラムの技術
にも優れているとは言えんぞ。
380ご冗談でしょう?名無しさん:02/10/06 20:02 ID:DpKjY8ZY
ドレクスラー・ナノマシンを研究したい場合物理を学ぶべきでしょうか?
それとも化学を学ぶべきでしょうか?
381大鳥居つばめさん ◆dpLvO.T.cY :02/10/06 20:04 ID:2mv+Fkfg
>>377-379
こういう意見を見るにつけ、つまらん創造的でないアルバイト(もちろんプログラミング以外)
は憎むべき。
382ご冗談でしょう?名無しさん:02/10/06 20:06 ID:???
>>380
両方。
383380:02/10/06 20:11 ID:DpKjY8ZY
>>382
あ、学部入学の際に物理学科と化学科どちらの方が向いているかです。
僕としては、
ドレクスラー・ナノマシン→物理
プロトニック・ナノマシン→化学、生物、医学
というイメージがあるのですが・・・
384ご冗談でしょう?名無しさん:02/10/06 20:32 ID:???
>>383
とりあえず、物理いっとけば?

どっちにしても、物理、生物、化学、工学など広範な知識がいる...

どのみち、学部では何も出来ん。基礎の基礎を固めるだけ。
大学院の研究室を選択する段階でどこに行くかの検討が重要だろう。
385380:02/10/06 20:56 ID:DpKjY8ZY
ドレクスラー・ナノマシンはプロトニック・ナノマシンの延長線上に存在するのでしょうか?
それともマイクロマシンとナノマシンの違いのように、異なる路線上に存在するのでしょうか?

もし前者が正解ならば私は化学科に進もうと思います。
逆に後者が正解なら私は物理学科に進もうと思います。
386ご冗談でしょう?名無しさん:02/10/06 21:04 ID:???
>>385
将来の技術がどの様に進化するかを見極めるのはほとんど不可能。

むしろ、何故に(分子)生物という選択肢がないのか聞きたいな。
387380:02/10/06 21:16 ID:DpKjY8ZY
>>386
ナノマシンという分野に関しては、生物という分野は物理・化学のどちらにも内包されて
いるように思えるからです。それは生物物理学しかり生物化学しかりです。
物理・化学は生物分野からのアプローチに加え、原子分子操作技術、分子合成技術などナノマシン設計
以外の異なるアプローチの方法を提供してくれるように思えます。
しかし生物に属することはこの利点を失ってしまうように感じます。

これが私の選択肢が物理・化学である理由です。
388ご冗談でしょう?名無しさん:02/10/06 21:30 ID:???
>>387
完全にイメージだけで語っていますね。
もし、そのイメージに沿って研究する事を望むなら、物理学科が最適でしょう。

ただし、分子1個単位でのナノ合成で、ある程度汎用性のあるのは
「今の所」DNAを使った手法である事をお忘れなく。

物理的な手法(?)の代表例であるAFMとかでは
分子の合成が出来るとは言い難い。。 (これも今の所

全ての基礎を押えたいなら、物理学科に来るのが良かろう。
多様な方法から入りたいなら、化学科も良かろう。

どっちにしても、4年間は基礎の勉強。
389380:02/10/06 21:36 ID:viT7PJRr
>>388
どうも。
今学科漁り中です。
とりあえずForesiteも読んでみたりして、総合的に判断したいと思います。
390380:02/10/06 22:03 ID:viT7PJRr
>>388
ところで、
>全ての基礎を押えたいなら、物理学科に来るのが良かろう。
>多様な方法から入りたいなら、化学科も良かろう。
についてもう少し詳しく説明して頂ければありがたいのですが。
かなり興味あります。
391ご冗談でしょう?名無しさん:02/10/06 23:57 ID:UiebSZCR
392ご冗談でしょう?名無しさん:02/10/07 13:46 ID:MYCE8yfI
電気回路で、電流を川の流れに例えるって考え方ありますよね。
先日友人に、
「交流の場合は流れが行ったり来たり(電流が正になったり負になったり)
するが、これじゃあ電流は進んでいかないはずなのに、実際は流れてる。
なんで?」

こう聞かれて、うまく答えることができませんでした。
だれか納得ゆく説明おねがいします。

スレ違いでしたらすいません。
393392:02/10/07 13:47 ID:MYCE8yfI
スレというより、板違いのようでした。
失礼しました。
394ご冗談でしょう?名無しさん:02/10/07 14:26 ID:???
>これじゃあ電流は進んでいかないはずなのに、実際は流れてる。

進んでいかないけど流れてるんだよ。
何か問題ある?
395ご冗談でしょう?名無しさん:02/10/07 14:27 ID:???
>>392
電子の流れと電流の流れは別。電子は進んで行かないが電流は流れる。
水流モデルは直流を説明するのには非常に有効なモデルだけど、あくまで
モデルであって、モデルの適用範囲を超えてまで固執してはいけない。

電子の流れと電流の流れは別というのは直流でも同じ。電線中の電子の動きは
蟻がはうようなスピードだけど、電流のスピードは光速に近い。

>>393
スレ違いではないと思うし、ましてや板違いでもないと思うよ
396質問です:02/10/07 14:41 ID:???
オークションで金をもうける方法を物理的な視点から教えてください。
397ご冗談でしょう?名無しさん:02/10/07 14:57 ID:jb4tgMW+
これって本当か?

↓ ↓ ↓
http://www.dream-express-web.com/space-trust.htm


398ご冗談でしょう?名無しさん:02/10/07 15:58 ID:JPGaomyF
>>392
確かに、時間平均すると電流は流れていない
しかし、伝えたいのは電流ではなく電力(エネルギー)
電流がどちら向きでも、電力はプラス
399ご冗談でしょう?名無しさん:02/10/07 18:32 ID:???
>>398
おっ、本質に近づいてきたぞ。
400277:02/10/07 23:36 ID:EpM94s+f
遅レスですいません。熱力学第2法則について質問したものです。
解答ありがとうございました。

>>287
マックスウェルの悪魔の話、載ってました。
面白く読んでて深く考えてませんでした。すいません。

>>大鳥居つばめさん
なるほど、やっぱり再び別れると考えることも出来るんですね。
友達に説明しようとして、「でも100%じゃないんでしょ?」って言われたら
どうしようと思って質問した次第です。んで、100%じゃないんなら
法則って呼べるのかなぁと。

その本に「宇宙はいずれ熱的平衡状態で終わる。終わりがあるということは
宇宙の始まりがある(非可逆だから)。」とか書いてあったけど、
熱的平衡状態にあっても、あるとききれいに速い分子と遅い分子に別れて
すごいエネルギーが生じる(エントロピーが減る)こともありえるって考えてもいいんでしょうか?

401ご冗談でしょう?名無しさん:02/10/08 00:31 ID:D2g7Wt37
>>395
水流モデルは交流でも使えると思うが
水流の向きが周期的に変わったって
エネルギーは伝わるわけだから
402ご冗談でしょう?名無しさん:02/10/08 00:32 ID:D2g7Wt37
>>392
電子は流れていかないが
エネルギーは流れてる
403ペコ:02/10/08 08:12 ID:IOOEnaBc
ねー、みなさん質問です。アポロって月に逝ってないの?合成写真?
404ご冗談でしょう?名無しさん:02/10/08 11:06 ID:rtLHi6Si
物理を独学で学ぶのはそう難しいことではないんでしょうか?
405ご冗談でしょう?名無しさん:02/10/08 12:05 ID:nDl9GW4/
>>403
天文板か航空板だと思うが、1行で済むので答えてやる。
これを読め>http://moon.nasda.go.jp/ja/school/popular/story03/
406ご冗談でしょう?名無しさん:02/10/08 16:20 ID:33lEFyvQ
ガラス管に柄付きピストンを取り付け、自由に動かせるようにします。
開口端近くにスピーカーを置き、一定の音を出し続けます。
ピストンを開口端側からゆっくり引いていくと、Lだけ引いたところで最初に共鳴が起こりました。
この状態のとき、管内の空気の密度が時間的にもっとも大きく変化しているのは、
開口端からいくらの距離のところですか?
という問題ですが、どのように考えたらよいのですか?
407406:02/10/08 16:33 ID:33lEFyvQ
>>406のついでですが、
「もっとも空気が激しく振動しているのはどこですか?」
というのも、考え方がよくわかりません。
408ご冗談でしょう?名無しさん:02/10/08 17:19 ID:???
>406
定常波の腹と節がどこに出来るか考えればいいんじゃない?
409406:02/10/08 17:28 ID:7F3h6ZZg
「腹の部分がもっとも激しく振動→時間的な密度変化も大」
でだいじょうぶでしょうか?
410ご冗談でしょう?名無しさん:02/10/08 17:55 ID:???
最近 よく考えるのだが,
自分で正しいかどうか判断ができない
ってやつが多くないか?

それとも そいつにとって まったく判断ができないくらい
高度なことなんだろうか?

>>409
これだけヒントをもらって
確認する方法も思いつかんのか?

質点の変位を元に書いたグラフがあるだろう
そのグラフに密度が高いところと低いところ
つまり気圧が高いところと低いところを書き入れてごらん
411ご冗談でしょう?名無しさん:02/10/08 18:13 ID:???
>>410
グラフ書いたぐらいでわかるもんなら実験なんていらない。
412ご冗談でしょう?名無しさん:02/10/08 18:14 ID:???
>>411
おいおい・・・(;´Д`)
大阪市立大学理学部からW32.Bugbear@mm????
http://scisv.sci.osaka-cu.ac.jp/

Operating System and Web Server for www.sci.osaka-cu.ac.jp
http://uptime.netcraft.com/up/graph?site=www.sci.osaka-cu.ac.jp&submit=Examine

OS Server Last changed IP address Netblock Owner
Solaris 8 Apache/1.3.14 (Unix) 20-Sep-2001 160.193.120.1 Osaka City University
SunOS 4 CERN/3.0 10-Dec-2000 160.193.120.1 Osaka City University

http://yuri.robot.mach.mie-u.ac.jp/~virusstat/today_log.html
2002-10-08 13:58:47 W32.Bugbear@mm< [email protected] >
2002-10-08 13:58:56 W32.Bugbear@mm< [email protected] >
2002-10-08 14:01:56 W32.Bugbear@mm< [email protected] >
2002-10-08 14:14:28 W32.Bugbear@mm< [email protected] >
414ご冗談でしょう?名無しさん:02/10/08 20:50 ID:a/htjsrA
物質の温度に上限はあるのでしょうか?

原子振動も光速は超えられないと思うから、
上限の温度は存在すると思うんだけど
どうでしょうか?
415ご冗談でしょう?名無しさん:02/10/08 21:30 ID:???
>>414
速度の上限はエネルギーの上限を意味しない
416ご冗談でしょう?名無しさん:02/10/09 01:55 ID:???
物理は赤、数学は青のイメージだよね
あとは史学が黄色、哲学は灰色
417ご冗談でしょう?名無しさん:02/10/09 02:41 ID:CtWjfMqV
思考の訓練として、問題を解くときにペンを持たない方法はお勧めです。
無駄の無い思考法が得られていく気がするのだがどうだろうか?
418ご冗談でしょう?名無しさん:02/10/09 02:45 ID:???
>>416
だいたいあってるな
俺は物理が焦げ茶、化学は白(白衣からの連想だなきっと)
史学が黄色というのは古文書の紙が黄ばんでいるからだろうか、
国語は緑、漢文は黒、英語はオレンジだな
419ご冗談でしょう?名無しさん:02/10/09 09:10 ID:???
>>414
E:=mc^2/√(1-(v/c)^2)
v→∞ のとき E→∞
「ひもで物体を引っ張ったばあい、物体もひもを引っ張っている。
この2つの引っ張る力は反対向きで大きさが等しいから、
ひもと物体の接点における2つの力は釣り合っており、
ひもで引っ張られた物体は動き出さない。」

この命題のどこが間違っているのでしょうか?
421ご冗談でしょう?名無しさん:02/10/09 09:11 ID:???
>>417
必要な知識がすべて頭にはいってればそれもいいかもな
422ご冗談でしょう?名無しさん:02/10/09 09:14 ID:???
>>420
君の言う2つの力は、「(ひもから)【物体】にかかる力」と「(物体から)【ひも】にかかる力」だ。
物体の運動に寄与するのは前者だけ。よって、物体にかかる力は
他にないのでつりあうことなく加速する。
423ご冗談でしょう?名無しさん:02/10/09 09:16 ID:???
>>419
間違えた。

E:=mc^2/√(1-(v/c)^2)
c→∞ のとき E→∞
424ご冗談でしょう?名無しさん:02/10/09 09:41 ID:JHz7tt2N
12個のパチンコ玉があります。その中に、一つだけ重さの違う玉があります。
でも、その玉は重いか軽いか分かりません。

問題
天秤を使って、その中の重さの違う一つを捜しましょう。
ただし、3回のみで見つけてください。
そして、その玉が重いか軽いか答えてください。

425ご冗談でしょう?名無しさん:02/10/09 10:22 ID:???
一回目:6個6個にわけてはかる
二回目:重いほうを3個3個にわけてはかる
三回目:
 1)二回目で釣り合った場合
  一回目の軽いほうの3個のうち任意に2個選びてんびんにのせる
  つりあったら選ばれなかった1個が軽い玉
  釣り合わなかったら軽いほうが軽い玉
 2)二回目で釣り合わなかった場合
  二回目の重いほうの3個のうち任意に2個選び(以下略
426ご冗談でしょう?名無しさん:02/10/09 10:27 ID:???
>>419>>423
v→cのとき、だろ
427ご冗談でしょう?名無しさん:02/10/09 10:33 ID:???
>>426
>>423
ボケまくってた。すまん
428キャンバースラスト?:02/10/09 10:35 ID:???
質問です。
普段、インラインスケート(タイヤが直線的に4つ並んでいるやつ)を使い
坂道でスキーのまねごとをしているのですが、なぜ曲がるのかよくわかりません。
大きく傾けた方が小さく曲がるのでキャンバースラスト効果なのかな?
とも思うのですが、最初からタイヤが直線的にお互い固定されていても
旋回力?というのは発生するのでしょうか?
429ご冗談でしょう?名無しさん:02/10/09 10:49 ID:a7tdziaJ
>414,5
我々の宇宙で最高のエネルギー状態が
あったとすればすなわちBIGBANN
然しプランク時間以前は???
物理=七色
430ご冗談でしょう?名無しさん:02/10/09 11:18 ID:???

>1)二回目で釣り合った場合
>一回目の軽いほうの3個のうち任意に2個選びてんびんにのせる
上の2行目の「軽い方の3個」がわかりません、6個ではないか?
 
431430:02/10/09 11:20 ID:???
>430は>>425さんへの疑問です。
432ご冗談でしょう?名無しさん:02/10/09 11:28 ID:???
物質をすり抜けてしまうニュートリノが検出できるのは何故ですか?>神
433425:02/10/09 11:51 ID:???
すまんtypoだ

×1)二回目で釣り合った場合
× 一回目の軽いほうの3個のうち任意に2個選びてんびんにのせる

○1)二回目で釣り合った場合
○ 二回目の軽いほうの3個のうち任意に2個選びてんびんにのせる
434425:02/10/09 12:05 ID:???
あ、やっぱりムリだ。
435ご冗談でしょう?名無しさん:02/10/09 12:42 ID:???
>>432
君はゴジラが東京湾に現れるところを見たことがありますか?
背中が青く光っているでしょう
同じ仕組みです
436ご冗談でしょう?名無しさん:02/10/09 12:42 ID:KoF7y13w
>>432
ごく希にすり抜けないから

もう少し説明すると、

電荷がある粒子は、検出器を構成する電子や原子核に少しでも近づくと、電磁力で相互作用する。
(電磁力は逆2乗でしか減衰しないので、なかなかゼロにはならない)
だから、簡単に検出できる。

電荷が無くてもハドロン数を持つ中性子などは、強い力≠ナ相互作用する。
強い力≠ヘ、原子核ていどの距離しか伝わらないが、近い距離では電磁力より非常に強い。
だから、それなりに検出できる。

しかしニュートリノは電荷もハドロン数も持たない。
弱荷はあるので、弱い力≠ナ相互作用する。
強い力≠ヘ、強い力≠フ1/10ていどの距離しか伝わらないうえ、とっても弱い。
だから、めったに検出できないない。

だがもちろん、まったく検出できないわけではない。
そもそも、全く検出できないなら、発見されていない(w
437ご冗談でしょう?名無しさん:02/10/09 12:46 ID:KoF7y13w
>>428
タイヤは完全な円筒?

もし中央より両端が、径が小さければ、
傾いた側が、より小さい径で回転し、距離が短くなるが
438ご冗談でしょう?名無しさん:02/10/09 12:49 ID:KoF7y13w
>>414
>>415の言うとおり
そもそも、『光速を越えられない』とは、
『いくらエネルギーをつぎ込んでも(=温度を上げても)光速を越えられない』
という意味。
439ご冗談でしょう?名無しさん:02/10/09 13:03 ID:???
>>428
地面はタイヤを「まっすぐ」押すので
タイヤを傾ければ 傾いた方向に押す
つまりカーブしたい方向に斜めに押されるってことなんだな

表現がいまいちか
440キャンバースラスト?=428:02/10/09 13:10 ID:???
>>437
バイクや自転車のタイヤと同じと考えてくださってけっこうです。
確かに一輪のタイヤを傾けると内側と外側で接地している部分の回転半径が異なり
曲がろうとしますよね。
でも、それらのタイヤが最初から直線的に複数個、固定されている場合でも
同じように曲がろうとしてくれるのでしょうか?
それぞれのタイヤが曲がろうとする中心が異なるのでお互いにジャマをしそうな
感じがするのですが。
441キャンバースラスト?=428:02/10/09 13:13 ID:???
>>439
いや、なんかシンプルだけど。
ピピーンとヒントになったような気がします。
442ご冗談でしょう?名無しさん:02/10/09 13:33 ID:/qir4Wx3
加速度が位置xの関数として与えられるとき、質点の運動を与えるためには、微分方程式
dx^2/dt^2=a(x)・・・@
をといてxを求めればいいのですが、速度をvとするとき、微分方程式
(1/2)dv^2/dt=a(x)
を@式から求める方法を教えてください。dv、dtなどはそれだけを文字式のように
単体の形で使用してもいいのでしょうか?
443ご冗談でしょう?名無しさん:02/10/09 13:34 ID:/qir4Wx3
442のレスの二つ目の数式の右辺は、a(x)v が正しい形です。
444ご冗談でしょう?名無しさん:02/10/09 14:35 ID:???
>dx^2/dt^2=a(x)・・・@
これも違ってねぇか?
 
d^2x/dt^2 = a(x)
 
だろ
445ご冗談でしょう?名無しさん:02/10/09 14:41 ID:???
で、質問の答えのほうは、

d^2x/dt^2 = dv/dt (v = dx/dt だから).

ここで、vをxの関数と見ると

dv/dt = (dv/dx)(dx/dt) = vdv/dx.(ここがポイント)

つまり vdv/dx = a(x).

よって vdv = a(x)dx.

めでたく変数分離できたね.
446ご冗談でしょう?名無しさん:02/10/09 15:02 ID:???
>>435-436
弱い力で引き寄せられたニュートリノがゴジラの青い光を出す???
何かと衝突するんですか?
447ご冗談でしょう?名無しさん:02/10/09 15:28 ID:???
>>424
パチンコ玉の問題の解答

1.一回目12個をA,B,Cの3群に分けてA,Bの2群を比較する。
傾く場合が最悪だからその場合のみ検討する。
Aが重いように傾いたとする。軽重が不明ながら悪玉はA,Bの
どちらかにある。
2.Aの4個のうち3個をBへ移し、Bの3個を取り去り、
AへCから3個補充してA,Bを4個づつに揃えて
二回目の比較をする。
傾きが変わればAからBへ持っていった3個の中に重い悪玉がある。
傾きが変わらなければ最初からA,Bに残っていた各1個のいずれかだから
軽重は不明でも2個のうち1個を特定するのは簡単。
三回目:3個の中に重いのがあるのだから簡単にわかる



448406:02/10/09 16:20 ID:???
>>410
その定常波の波形が、空気の密度の大小を表しているものと勘違いしてました。
もうわかりました。
教科書だけ頼りに独習しているんで、結構つらいです。
449名無し:02/10/09 17:29 ID:j3We7fJj
動粘性係数のくわしいデータがのっているのってどこ?
450ご冗談でしょう?名無しさん:02/10/09 18:25 ID:???
クリストッフェル記号の添え字のi,jはなんとなくわかるんですが
kって何ですか?どうやって計算すればいいんですか?
451ご冗談でしょう?名無しさん:02/10/09 18:37 ID:aPPsbjON
>>450
気合で・・・
summationの規則を分かっていれば面倒なだけで別に難しくない。
いきなり4次元でなく2次元ぐらいからやってみたら?
452ご冗談でしょう?名無しさん:02/10/09 18:49 ID:???

summationの規則って何ですか?
もしよかったら教えてください
453ご冗談でしょう?名無しさん:02/10/09 18:53 ID:???
>>451
i,j,kって4つの数から異なる3つの数をとるってことですか?
454ご冗談でしょう?名無しさん:02/10/09 18:54 ID:aClOkmDt
>>452
つーかそんなことも知らずにクリストッフェルとか使ってるわけ?
どういう勉強の仕方をしてるんだ…。
455ご冗談でしょう?名無しさん:02/10/09 19:01 ID:???
>>448
あるモデルが物理事象のどんな性質を表現しているものか
注意する必要がある。
がんばってね
456ご冗談でしょう?名無しさん:02/10/09 19:09 ID:???
>>454
勉強不足ですいません
でも、相対性理論の本を何冊もあさって読んでるんですが
summationの規則ってどこにも書いてないんです・・・。
こういう低レベルな質問ってここではだめですか?
457ご冗談でしょう?名無しさん:02/10/09 19:23 ID:fcm/CfNb
左右から光(電子波でも)がやってきて、
同振幅、同位相で完全に干渉するとどうなるの?
この世から完全に消えてなくなるわけ?
458ご冗談でしょう?名無しさん:02/10/09 19:25 ID:???
>>446
>>435はネタだとわかってないのか...

>何かと衝突するんですか?
そうです。ニュートリノが(例えば)電子と衝突します。これは
弱い相互作用なので滅多に起こりません。衝突された電子は
弾き飛ばされ、水中でチェレンコフ光という特殊な光を出すので
それを捕まえます。これが(スーパー)カミオカンデによる
ニュートリノ検出の原理
459456:02/10/09 19:26 ID:???
すいませんが
誰か助言を
460のんちゃん  ◆YJw/wwHt/g :02/10/09 19:29 ID:sUWFttZI
>>456
上と下に同じ文字があったらその文字に0,1,2,3を代入したもの
を全部足し合わせるということです(四次元の場合)。ご存知でしょう?
461456ではないが:02/10/09 19:49 ID:???
summationの規則ってそのことだったのか。
sum rule(和則)と紛らわしいので
Einstein記法と呼んでくれ!
462ご冗談でしょう?名無しさん:02/10/09 20:09 ID:???
>>457
右に走る波はsin(kx), 左に走る波はsin(-kx)
sin(-kx)=-sin(kx)なので,二つの波が合わさるとsin(kx)-sin(kx)=0
となり,波は完全に消えます。
しかし、コサイン波の場合は偶関数なので、合わさると2倍になります。

463456:02/10/09 20:09 ID:???
>>460
ああ、縮約のことですか。それなら知ってます。
でも、クリストッフェル記号で縮約をとるってどこをどうすればいいのか
ぜんぜん理解できないんですけど。

>上と下に同じ文字があったらその文字に0,1,2,3を代入したもの
>を全部足し合わせるということです(四次元の場合)。ご存知でしょう?

クリストッフェル記号で、上と下って何と何のことを言ってるんですか?

464ご冗談でしょう?名無しさん:02/10/09 20:18 ID:???
>>463
かっこつけずに超簡単な本を読めよ。
なっとくする相対論とか、相対論のための数学とか。
猿でもわかるようにかいてあるんじゃないか?
465464:02/10/09 20:20 ID:???
>>463
ところでいろいろ読んだそうだがどんなの読んだの?

ランダウ、内山、佐藤勝彦、文隆、シュッツ、ディラックには書いてあったけど。
466456:02/10/09 20:35 ID:???
>>464
その納得する相対論を読んでて測地線方程式のところでつまってて
どうしてもわかんなかったからここに相談に来たわけです
467ご冗談でしょう?名無しさん:02/10/09 20:35 ID:hQXod3of
まさにちょっとした疑問で申し訳ないのですが、物理のレポートを今TeXで書いています。
そこでちょっとした図を書きたいのですが、みなさんはどのようなツールを使って図を書かれていますか?
ぜひマジレスで教えてください。よろしくお願いします。
468456:02/10/09 20:40 ID:???
>>465
テンソル 石原繁
なっとくする相対性理論 松田卓也
相対性理論 佐藤勝彦
などです

めちゃめちゃ相対性理論に興味があるので
それ系の本を読んでいるんですが
まだ高校生なのでリーマン幾何学がきつくて困っているしだいです。
やっぱあきらめたほうがよいのですか?
469ご冗談でしょう?名無しさん:02/10/09 20:51 ID:???
>>467
Windows+TeXならWinTpicを使うのが簡便かな。
ただ、ゼロ半径の円とか間違って描いてしまうと
セーブしたときにバグって固まって絵を失ってしまうので注意!
470ご冗談でしょう?名無しさん:02/10/09 20:55 ID:???
WinTpic情報、有難うございます。とりあえず調べて、簡単なようなら
早速今回のレポートに使ってみたいと思います
471ご冗談でしょう?名無しさん:02/10/09 21:06 ID:hQXod3of
WinTpic使ってみましたが、これはLaTeXの\pictureを視覚的に編集する環境みたいですね。
色々応用方法がありそうで、とても参考になりました。
また、こういうのではなく、例えば回路図や実験の簡単な説明みたいに、
ある程度の規模の図を書く場合に操作しやすいツールなどがありましたら教えてください。
472ご冗談でしょう?名無しさん:02/10/09 21:19 ID:???
朝昼晩
473ご冗談でしょう?名無しさん:02/10/09 21:25 ID:4a7cmRAP
http://www15.tok2.com/home/toksix/download/no20021009211625.gif

このような問題があるんですが、(見にくくてスイマセン)
この(2)について質問です。
各時刻における発音体の位置は、t1:A t2:E t3:C t4:F t5:A ですよね。
発音体は等速直線運動をしているんだから、時刻(t3-t1)、つまりAからCの円周を移動するのにかかる時間と、
時刻(t5-t3)、つまりCからAの円周を移動するのにかかる時間は同じですよね。
∴(t3-t1)=(t5-t3)
だと思うんです。

なのに、解答では「(t5-t3)より(t3-t1)のほうが4a/Vだけ大きい」なんて
わけわからん事が書いています。なぜですか???
474ご冗談でしょう?名無しさん:02/10/09 21:29 ID:???
>>463
クリストッフェル記号には
上に1つ、下に2つの添え字があるはずですが。
テンソルではありませんが、Einstein記法の規則は
同じように適用されます。
475464:02/10/09 21:30 ID:???
>>468
大学生なら逝ってよしというぐらいだが、、、高校生か。
もしかしたら、将来、優秀な人材に育つかもしれないので、
少し教えてあげよう。

「a_i」はaの右下にiをちっちゃく書いてるとおもって。(下付き)添え字という。
君が学校で習った数列と同じようなものだ。iは区別のためにかいてある(後述)
a^i はiをちっちゃく上に書いてるって意味だ。みかけは、累乗とおなじだけど、
相対論の分野にかぎっては、これは違う。添え字だ。
相対論ではi=0,1,2,3までしかとらない。
これが、数列の場合との違いだ。
つまり、a^iとかいたら、a^0 , a^1 , a^2 , a^3の4つの変数を意味する。
もし、a_iとlかけば、a_0 , a_1 , a_2 , a_3の4つを意味する。これら8個の
変数はすべて別物だ。しかし、関係はある。

その意味を書き出すと話がそれすぎるので、和の規約に。

a_i b^i のように1つの項の中に上下におなじ添え字(この場合はi)
がでたときは、i=0,1,2,3で和をとるという意味になる。

a_i b^i = a_0 b^0 + a_1 b^1 + a_2 b^2 + a_3 b^3

という意味だ。
476464:02/10/09 21:33 ID:???
クリストッフェル記号は、
Γ_{ik}^j だよね。(下にi k がついて上にjがついてるという意味)

たとえば、Γ_{ik}^kとかなると、上下におなじ文字があるので和の規約が
適用されて、

Γ_{i0}^0 + Γ_{i1}^1 + Γ_{i2}^2 + Γ_{i3}^3

という意味になる。

混乱の無いように注意。

和の規約=Einstein記法=summationの規則

全部同じ意味
477456:02/10/09 21:35 ID:???
>>475

縮約はしってるって言ってるだろ!ボケ!
478ご冗談でしょう?名無しさん:02/10/09 21:40 ID:???
縮約って、同じテンソル/行列の中で和を取ることじゃなかったっけ?
479ご冗談でしょう?名無しさん:02/10/09 21:41 ID:???
>>477
縮約の話がどこに出てるのか教えてください
480ご冗談でしょう?名無しさん:02/10/09 21:42 ID:???
縮約を知っていてEinstein記法を知らないというのはどういうことなんだろう、と言ってみるテスト
481ご冗談でしょう?名無しさん:02/10/09 21:43 ID:???
というか、456≠477 でしょ?
456 は今まで sage てないし。
482ご冗談でしょう?名無しさん:02/10/09 21:47 ID:???
>>481
それだ
483ご冗談でしょう?名無しさん:02/10/09 21:48 ID:???
>>477=479
484ご冗談でしょう?名無しさん:02/10/09 21:52 ID:???
>>473
発音体の移動時間じゃなくて
振動数の時間変化について考えてるんだろ
485473:02/10/09 21:52 ID:4a7cmRAP
>>473の質問誰か分りませんか?
というかここで聞くのはお門違いでしたか?
486473:02/10/09 22:08 ID:4a7cmRAP
>>484
なんとなく分ったような・・・
発音体が出した音が観測者Pに届くまでの時間も考慮に入れろ、ということですか?
487484:02/10/09 22:33 ID:???
>>486
全然違う
発音体が観測者に向かっているときと離れていくときでは
「ドップラー効果」によって振動数の時間変化に違いが生じるから

この問題は「ドップラー効果」の問題でしょうが
488484:02/10/09 22:40 ID:???
ごめん
全然違うは言い過ぎだな
489ご冗談でしょう?名無しさん:02/10/09 22:45 ID:???
>>484はイイ人だと判明しますた。
490ご冗談でしょう?名無しさん:02/10/09 22:58 ID:???
>>486
AおよびCから出た音がPに到着する時間は、それぞれa/V、3a/Vと
おけるから、それを考えてやれば[答 (t3-t1)の方が4a/Vだけ長い]が出せるから
その考え方でも、まぁ良いんだけどね.......

ドップラー効果とはなんなのか基本から確認しなおしてみたほうがいいよ。
491826:02/10/09 23:54 ID:vUzZ2JdV
生物板から来ました。
NMRで自己拡散係数を測定できると聞いたのですが、
その原理をわかりやすく教えてください。
492ご冗談でしょう?名無しさん:02/10/10 01:59 ID:L6iG4f43
ラザフォード散乱の古典力学の問題なんですけど
α粒子が電荷Ze,質量数Aの十分重いターゲット原子核によって
散乱されるときの弾性散乱におけるエネルギー保存から、散乱前後
(ともに無限遠方)におけるα粒子運動量ベクトルの変化を求めよ
という問題なんですけどエネルギー保存は
m(\dot{r}^2+r^2\dot{\theta}^2)+k/r=E
ですよね。(クーロンポテンシャルの係数はkにしました。)
これからどうやって運動量ベクトルの変化を出すんですか?
493ご冗談でしょう?名無しさん:02/10/10 02:30 ID:???
日系サイエンスに修正ニュートン力学なんて記事が載ってましたが、
具体的にどうF=maを修正したのか、式がどこにも載っていませんでした。
なんとなく文章からは「F=m(a+a_0)」と修正したいと言ってるような
雰囲気が読み取れないでもなかったんですが、まあ細かい形はともかく
なんでこれが「ある特定の力、またはポテンシャル」の
関数形の変更ではなくてニュートンの運動法則のほうの修正
という話になるのか、さっぱりわかりません。
原理のない現象論的な式だというし。
494ご冗談でしょう?名無しさん:02/10/10 02:35 ID:???
>>493
原論文よめば?
495456:02/10/10 02:38 ID:???
>>464
こんなに丁寧に書いてくれるとは思ってなかったので
大変うれしいです。ありがとうございました。
クリストッフェル記号の縮約がなんとなくわかってきた気がします。
ほんとになんとなく・・・。
でもクリストッフェル記号を計量テンソルであらわしてる式
をみると半端じゃなく計算難しそうですね・・・。
496456:02/10/10 02:39 ID:???
ちなみに477は僕じゃないです

497ご冗談でしょう?名無しさん:02/10/10 02:45 ID:4L/OUZti
>>493
加速度が小さいときは
加速度の二乗に比例するとかそんな感じだったと思う。
小耳に挟んだだけなんで分からんが。

このページを読んでみるといいかも。
俺は面倒なんで読んでないが。
http://www.astro.umd.edu/~ssm/mond/
498ご冗談でしょう?名無しさん:02/10/10 03:09 ID:???
>>497
ありがとう。
a=F/[m*mu(x)],x = a/a0で、
mu(x)を観測に合うような形に勝手に決めてるのか。
なんかこれってダークマターの特定の質量分布による影響を
再現するように決めれば、だいたい重力でつくられる系は
相似な形なるから似たような銀河一般に当てはまるってだけじゃないのか、
なんて思っちゃったりするんだが、どうなんだろうね?
球状星団のなとかかんとかはうまく説明できなくて
そこだけ暗黒物質を仮定するなんていう本末転倒なことをしてたし。
499ご冗談でしょう?名無しさん:02/10/10 09:10 ID:???
>>497
投稿された論文?ちょいと興味があるな。
500464:02/10/10 09:29 ID:???
>>495
もしかして、縮約の意味なんて考えてる?
意味は、考えないほうがいいよ。そういう計算なんだなフーン。
と計算方法を暗記すればいい。


501ご冗談でしょう?名無しさん:02/10/10 11:05 ID:???
なんで誰も>>462につっこんであげないの?
502ご冗談でしょう?名無しさん:02/10/10 11:54 ID:???
>>501
お前が(ry
503ご冗談でしょう?名無しさん:02/10/10 12:03 ID:???
じゃあつっこむか…

>右に走る波はsin(kx), 左に走る波はsin(-kx)

走ってねぇじゃん!
504ご冗談でしょう?名無しさん:02/10/10 13:22 ID:???
462です。
誰も突っ込んでくれなかったのでうれしいです。
>>492
エネルギー保存則からはpの変動出すのはムリ。
真面目に方程式解くか、近似で冪=F冲〜Fb/v (bは最近接距離)
として、刄ニ=冪/p〜2Ze^2/(bE)くらいでガマンするか。

505ご冗談でしょう?名無しさん:02/10/10 14:43 ID:SL9ktZqA
すいません、ちょっと質問させてください。
オーストラリアの映画「ピクニック・アット・ハンギングロック」の中で
こんな台詞があります。

「物事はみな始まり、そして終る。定められた時に定められた場所で。」

この台詞の後、数学教師が本に描かれた図形を見るのですが
この場面がずっと気になっているのです。
その図形が台詞と関係があるように思われるのですが
はたして、上の台詞を表すような数学の定理があるのでしょうか?
506ご冗談でしょう?名無しさん:02/10/10 15:07 ID:???
>>495
測地線の方程式 d^2xi/ds^2+Γijk(dxj/ds)(dxk/ds)て、最小作用の原理 δ∫ds=0から
すんなりと出てくる。
一方、Γijkが適当な座標変換で全成分をゼロにできる事は、Γの座標変換則から出てくる。

て事は、適当な座標系(Xj)を選べば d^2Xj/ds^2=0となって局所的に重力を消せる事が言えるわけで、
これは「等価原理」そのもの。導出できるんだから「等価原理」て「原理」じゃないじゃんと思うのよ。
君の意見を聞かせてくれ。

507ご冗談でしょう?名無しさん:02/10/10 15:39 ID:???
(続き)
電磁場のLagrangianをLorentz Gauge込みで共変形にして場の方程式を求めると、
曲率テンソルが入って来る。これは、電磁場があると等価原理は成立しない
って事ですよね。だから「等価原理」て大げさな言い方おかしいように思う。
あと、Gaugeが変わるとどうなるか、式が面倒になってよく分からん。
508456:02/10/10 16:55 ID:???
>測地線の方程式 d^2xi/ds^2+Γijk(dxj/ds)(dxk/ds)て、最小作用の原理 δ∫ds=0から
>すんなりと出てくる。

知ってます。物理の本のオイラーラグランジュ方程式のところから勉強し始めたので
苦労しました。でも、いまだに共変、反変の概念がわかってないのか、測地線
方程式の中に出てくる添え字の変換がよく理解できなくて困ってます。
とりあえずテンソルの本を読んでじっくり勉強しようと思ってます。
509水理:02/10/10 16:58 ID:GloFij9E
管路の急拡部の形状損失係数はh=KV~2/2gであらわされることが証明できない。
どおやるんだーーーーー!
いっこは、ベルヌーイの式を使うことはわかったんだが・・・。
どなたか、回答を求む。
かなり深刻。

510456:02/10/10 17:19 ID:???
>>506
dsって距離のことを言ってるのかとおもって
一瞬わからなかったんですが
測地線に沿ったパラメータのことなんですね。
って言うか同じ意味ですか?
っていうか測地線に沿ったパラメータってそもそもなんなんでしょうか・・・?

>だから「等価原理」て大げさな言い方おかしいように思う。

そもそも原理って言葉の意味がよくわからないんですが
アインシュタインのころはまだ誰も宇宙に行ったことはなかったんだろうから
無重力空間を具体的にイメージすることって難しかったんだと思います。
そんな時代にエレベーターに乗っただけでそれをイメージできたアインシュタインの
等価原理の発想は天才的といえるのではないでしょうか?
っていうか話がずれてたらごめんなさい。
511ご冗談でしょう?名無しさん:02/10/10 17:33 ID:???
>>510
通常はそっち線の長さをあらわすパラメータを指す。
つまり、ある点を固定してそこから考えている点までの距離を
そっち線に沿って測ったときの長さ。
512ご冗談でしょう?名無しさん:02/10/10 18:27 ID:???
>>507
電磁場の方程式に曲率なんて出てこないよ。
だから等価原理は破れていない。
513のんちゃん  ◆YJw/wwHt/g :02/10/10 19:00 ID:2QMxHrZt
>>506
重力が空間の性質からくる見かけの力だという考えから一般相対論
は作られています。つまり、自由粒子の経路を決定するのが最小作
用の原理δ∫ds=0である、と仮定した時点で既に等価原理の考え方
を用いていることになっていると思います。dsの表式の中に重力の
元となっている(とアインシュタインが考えた)計量テンソルが入っ
ているからです。等価原理を仮定した理論体系から、適当な座標系
を選べば運動方程式がd^2Xj/ds^2=0になるという「等価原理」が導
かれるのは至極当然のことだと思います。
514ご冗談でしょう?名無しさん :02/10/10 21:25 ID:gsk/0Mrz
数値流体力学なんですけど、
MAC法でやってて、最初、すごい値の誤差が出て、
その後だんだん小さくなって行くのですが、
それでいいんでしょうか?
最初から誤差出てたらどこかおかしいのでしょうか?
515ご冗談でしょう?名無しさん:02/10/10 22:24 ID:???
>>514
初期条件。
516ご冗談でしょう?名無しさん:02/10/10 22:34 ID:???
>>514
初期条件。
517=492:02/10/11 01:14 ID:zhqH3CWN
>504
ごめんなさい。問題書きおとしてました。

ラザフォード散乱の古典力学の問題なんですけど
α粒子が電荷Ze,質量数Aの十分重いターゲット原子核によって
散乱されるときの弾性散乱におけるエネルギー保存から、散乱前後
(ともに無限遠方)におけるα粒子運動量ベクトルの変化を求めよ。
ただし一核子の質量はmとして、α粒子は速度vで入射してきたものとし
散乱角は\thetaとする。

という条件付でした。
518水理:02/10/11 01:15 ID:lzNPoAXb
運動量方程式がたてられないー
だれか、まじ 頼みます
519ご冗談でしょう?名無しさん:02/10/11 01:38 ID:UTyHm2Vd
J.J.sakurai の一文がどうしても訳せないです。お願いします

It is not a priori obvious that nature takes advantage of
the lowest dimensional realization of (3.1.20), but numerous
experiments-from atomic spectroscopy to nuclear magnetic
resonance-suffice to convince us that this is in fact the case

原書でP159の1行目にあります。
3章で3.2 Spin 1/2 Systems and Finite Rotation
(スピン1/2系と有限回転)のとこです
520456:02/10/11 01:42 ID:???
>>511
解説ありがとうございます
521ご冗談でしょう?名無しさん:02/10/11 02:05 ID:???
>>519
自然界が、(3.1.20) を実現するこの最小次元を
なぜ利用しているのかは自明ではないが、
原子分光から核磁気共鳴に至る
数え切れないほどの実験から、
実際にそうであることが十分確かめられる。

訳書の P213 より。
522ご冗談でしょう?名無しさん:02/10/11 02:07 ID:???
>>519
問題なのは"this is the case"だろうか?↓な感じで銅だろう。

多くの実験結果から、nature〜(3.1.20)が実際に正しいと言うことが出来る。
523522:02/10/11 02:09 ID:???
あぅぅ…。
524521:02/10/11 02:13 ID:???
>>522
手抜きしてすまそ (;´Д`)
丁度手元にあったので。
525ご冗談でしょう?名無しさん:02/10/11 02:18 ID:UTyHm2Vd

 >>521
ありがとうございます。
あと
 同じページの(3.2.3)でJでなくなぜSかがわりません。
526521:02/10/11 02:28 ID:???
>>525
3.1.20 を満たす例として S を考えて、
それによる回転を考えようとしているからです。
527ご冗談でしょう?名無しさん:02/10/11 02:30 ID:UTyHm2Vd
>>522
ありがとうございます
528ご冗談でしょう?名無しさん:02/10/11 04:06 ID:UTyHm2Vd

>>525
ありがとうございます
529ご冗談でしょう?名無しさん:02/10/11 07:53 ID:6bBjrVxG
質問です。
物理学科の学生が4年間掛けて学ぶ内容を独学で理解することは難しい話でしょうか?
また、そうする場合、カリキュラムの一端として物理の授業が組まれている環境の方がよいのでしょうか?
530ご冗談でしょう?名無しさん:02/10/11 09:11 ID:???
おまえら真面目にやれよ!!!
アニヲタの自覚あんのかよ!!
ラムが基本なんだよ!すべてはここから始まるんだよ!!
原作を完読しれっ!!TVシリーズを10日で全部見ろっ!!
亜空間アルバイトの回やつるつるセッケンの回をキャプしろっ!!!
口付け宅急便の扉絵を涎と精液でドロドロにしろっ!!!
話 は そ れ か ら な ん だ よ !!!
他のキャラに走るのはラムが卒業できてからなんだよ!!!
たとえ卒業できなくたって構やしねぇんだよっ!!!
原点なんだよ!!!伝説なんだよ!!!神話なんだよ!!!
ラム語れねぇヤシが偉そうな口たたくんじゃねぇよっ!!!
卒業できねぇヤシが『本気ラム』なんだよっ!!
もう一回言うぞっ!!!
おまいら真面目にやれよっっっ!!!!
531ご冗談でしょう?名無しさん:02/10/11 09:25 ID:???
>>513
δ∫ds=0のdsは、固有時間dτの方が意味は分かりやすいですね。この式は、
「重力場中を自由落下する質点は、その固有時間が最小になるように軌道(測地線)が
決まる」てことで、何となく等価原理より多くの事を述べているように思ったん
ですが、考えてみると、上の主張は等価原理と等価ですね。
おっしゃる通りです。ははー。

>>512
一般相対論では電磁場の方程式は □Ai+Rij Aj+ji=0で共変微分の非可換項を
通して曲率テンソルが出てきちゃう。
だから電磁場中では等価原理は崩れてますが。
532ご冗談でしょう?名無しさん:02/10/11 09:50 ID:???
あんたら素人向けの説明へたくそだね
533ご冗談でしょう?名無しさん:02/10/11 11:13 ID:???
じゃあお前が以下略
534ご冗談でしょう?名無しさん:02/10/11 13:06 ID:???
>>529
数学を一緒に勉強しないと物理は分かりません。
最低、フーリエ変換、複素関数論程度まではやらないと。
問題はあなたが数学に向いているかどうかです。
535ご冗談でしょう?名無しさん:02/10/11 13:31 ID:???
>>531
ダランベルシアンの形に変形するのがトリックですね。
□A_i に含まれる曲率と R_ij A_j がキャンセルするのですよ。
だから電磁場の運動方程式に曲率は出てこない。
計量で書いてみれば一発で分かりますよ。

そもそも最小結合しかしないんだから等価原理は破れないのです。
536ご冗談でしょう?名無しさん:02/10/11 15:53 ID:+rckJnHB
長年の疑問があります
デカいの「デカ」とはSIのデカでしょうか。

さらにどなたか
ダルいの「ダル」が英語のダルなのかどうかもご存知でしたら教えてください。
537ご冗談でしょう?名無しさん:02/10/11 16:15 ID:???
>>535
俺アホでよくわかんないからあからさまに計算してくれたら嬉しい…
538535:02/10/11 16:52 ID:???
>>537 めんどくさいなあ。
J_i = g^jk ∇_j F_ki , F_ki = ∇_k A_i - ∇_i A_k
だから
J_i = g^jk ∇_j ∇_k A_i - g^jk [ ∇_j , ∇_i ] A_k - g^jk ∇_i ∇_j A_k
右辺第1項がダランベルシアンで、第3項がダイバージェンス。
んで第2項がリッチテンソルってわけだね。

これが共変な形での式。つづく。
539535:02/10/11 17:29 ID:???
つづき。だけど F って
F_ki = ∂_k A_i - ∂_i A_k
のようにただの微分なので( F の反対称性を使うので添字を上げるね)
J^i = ∂_j F^ji + Γ^k_jm F^mi = (√-g)^(-1) ∂_j ( (√-g) F^ji)
これでもう計量の2階微分が含まれていないことがわかるので、
リッチテンソルは出てこない。

実際、局所慣性系では
Γ = 0 ⇔ ∂g = 0
なので、局所慣性系のもとで上の方程式は
J^i = g^jk g^im ∂_j ( ∂_k A_m - ∂_m A_k )
になる。というわけで等価原理は破れていない、と。
540537:02/10/11 17:52 ID:???
なるほど。ありがとうございます。
541ご冗談でしょう?名無しさん:02/10/11 17:59 ID:FGP/L4CZ
A―w―――w―−−w−−−w−ー
    |   |   | 
    w   w   w
    |   |   |
B―w―――w―−−w−−−wーー
無限に続いている
w:抵抗R 
AB間の合成抵抗rを求めよ。という問題なのですが
2R<r<3Rということくらいしか分かりません
並列の回路としてみてよいのでしょか?
無限に抵抗があるのでΣを使い計算するのでしょうか?
すいません、分かる方教えていただけませんか?
542ご冗談でしょう?名無しさん:02/10/11 18:42 ID:1rUz6gu7
エネルギー保存と運動量保存って、
どうして違うの?
543ご冗談でしょう?名無しさん:02/10/11 18:59 ID:PQJgn6r+
DQNな質問ですみません。

核崩壊の式 dN/dt = −λN から平均寿命を求めたいのですが、
寿命と言うものをどのように表せば良いかが分かりません。
544ご冗談でしょう?名無しさん :02/10/11 19:12 ID:Z0uRMbGT
金属でフェーズダイアグラムってありますよね。
それでアルファフェーズ、ガンマフェーズとありますが
意味がさっぱりわかりません。
簡単にでいいですので誰か教えてください。
もしくは、その関係が書かれてあるお勧めの本があれば
教えてください。
545ご冗談でしょう?名無しさん:02/10/11 19:23 ID:???
>>541
(1+√3)R
546ご冗談でしょう?名無しさん:02/10/11 21:46 ID:JmBViPex
標準状態で、40リットルの酸素は、740mmHg、273℃で何リットルとなるか?

を、教えてもらいたいのですが。

547ご冗談でしょう?名無しさん:02/10/11 22:02 ID:???
>>546
教科書に載ってます
548ご冗談でしょう?名無しさん:02/10/11 22:17 ID:MP1029On
E=(m^2c^4+p^2c^2)^1/2
この式ってどうやって出てくるんですか?
549546:02/10/11 22:20 ID:???
もういいです…。
550ご冗談でしょう?名無しさん:02/10/11 22:23 ID:???
>>549
いや、マジで教科書読めばすぐわかると思うが・・・
つーか>>1読んでから質問しろ
551しゅわ:02/10/11 22:46 ID:mIUNYVMm
>>544
相図のことですね。熱力学の教科書を見ればわかる。
金属が混ざる時、その相(Phase)は組成と温度に依存する。
それを図示したのが相図(Phase diagram)です。
結晶成長の本を見てもよい。
552ご冗談でしょう?名無しさん:02/10/11 23:06 ID:???
>>542
運動量とエネルギーの違いはまず分かっていますか。
553ご冗談でしょう?名無しさん:02/10/11 23:07 ID:???
>>543
 半減期でいいんじゃない?
554ご冗談でしょう?名無しさん:02/10/11 23:58 ID:???
>>548
エネルギーと質量の等価性とローレンツ不変性からかな?
555543:02/10/12 00:05 ID:ZALSYBGW
>>553 レスしていただきありがとうございます。
実は、自力で解いてしまいました。

∫[0,∞]λNte^(-λt)dt /N = 1/λ
556: FKHba-29p149.ppp13.odn.ad.jprlo:02/10/12 00:17 ID:???
真・スレッドストッパー。。。( ̄ー ̄)ニヤリッ
557ご冗談でしょう?名無しさん:02/10/12 00:39 ID:???
>>555
元の式を変形して

 dt = -1/λN dN = -1/λ dlogN

両辺を積分して

 ∫[0,τ] dt = ∫[log(N0),log(N0/e)] -1/λ dlogN
 τ = -1/λ log[(N0/e)/N0] = 1/λ
558ご冗談でしょう?名無しさん:02/10/12 00:39 ID:shRFxG06
?
559ご冗談でしょう?名無しさん:02/10/12 01:17 ID:FCqxiqlM
>>541の者ですが、
 ――w―――w―――B
 |   |   |
 w   w   w 
 |   |   |
 ――w―――w――
 |   |   |
 w   w   w
 |   |   |
 |―w―――w――
 |
 A
同じく、w:抵抗=R
AB間の合成抵抗はいくつになるんですか?
これはキルヒホッフ→オームの法則という解き方しかありませんか?
お願いします、上の問題も考えていませが分かりません。。。
560ご冗談でしょう?名無しさん:02/10/12 02:02 ID:N35w5/v8
理学部物理と工学部応用物理は一体何がどう違うのですか?
どちらが本物の物理なんでしょうか???
561ご冗談でしょう?名無しさん:02/10/12 02:07 ID:???
偽モノの物理ってなんだよ。




トンデモか?
562ご冗談でしょう?名無しさん:02/10/12 02:15 ID:???
工学部応用物理が本物。これ常識。
563ご冗談でしょう?名無しさん:02/10/12 05:44 ID:At9QiVIF
>>559
対称性を使いましょう。
真ん中の所で切り離して考える。
/\
◇ ◇
\/
切り離しても対称性より電位差は生じないので、このような回路の合成
抵抗を求めれば良い。
564563:02/10/12 05:45 ID:At9QiVIF
ずれた。
 /\
◇  ◇
 \/
こんな感じ。
565ご冗談でしょう?名無しさん:02/10/12 11:57 ID:K0HdZegF
空気抵抗がないとき、同一の加速度で角度αとβで投げ上げて、
それらの和α+β=Π/2であれば着地点がどうして同じになるのでしょうか?
566ご冗談でしょう?名無しさん:02/10/12 12:19 ID:???
閑話休題、請聞我的話。先日我去了近在吉野家、吉野家。
座席満席、我不可能座。我凝視垂幕 「一五〇円値引」
我思、嗚呼馬鹿、嗚呼阿呆。
御前等、無意味来通常不来吉野屋為壱百五拾円引、阿呆。
壱百五拾円、只壱百五拾円。親子連来店。一家四人来吉野屋。目出鯛。
「好!父、注文特盛!」的発言、我不可視。我謹呈壱百五拾円、為御前等開放席。
吉野屋本来要殺伐雰囲気。機会多々有喧嘩上等共奴反対座食卓。
雰囲気最高、我刺御前、御前刺我。女子供不要干渉。
長時間後、我可能着座、隣男注文「大盛、汁沢山」我再度激切了。
聴、「汁沢山」是不流行現在、阿呆。我呆、御前注文得意顔「汁沢山」
我欲問「御前本当不本当欲食『汁沢山』?」我欲問詰。小一時間我欲問詰。
我欲問「御前只欲発言『汁沢山』」我是吉野屋通。最新流行間吉野家通是「葱沢山」
大盛葱沢山玉。是注文方法為吉野家通。葱沢山是、多々葱少々肉。是。
要注意是注文即店員警戒危険、是諸刃的剣。不可推奨素人。
御前等超素人、推奨食牛鮭定食。
567ご冗談でしょう?名無しさん:02/10/12 13:28 ID:???
>>565
 なるほど、確かにそうなるな。あ、加速度じゃなくて速度だよな?

 一応計算を。初速度をv、仰角をαとすると、到達距離は
 v^2/2*sin(2α)
 この値と等しくなる別の仰角をβとすれば、
  α+β=π/2
 となる。
 
>>566
 なんかすごく分かりやすいんだが、これで合ってるの?
ちゃんと通じるのかね?
568ご冗談でしょう?名無しさん:02/10/12 13:31 ID:???
>>566
激ワラ。さりげに「目出鯛」とか書いてあるし。
569567:02/10/12 13:31 ID:???
 すまん、式を間違えた。到達距離は、
v^2/g*sin(2α)
だな。(g:重力加速度)
570ご冗談でしょう?名無しさん:02/10/12 13:34 ID:???
つーか思いっきりガイシュツのコピペ
571565:02/10/12 14:01 ID:???
>569 ありがとうございます。
 教えてもらった式を参考に自分でも解いてみます。
572シケメソ ◆AirH/XmY.w :02/10/12 16:11 ID:HHmhZbvc
大学で物理を勉強してて、面白いと思い始めたのはいつごろから?
573名無しさん:02/10/12 16:29 ID:???
うあぎゃsdkkさkkらおえprぎじょえいrrtdhbrdtdrzhgbzfdt
あえrswgヴぃろせjg@えごえいおjhgfxghんsfgchんfgc
http://genie.gaiax.com/home/nakatai
574ご冗談でしょう?名無しさん :02/10/12 17:11 ID:B5ErY5Pr
>>551
ありがとうございます。
読んでみましたが、そもそも相を
どういう風にイメージしたらよいのか
わかりません。
みなさんはどう考えていますか?
575age:02/10/13 00:58 ID:???
age
576ご冗談でしょう?名無しさん:02/10/13 03:44 ID:tczys8c6
>572
小学校とかで、理科として勉強してるときから好きだ、ってのはだめですか?
577ご冗談でしょう?名無しさん:02/10/13 03:54 ID:65piTx6/
>>564
すいません、どこできってるのか分かりません。
◇は抵抗ですか?
578ご冗談でしょう?名無しさん:02/10/13 03:56 ID:???
>>572
特殊相対論、量子力学、解析力学のあたり。
579ご冗談でしょう?名無しさん :02/10/13 07:04 ID:F6AGe5Nc
>>574
551ではないが、単純にいえば結晶構造が違うね。
アルファー、ガンマと逝ってる所を見ると、例えば鉄
をイメージしてるのかな。室温で安定な鉄の安定相
はアルファー相でb.c.c.(体心立方)構造。高温で
安定な相はガンマー相でf.c.c(面心立方)構造。
この場合は、910℃を境に、原子が動いて結晶構造が変わる。
フェーズダイアグラムは「状態図」と称する事もある。
金属やセラミックスの本、あるいは固体物理の本を読めば
載っておる。ところで材料ネタなのに何故に材料板で訊かぬ。
580579:02/10/13 07:36 ID:mn+bmlJM
>>574
しつこいようだけど、ちょっと補足。
上のは温度に対する変化だけど、>>551氏が言っている
ように組成に対しても、相が異なる。

例えば銅はf.c.c.構造(アルファー相)だけど、亜鉛を
加えていくと、35wt%亜鉛くらいまではアルファー相の
ままだけど、それを超えるとb.c.c.構造のベータ相が
まじった2相構造になり、50wt%亜鉛ではベータ相のみ
になる。知ってると思うけど、銅と亜鉛の合金を真鍮と呼ぶ。
581ご冗談でしょう?名無しさん:02/10/13 10:00 ID:???
>>572
解析力学。理論の美しさを実感しました。

582ご冗談でしょう?名無しさん:02/10/13 13:34 ID:???
>>572
わたしも解析力学。
クールでスリムないい女て感じだった。

量子力学はシュレ方程式を特殊関数使って解いてた時はヤナ奴と思ってたが
調和振動子のエネルギーや角運動量固有値が演算子の交換関係使って出る事知って
見直した。本妻なので今は感動も何もなし。

特殊相対論は、遊び慣れてる女みたいでこちらも楽しませてもらったけど、
一般相対論まで行くとこちらの体力もたないって感じ。
統計力学は(以下略)

583ご冗談でしょう?名無しさん:02/10/13 16:49 ID:???
雨って空気抵抗があるからあったっても死なないんですよね?
じゃあ、雲の上からバチンコ玉を降らすと弾丸よりも早くなって
あたったら詩にますか?
584ご冗談でしょう?名無しさん:02/10/13 17:18 ID:???
人の生死は落下物の終端速度だけで決まるものでもないと思う
585ご冗談でしょう?名無しさん:02/10/13 17:22 ID:???
>>583
確実に氏にます
586574:02/10/14 00:39 ID:24xbynBN
>>579
ありがとうございます。
アルファフェーズがb.c.c.(体心立方)構造
ガンマフェーズがf.c.c(面心立方)構造
というわけですか。
あとの説明を見ると
ベータフェーズがb.c.c.(体心立方)構造
アルファとベータの違いは何ですか?
しかし、温度によって安定な構造に変化する、って面白いですね。

アロイの相は温度と組成に依存するってわけですね。
確かフェーズダイアグラムもそのような書き方になってました。
587狗王苦:02/10/14 01:18 ID:eeOet5rp
π+のクォーク波動関数が分かりません。
L=S=0でスピン部分は↑↓-↓↑ってのはいいんですが、残りのフレーバーとカラーの波動関数が分かりません。
二つの粒子の入れ替えに対して反対称にしなきゃいけないんですよね。
588579:02/10/14 04:15 ID:IkU4NVwq
>>586
違う、違う。
純鉄の場合や銅-亜鉛合金の場合がたまたまそうなってる
だけで、相の名前と結晶構造が対応するわけではない。

普通は低温で安定な相から順番にアルファー、ベータ、
ガンマ・・・(鉄は例外)、組成に関してはメインの元素
(銅-亜鉛なら銅)に近い方から順にアルファー、ベータ、
ガンマ・・・となる。

結晶構造が違うというのは実は相の正しい定義ではないが、
君にとってはこの説明の方が理解しやすいと思う。因みに
合金(アロイ)に限らず純物質でもそうなのだが、温度、
組成のみならず圧力・外磁場...etc.によっても安定な
構造が変化する。普通の「状態図」は君の言う通り、横軸
が組成・縦軸が温度になっている。材料板の方が専門だから
これ以上詳しくは向こうで。
589ご冗談でしょう?名無しさん:02/10/14 06:25 ID:???
>>577
AとBの真ん中で切れてる。>>564の図の下と上がA,Bにつながっているのだと思う。
一本の線が一つの抵抗になっていて、◇は4つの抵抗のブリッジ。
590ご冗談でしょう?名無しさん:02/10/14 15:01 ID:BcQLZrjx
まじくだらねー質問ですみません。
重力波ってまだ観測されたことないんですよね?
591ご冗談でしょう?名無しさん:02/10/14 18:08 ID:qVxfb2LP
>>589
そうですよね!そう思って計算したんですけど答えが合わないんですよ
まず{(1/2R)+(1/2R)}^−1=(1/R)
でR+(1/R)+R={(2R^2+1)/R}
でこれをrとすると{(1/r)+(1/r)}^−1
となりますよね、こたえが1,5Rなんですがうまく消えません
どこか途中の計算間違っていますか?お願いします。
592591:02/10/14 18:19 ID:qVxfb2LP
訂正:こたえが1,5Rなんですがうまく消えません
          ↓
   答えには1,5Rとなっているのですが合いません
593ご冗談でしょう?名無しさん:02/10/14 19:35 ID:???
無意味に全角使うな
「,」と「.」を分けわからんところで使うな
594ご冗談でしょう?名無しさん:02/10/14 20:31 ID:???
うるせーバカ
てめー死ねよ
595ご冗談でしょう?名無しさん:02/10/14 20:59 ID:???
複素関数の積分はどういうイメ−ジで考えたら
いいのでしょうか?実数関数の積分のように面積
みたいな分かり易いモノは無いでしょうか?
596ご冗談でしょう?名無しさん:02/10/14 21:03 ID:???
>>595
閉経路であれば、ブランチカットなり穴を潜るかどうかのイメージかと。
597ご冗談でしょう?名無しさん:02/10/14 21:04 ID:+bcE6Juw
足し算や割り算の有効数字ってのはどうすればいいんでしょうか?
掛け算の場合は、何も考えず精度のひくい物に合わせるんですよね。
598ご冗談でしょう?名無しさん:02/10/14 21:11 ID:???
>>597
誤差付の数字の加減乗除をやってみよう。
自ずから分かると思う。

599ご冗談でしょう?名無しさん:02/10/14 22:28 ID:???
0.5kWHの電力量を熱量に換算すると何kcalになりますか?
600ご冗談でしょう?名無しさん:02/10/14 22:36 ID:???
>>599
1Jは何kcalですか?
601111!!:02/10/14 22:36 ID:???

602599:02/10/14 22:38 ID:???
1kcal=4.2Jです!
603ご冗談でしょう?名無しさん:02/10/14 22:43 ID:???
>>599>>602
1kWHは何J?
604597:02/10/14 22:48 ID:+bcE6Juw
>>598
3,333なら、3,333yにして、誤差yはその桁における±4
ってふうにして計算するってことですよね。
それで結果にてyの含まれる桁を切り捨てれば良いんですか?
割り算の筆算はどうやれば?
605ご冗談でしょう?名無しさん:02/10/14 22:50 ID:???
>>604
割算も素直にやれば?
数字を入れてやれば、だんだんわかってくると思う。

まじにやるなら、誤差伝播を勉強しる。
606ご冗談でしょう?名無しさん:02/10/14 23:33 ID:???
なんで小数点に,を使うわけ?なんで?素朴に。
607597=604:02/10/15 01:11 ID:IiTxk9TM
たとえば、24.y÷3.33xなんかは、
まず商の一の位が7で、
7*3.33x=22.21(7x)を24.yから引いて、
1(y-3)5(10-7x)と余りが出る。
次に、商のコンマ以下一桁に0、
余りは1(y-3)5(10-7x)0。
この後、商のコンマ以下二桁目はどうするんですか?
yが何であれ、コンマ以下二桁目は4以下kだから
切り捨てて、商7でいいんですか?

マジにやるというか、明日ちょっと要るんでどうか。
608ご冗談でしょう?名無しさん:02/10/15 01:28 ID:???
>>607
24.y÷3.33x は
F(x,y) = (24.0 + 0.1*y) / (3.330 + 0.001*x)
で、xとyがプラマイ5くらいをふらつく時にF(x,y)がどれくらい
ふらつくかを考えればよろし。
特に、x,y(の係数)が小さいと思ってテーラー展開するのも手。
609ご冗談でしょう?名無しさん:02/10/15 01:43 ID:???
>>606
ヨーロッパだと , を使った気が。
610607:02/10/15 01:50 ID:???
>>608
ええと、それで変わらない部分を残して、
それより下を切り捨てればいいんですか?
掛け算のやり方とはかなり違うんですね。
4から6の辺りや8,9の辺りをふらついても、捨てますか?
多変数のテーラー展開は知らないです。
611ご冗談でしょう?名無しさん:02/10/15 01:56 ID:???
>>609
なるほど、イギリス以外のヨーロッパ圏はそうらしいね。
しかしどこでそういう習慣を身につけるんだろう。
現在の日本でそういう表記に触れることなんて
ほとんどなさそうなものなのに。帰国子女?
でもなけりゃ、ただの誤表記としか
612つばめさん ◆dpLvO.T.cY :02/10/15 03:41 ID:J9eK6Rgu
>>610
一般的なやり方(>>607よりも頭をかえって使わない)
F(x,y)=F(x0,y0)+(∂F(x,y)/∂x)_0*Δx+(∂F(x,y)/∂y)_0*Δy+1/2<ΔX、D(∇F)ΔX>+・・
だから、
F(x,y)=x/y=x0/y0+1/y0*Δx−x0/y0^2*Δy
x0=24、y0=3.33、Δx=0.1、Δy=0.001
613ご冗談でしょう?名無しさん:02/10/15 03:46 ID:I+cE7ODx
>>591
◇の抵抗はRになります。
※これを1/Rとしているのが間違い。
なので、
 /

 \
の抵抗はR+R+R=3R。
これの並列回路の抵抗rは
1/r=1/3R+1/3Rよりr=1.5R。
614ご冗談でしょう?名無しさん:02/10/15 03:47 ID:???
ラグランジアンの物理的な意味がよくわかりません。
実際物体の運動においてラグランジアンが大きいとどうなんですか?
ハミルトンの原理では物体のとおりにくさをあらわしているように
思うのですがハミルトンはなぜそういう風に考えたのでしょうか?
誰かわかりやすく教えてください。
615ご冗談でしょう?名無しさん:02/10/15 04:56 ID:???
相対性理論の測地線方程式を求める過程で
メトリックテンソルがたびたびg_μνからg_ανとかg_μβとかに
変換したりされたりしててよくわからないんですが
これってどういうことですか?
(読んでる参考書は「なっとくする相対性理論」です)
616ご冗談でしょう?名無しさん:02/10/15 11:26 ID:???
>>615
添え字変えただけじゃないの?
617ご冗談でしょう?名無しさん:02/10/15 11:29 ID:OJM3BR8A
>>613
ありがとうございます、基本的な間違いをしてました・・・
これと同じような回路で
 □□□ ̄
 □□□
_□□□
というのも同じようにとこうとしたのですが、これは無理でしょうか? 
618ななし:02/10/15 11:56 ID:wcdxeVjy
定常波とかなでる、線密度ρってなんですか?
619ご冗談でしょう?名無しさん:02/10/15 14:54 ID:???
先生、相転移現象ってどういうのですか
620ご冗談でしょう?名無しさん:02/10/15 15:56 ID:???
たとえば、水が氷になったり水蒸気になったりすること。
…って、中学生か?
621ご冗談でしょう?名無しさん:02/10/15 16:04 ID:???
>614
>実際物体の運動においてラグランジアンが大きいとどうなんですか?

ラグラジアンの値自体にはあまり意味はないです。
622ご冗談でしょう?名無しさん:02/10/15 18:05 ID:6fDaQCQH
まったくもって低レベルな質問ですが
相対性理論的には回転ってどう考慮されてるんですか?
回転って絶対的な中心と遠心力を受けるその他ができますよね

623ご冗談でしょう?名無しさん:02/10/15 19:26 ID:???
>>622
あなたは相対性理論を知っていますか?
624御冗談デセウ?名無シサン:02/10/15 20:14 ID:???
>>618
線状の物体の単位長さあたりの重さ。
625HAL ◆2eOaL2U.Gg :02/10/15 21:08 ID:???
>>614
たしかにエネルギーや運動量等のように物理的意味ワカランね。
形式的なものとおもってだまって覚えるのがよかろう
626622:02/10/15 22:26 ID:JbMI2F0s
いや、ほんと詳しくは知らんないんです
稚拙な知識では
「異なる観測者から互いを観測した場合、それは自己を中心とした相対的な結果しかでない」
「つまり世界に絶対的な座標は作れない」ということでしょうか
しかし回転する物体の場合そこに中心という絶対座標が生まれますよね
周囲の観測が一切不能なボックスに閉じ込められ回転した場合、たしかに中心点以外からは中心点が回転しているように見えますが(相対的)
遠心力が働く以上遠心力が低くなる方向へ向かえば絶対的中心にたどり着きます
まあ、相対性理論自体勘違いしている人間の考えですが
これでもしわずかでも宇宙が回転していたら宇宙に絶対番地が振れるかと
627ご冗談でしょう?名無しさん:02/10/15 22:40 ID:???
>>626
物理を哲学と混同されているように感じます

絶対的な座標を設定したければ
今すぐあなたご自身を中心として座標を設定してしまえばよい
628ご冗談でしょう?名無しさん:02/10/15 22:52 ID:???
電荷の問題で、長さ2aの絶縁物の棒の両端に+Q、−Qの点電荷があって、
棒の垂直二等分線上に点電荷qがあると棒に働くトルクはどうなりますか?
629ご冗談でしょう?名無しさん:02/10/15 22:58 ID:???
>>628
何が分からないの?
630ご冗談でしょう?名無しさん:02/10/15 23:00 ID:???
>>627
いやぁ、「絶対的な」座標なんて存在しないでしょう。

自分を絶対的座標の中心とする考えのほうが哲学っぽい気がしますが…。
631ご冗談でしょう?名無しさん:02/10/15 23:00 ID:VgrKtmtt
プリズム分光器でナトリウムランプのスペクトルを
観測したのですが、波長 735nmくらいの、
文献に載っている波長とは明らかに異なる
線スペクトルが見えてしまいました。
ほかの(部屋などの)光が入ったのではないとすると、
原因として何が考えられますか?
632628:02/10/15 23:02 ID:???
>>629
棒の場合どう考えるのわからないです。
あとトルクって何ですか?
633ご冗談でしょう?名無しさん:02/10/15 23:07 ID:???
>>632
どっちにしろqまでの距離が分からないとどうしようもない
634632:02/10/15 23:09 ID:???
>>633
すみません、qまでの距離はbです
635ご冗談でしょう?名無しさん:02/10/15 23:11 ID:???
じゃあ点電荷から受ける力が分かるね、それでその力のうち
棒を回転させようとする成分がどれだけあるか分かるね。
トルクはその成分と回転軸からの距離をかけたもん。
636ご冗談でしょう?名無しさん:02/10/15 23:15 ID:???
>>634
トルクって用語をまず調べてみ
それからじゃないか?

でも「てこの原理」知らんの?
637ご冗談でしょう?名無しさん:02/10/15 23:18 ID:???
>>630
「物理では」どのような動機で座標を設定するとお考えですか?
638634:02/10/15 23:23 ID:???
んー。。
パニック。。。
もう少しわかりやすく・・?
639ご冗談でしょう?名無しさん:02/10/15 23:25 ID:???
トルクが何か調べたかい?
640634:02/10/15 23:28 ID:???
>>639
力×長さですか?
641ご冗談でしょう?名無しさん:02/10/15 23:31 ID:???
>>640
で てこの原理 って 知ってる?
642640:02/10/15 23:33 ID:???
>>641
は、あまりわからないです。。
643ご冗談でしょう?名無しさん:02/10/15 23:35 ID:???
惜しいな。棒で考えると「力の方向」と「棒の軸から作用点を結ぶ直線」
の方向は直角じゃないとだめ。直角じゃないなら力のうち、直角方向の
成分だけ取り出して考えなさいってこった
644ご冗談でしょう?名無しさん:02/10/15 23:36 ID:???
>>631
735nmの赤外LEDってのはあるけど、たぶん無関係。
645ご冗談でしょう?名無しさん:02/10/15 23:38 ID:???
>>637
 物体の(ある時刻における)位置を表記するためだろ?
 計算しやすい座標系というのはもちろんあるが、特に
「こう定義しなくちゃだめ!」なんていうやり方がある
わけじゃない。どんな座標系だろうと、つじつまがあって
ればいいんだよ。
646ご冗談でしょう?名無しさん:02/10/15 23:43 ID:???
>>642
ドライバーの柄やドアノブのとっては
どうして太くなっているか知ってる?

バットの細いほうと太いほうを握って
お互いに反対方向に回したとき
どちらが有利かわかる?
647ご冗談でしょう?名無しさん:02/10/15 23:47 ID:???
>>645
ん?
622に突っ込んでたつもりなんだけど

まさか あなた622じゃないよね
648ご冗談でしょう?名無しさん:02/10/16 00:24 ID:???
>>647
 おや? これは失敬。レスの流れを勘違いしていたようだ。
もちろん622じゃないよーん。
649 :02/10/16 00:26 ID:7cDQYfQF
アントニオ猪木の発明した、
水から(?)エネルギーを取り出す
ものすげー装置ってなんだったか・・
とっても気になるのでおしえてちょーだいー
650 :02/10/16 00:35 ID:7cDQYfQF
http://www.so-net.ne.jp/srs-dx/news_series/kiji02_0314_inoki.html
これだった。
自己レススマソちなみに水ではない。
651ご冗談でしょう?名無しさん:02/10/16 00:39 ID:???
>>648
いや 手間が省けて良かった
652ご冗談でしょう?名無しさん:02/10/16 00:44 ID:LO4upO9l
>>617
まず半分にする。ずれてたら、適当に判断して下さい。
    /
   /\
  /\/
  \/\
   \/
    \
で、この中の
   /\
  /\/
  \/\
   \/
を、
   / \
  /\ /
  \/ \
   \ /
の並列と考えて抵抗を求める。
なぜこうできるかは>>563に同じ。
653ご冗談でしょう?名無しさん:02/10/16 06:13 ID:???
>>616

だから、どうして添え字を変える必要があるのかきいてんだすよ
教えてくんで申し訳ない
654のんちゃん ◆YJw/wwHt/g :02/10/16 06:35 ID:???
どういった式変形で添え字が変わっていたのか具体的に示してくれな
いと答えようがありませんよ。
でも一応推測でお答えしておきますと、おそらくsummationをとる添
え字を変えているだけだと思います。最終的には残らない添え字です
から何に変えてもかまいませんよね。
655ご冗談でしょう?名無しさん:02/10/16 06:44 ID:???
>>653
あなたg_ανg^μβとg_μνg^μνは意味が違うことはわかってますか?
わかっているなら前者を表したいのに後者のように書いてはまずいってことは
理解できますね。

このように同じ添え字を使うと具合が悪いときは仕方なく別の文字を使うわけ。
656ご冗談でしょう?名無しさん:02/10/16 06:56 ID:???
>>654
summationって
添え字が3つあったら
4次元では何個の数の和をとることになるんですか?
657ご冗談でしょう?名無しさん:02/10/16 06:59 ID:???
>>654
それとなぜ最終的には残らないのでしょうか?

>>655
なぜか得る必要があるんですか?
g_μνg^μνのままだと何かまずいことでもあるんですか?
658のんちゃん ◆YJw/wwHt/g :02/10/16 07:07 ID:???
>>656
そんな状況はありえません。
同じ添え字が3つとか、教科書にも絶対載ってないでしょう?
>>657
Σ_{μ=0}^{μ=3}Γ_{να}^{μ}A_{μ}
という式の添え字はνとαしかありませんよね。アインシュタインの記約
ではΣを抜かしているので分かりにくいのかもしれませんが、上と下に
同じ添え字がある場合、その添え字はあくまで「ダミー」であって、
最終的には残らないのです。
659ご冗談でしょう?名無しさん:02/10/16 07:21 ID:???
>>657
あなたアインシュタインの和の約束を知らないのですね。
その状態で参考書などを見てもわかるはずありません。
和の約束のオンパレードですから。
ましてや測地線方程式をやろうなんて100万年早すぎます。

同じ添字が現れたら、特に断らない限り和を取ります。
μとかνなどのギリシャ文字なら0〜3(本によっては1〜4)、
iとかjなどのローマ文字(でいいのかな?)なら1〜3で和を取ります。
これが和の約束。

この約束を頭に入れた上で、他の人のレスや参考書を読み直すこと。
660ご冗談でしょう?名無しさん:02/10/16 07:29 ID:???
追加。
和を取らないとしても、g_ανg^μβとg_μνg^μνは違うってわかりませんか?
g_01g^23とかg_31g^03とか任意の添字の組合せを扱いたいときに
後者のように書かれてたら困るでしょ。

661657:02/10/16 07:52 ID:???
じゃあ
Γ_{να}^{μ}は
Γ_{11}^{1}+Γ_{22}^{2}+Γ_{33}^{3}+Γ_{44}^{4}
ってことですか?
662657:02/10/16 08:22 ID:???
あぁぜんぜんわからないです
添え字の意味すらもわからなくなってきました
663ご冗談でしょう?名無しさん:02/10/16 08:35 ID:???
>>661
それは同じ添え字がないので和はとらなくてもいいのです。
664657:02/10/16 08:56 ID:???
>>663
ああ、なるほど!
やっとアインシュタインの規約の意味がわかりました。
自分思いきり勘違いしてました
ありがとうございました。
665657:02/10/16 09:22 ID:???
ここでもうひとつ質問なんですが
測地線方程式を解く過程で
∂L/∂x^μ = 1/(2L)・∂gαβ/∂x^μ・u^α・u^β
って式が出てくるんですが
微分した後のLの中の添え字がμ、νとはまったく別物になってるんですが
どうしてですか?
∂x^μの添え字μとは関係なくなるんですか?


ただし
(L=ds=√(gμν(x)・u^μ・u^ν))
(u^μ=dx^μ/dλ)

です
666HAL ◆2eOaL2U.Gg :02/10/16 09:41 ID:???
>>665
上下にでている添え字は、和をとるのはご存じですよね。
ダミーの添え字ともいわれて結局和をとるので意味がないのです。

_aが下付き添え字、^aを上付きとすると

U_a V^a=U_0 V^0 + U_1 V^1 + U_2 V^2 + U_3 V^3

です。和をとると数字になるので、添え字はなんでもかまわないのです。

U_b V^b=U_0 V^0 + U_1 V^1 + U_2 V^2 + U_3 V^3
U_c V^c=U_0 V^0 + U_1 V^1 + U_2 V^2 + U_3 V^3




「和をとる」添え字は、なんでもいいのです。
好きな添え字にかえてもいいわけです
667657:02/10/16 10:16 ID:???
>>666
それはわかるんですが
Lのなかのuの添え字μが、同じ添え字をもつx^μで、
Lが微分されることによりαに変わることがよく理解できないです
x^μはなぜダミーの添え字に変形しないのですか?

6682チャンネルで超有名:02/10/16 10:32 ID:o7fQ8xWt
http://mona.2ch.net/546/qwertyuiop.html

http://jumper.jp/yyyu/ 携帯用

中高生とHな出会い
  即アポ即H出来る
  超最高なH&Hが・・ 
669ご冗談でしょう?名無しさん:02/10/16 13:40 ID:???
>>667
問題を簡単にしてAνAνをAνで微分したらどうなるか、これが自力で分からないと
話しになりません。
これでつまづくようなら相対論どころか、数式使って物理やるのは辞めた方が
いいと思います。


670狗王苦:02/10/16 18:07 ID:lIYexRDF
フェルミオンとその反粒子の対、たとえばe^-とe^+を入れ換えるとフェルミ統計により符合が変わるらしいですけど、
そうするとメソンの波動関数はクォークと反クォークの入れ換えに対して反対称化しないといけないと思うんですけど。
バリオンのときは3つのクォークで同種粒子だからという理由で完全反対称化しているのにメソンではしなくていいんでしょうか。

超困ってます。教えてください。マジで。
671のんちゃん ◆YJw/wwHt/g :02/10/16 18:47 ID:???
>>670
>メソンの波動関数はクォークと反クォークの入れ換えに対して反対称化し
ないといけないと思うんですけど

この言い方だと理解されているかどうかよく分からないですが、一応言って
おきますと、反対称化しなければならないのは「粒子1と粒子2の入れ替え
について」です。クォークとか反クォークという「状態」は波動関数ですの
で、それを入れ替えるというのはナンセンスです。
672ご冗談でしょう?名無しさん:02/10/16 19:30 ID:V66coA1z
ケプラーの第三法則においての、
周期の二乗と長軸半径の三乗の関係式の導き方を
教えては頂けないでしょうか?
673HAL ◆2eOaL2U.Gg :02/10/16 19:46 ID:???
>>672
ぜんぜん導出ではないですが、公式を忘れたときの暗記法に・・・
暗記事項は、短半径ではなく長半径に依存すること。

運動に関係しそうな量は、
太陽質量M、地球の質量m、長半径a、そして、
万有重力定数Gです。

一般に重力だけしかない運動では自分自身の質量は、運動に関係しない。
よって、mは運動に関係せず残るM,a,Gで周期Tが表されるはずです。
ここで、これらで時間の次元をつくると、(GMa^-3)(-1/2)ですので、

(Gと質量密度ρで時間の次元をつくると、(3π/(32Gρ))^(1/2)です。次元解析で
すぐに分かります。係数は自由落下時間にあわせました)

T = C(GMa^-3)(-1/2)

T^2 = C a^3

なんてのが、暗記法としてどうでしょうか?
円運動の周期だったら議論に曖昧はないですが、楕円だと
どうしても、aかbのどちらを用いるか次元解析だけではきまらない・・・。

完璧な導出は、

・運動方程式を極座標で書く
・ラグランジアンを書いてELeqをたてる

なんてところでしょうか。むしろこちらをお望みで?
674工房:02/10/16 22:52 ID:/+EmHj/z

a|----------|d
| |
| |
e|----------|f
| |
| |
| |
b|----------|c

難系の例題65です。

起電力Eの電池に上の回路が接続されていて(abと並列)、

ab,bc,cd,adが全て抵抗Rで、

e,fがそれぞれab,cdの中点。

efの抵抗をrとすると、

「キルヒホッフの法則により、

ef間の電流がi=2E/2R+3r,ebcf間の電流がI=rE/R(2R+3r)となる」

とだけ書いてあるんですが、どうやって求めるのかが判りません。

efとebcfが並列なので、電流が抵抗の逆比になる、

つまりi:I=2R:rなので、合成抵抗がE/2R+3rと逆算出来るのですが、

この合成抵抗はどうやって求めたら良いのでしょうか。お願いします。
675工房:02/10/16 22:54 ID:/+EmHj/z
失敗しました、dfとfcは繋がってます。


676ご冗談でしょう?名無しさん:02/10/16 23:01 ID:???
http://science.2ch.net/test/read.cgi/sci/1021694226/208-209n

こういう人がいるんですが……どうしたらいいんでしょう。
“連立微分方程式の解法を知らないからお前は分からないだけだ”と言われて困っております。
私が勉強すればいいんでしょうか?
677HAL ◆2eOaL2U.Gg :02/10/16 23:30 ID:???
>>674
助けてあげたいが、図がよくわからむ。
スキャソしてアップできない?

678ご冗談でしょう?名無しさん:02/10/16 23:34 ID:VP5fIpy5
完全に球体のマジックミラーボール(内側が鏡)を作って
中は真空で外から光を照らすとどうなりますか?
俺の脳内妄想だと永久に反射しつづけて永久に光る物体の出来あがりなんですが。
679HAL ◆2eOaL2U.Gg :02/10/16 23:43 ID:???
>>678
理想的なマジックミラーを考えた場合、
外から光を照らしても中に光が届かないんじゃないかな。
中からは、一部だけ光がでてくるだろうけど。
680ご冗談でしょう?名無しさん:02/10/16 23:47 ID:VP5fIpy5
>>679
おれは、馬鹿だったようだ。
確かに外に光が漏れないと何も見れない罠。
追加でお願いします、マジックミラーではなく普通の鏡の球に
光を閉じ込めると内側ではどう言う状況に。
681工房:02/10/16 23:51 ID:/+EmHj/z
>>677

スキャナないんです・・。すみません。。

図がうまく出ませんね・・。

普通の並列回路なのですが。

「目」の形の。

|----||(電池)----|
| |
a|--------------|d
| |
| |
e|--------------|f
| |
| |
| |
b --------------|c

682工房:02/10/17 00:09 ID:???
何度もすみません、。。

板を汚してしまってますね。。

|----|i(電池)----|
|.................................|
a|--------------|d
|................................|
|................................|
e|--------------|f
|................................|
|................................|
|................................|
b --------------|c

電池が起電力左向きでE。

ab=bc=cd=daで皆抵抗R。

e,fはab,cdのそれぞれ中点です。

ef間を抵抗rとしたとき、「キルヒホッフの法則により、

ef間の電流がi=2E/2R+3r,ebcf間の電流がI=rE/R(2R+3r)となる」

とだけ書いてあるんですが、どうやって求めるのかが判りません。

efとebcfが並列なので、電流が抵抗の逆比になる、

つまりi:I=2R:rなので、合成抵抗がE/2R+3rと逆算出来るのですが、

この合成抵抗はどうやって求めたら良いのでしょうか。お願いします。
683ご冗談でしょう?名無しさん:02/10/17 00:12 ID:???
 |----||(電池)----|
 |           |
.a|--------------|d
 |           |
 |           |
.e|--------------|f
 |           |
 |           |
 |           |
.b|--------------|c
こんなんでどう?
684狗王苦:02/10/17 00:14 ID:XMbD3lBB
>>671
例えば陽子の波動関数だと、「粒子1,2の入れ換えに対称な波動関数uud(2↑↑↓-(↑↓+↓↑)↑)」
とその巡回置換の和をとっています。この説明を読む限り「粒子1,2の入れ換え」というのは
はじめの二つのフレーバー(uとu)、はじめの二つのスピン(↑と↑)を取り替えるという
ことだと思います。で、実際1,2の入れ換えでこの波動関数は不変だけれども、別の入れ換えでは
不変になっていない、だから巡回置換の和にして任意の入れ換えで不変になるようにしなきゃならん、
なぜなら三つの粒子は同種粒子のフェルミオンだから。・・・ってことですよね。

そう考えてしまうと、今度はπ^+の波動関数を考えたとき、スピンは0だから↑↓-↓↑で、全体として
反対称にならないといけないから
((u\bar{d}(r\bar{r}+g\bar{g}+b\{b}))+(粒子1,2を入れ換えたもの))(↑↓-↓↑)
なんじゃないかと思ってしまうんですよ。違うんだろうけど。
何か重大な勘違いをしていることは自分でも分かっています。でもどうしてもダメなんです。
誰か僕を止めてください!
685工房:02/10/17 00:26 ID:???
>>683

あっすごい!有り難うございます!!

友人が文系ばかりで誰にも訊けないんですよ・。
686ご冗談でしょう?名無しさん:02/10/17 00:33 ID:???
>>684
パイオン(メソン)はボソンなんで
全体で対称になるんじゃないの?
687ご冗談でしょう?名無しさん:02/10/17 00:38 ID:phGP7vii
すいめせん 誰か科学のいいサイトおしえてくだちゃい
688ご冗談でしょう?名無しさん:02/10/17 01:00 ID:LI9bfnAd
超音波洗浄機(メガネを洗浄するのと同じ)に入れる水に、台所用洗剤をいれると超音波の勢い?が強まるのか、水の揺れが大きくなります
これは何に由来するのでしょうか?
媒質の粘性などが関係するのでしょうか?

よろしくお願いします
689ご冗談でしょう?名無しさん:02/10/17 01:10 ID:???
>>688
表面張力が小さくなるからでは?
690ご冗談でしょう?名無しさん:02/10/17 01:21 ID:EeSdDgGs
>>688
共振です。
691ご冗談でしょう?名無しさん:02/10/17 01:24 ID:Knv1AEsA
>>682
漏れも工房なので、工房の問題について協力しまふ。

ae=eb=df=fcから、この4つの区間の抵抗はR/2。
キルヒホッフの法則より
回路一周分の電圧降下・上昇の和は0に等しいから、
電池を流れる電流をI、ab,ef,bcの電流をそれぞれi[1],i[2],i[3]とすると、
I=i[1]+i[2]+i[3]:電流の保存
i[1]R=E:電池とabの閉回路でキルヒホッフ
2*(I-i[1])*(1/2)*R+i[2]r-i[1]R=0:abとefの閉回路でキルヒホッフ
2*i[3]*(1/2)*R+i[3]R-i[2]r=0:efとbcの閉回路でキルヒホッフ
4式で3文字消去してIについて解くと、IはRとrとEを用いて表せる。
I=4E(r+R)/R(3r+2R)
さて、abcdで囲む部分を一つの抵抗R[ALL]と見なすと、E=IR[ALL]の式で
表せる。Iを代入するとEを約分できるので、R[ALL]について解くと、
R[ALL]はR,rを用いて、R[ALL]=R(3r+2R)/4(r+R)となる。

つまり、この回路の合成抵抗はR(3r+2R)/4(R+r)となる。
692ご冗談でしょう?名無しさん:02/10/17 01:54 ID:cLIEbIdY
調光回路を用いて点弧角に対する出力などを電球とホーロー
抵抗の2つの負荷に対して求めたのですが
ホーロー抵抗は抵抗値が一定で 電球は抵抗値が減少しました。
この理由がわからないので誰かわかる方お願いします。
693ご冗談でしょう?名無しさん:02/10/17 02:36 ID:???
物質の結晶軸ってなんですか? ただの結晶とは違うのですか?
それと、<100>とか<110>とか<111>とかってどういった意味なのですか?
ご存知の方、教えてください。
694ご冗談でしょう?名無しさん:02/10/17 04:31 ID:yp5zRLEy
静止衛星の打ち上げ時に
Venは静止衛星の円軌道の接線方向に対する速度、
Viは静止衛星を地表から打ち上げる時の初速度、
Rgは静止衛星の円軌道の半径、
Mは地球の質量、Gは地球の重力、Reは地球の半径の時
Ven=EQR{Vi^2-2GM(1/Re-1/Rg)}
を力学的エネルギー保存則を用いて導きたいのですが、ご存知の方がいらっしゃったら教えてください。
695ご冗談でしょう?名無しさん :02/10/17 05:29 ID:EfprJCdo
何故惑星は回転するんでしょうか?
696657:02/10/17 06:34 ID:???
>>669
つまり
∂AμAμ/∂Aμ=∂A0A0/∂Aμ+∂A1A1/∂Aμ+∂A2A2/∂Aμ+∂A3A3/∂Aμ
ってことですか?だとしたらなんとなく納得です。

で、
たびたび質問で申し訳ありませんが
Vi(x)をΔxずらしたときの変化量がなぜ
δVi=Γ^k_ij(x)Vk(x)Δxj
このような式になるのかよくわかりません。
どういうことですか?Vkってなんですか?
697ご冗談でしょう?名無しさん:02/10/17 07:09 ID:0DFj/GHo
>692
点弧角って何か知らん。
因みに電球は明るく点灯してる時にはフィラメントが高熱になっているので冷えてる消灯時に比べ著しく抵抗値が高くなる。電流が変化すると抵抗も変化する。それと関係あるんかね?
抵抗器は普通なるべく変化しないように作ってある。

>693
わけわからん。
結晶と結晶でないのの違いは原子がきちんと並んでるかどうかだろ。並んでるなら向きがあるだろ。
どっちにどう並んでいるかの基準を軸とか言うんじゃないの?数字は角度?
698ご冗談でしょう?名無しさん:02/10/17 07:31 ID:???
>694
EQRってのはSQRの間違い?
運動エネルギーと重力ポテンシャルエネルギーの和は、地表にいるときと静止軌道にいるときで等しい
というところから導いてください。
699工房:02/10/17 07:49 ID:???
>>691

レス遅れてごめんなさい。

そしてありがとうございます!

其のとおりやってみたら解けました!

閉回路ごとにキルヒホッフを使うんですね・・。そうか・・。

あとi[1]はadを流れる電流ですよね・・。多分。

本当に有り難うございました。。今日も解き進めていこうと思います。
700ご冗談でしょう?名無しさん :02/10/17 09:56 ID:WNbkc+qi
>>693
結晶ならば原子(又はイオン)の並びを考えると、どこか
に直線が存在するよね。その直線(軸)が結晶軸。
<132>ってのは結晶内での方向を示し方。結晶格子は分っ
てるよね。原点からa軸方向に+1,b軸方向に+3,c軸方向
に+2進んだ時の方向が[132]。立方晶(立方体)の場合
a軸,b軸.c軸は区別が付かないよね。だから、[100]方向
と[010],[001],[-100],[0-10],[00-1]方向ってのは
区別が付かないよね。これらの方向を等価な方向というが
これら(この場合は6つ)をまとめたのが<100>なる表記。
固体物理、あるいは金属・セラミックスの教科書を見るべし。

>>692
金属と絶縁体(半導体)の電気抵抗の温度依存性の違いだと思う。
これで答えのようなもんだが、判らなければあとはご自分で。

>>697
ここはあんたの無知さ加減を披露する場じゃないんだが・・・。
701ご冗談でしょう?名無しさん:02/10/17 10:21 ID:???
>>696
>∂AμAμ/∂Aμ=∂A0A0/∂Aμ+∂A1A1/∂Aμ+∂A2A2/∂Aμ+∂A3A3/∂Aμ
左辺は∂AνAν/∂Aμと書かねばならないってこと以外は正しい。
∂AμAμ/∂Aμではどの2つのμについて和を取るかで意味が
変わってしまう。

L=AμAμと与えられていて∂L/∂Aμを計算しなさいと言われたとき、
∂AμAμ/∂Aμなんて書いちゃダメなわけ。Lの中のμは和を取るための
ダミー変数なので別の文字に置き換えてL=AνAνやL=AαAαとしても構わない。
で、∂L/∂Aμを計算しようとすると、Lの中のμとバッティングして
しまうので、それを避けるために積極的にLの中の文字を変えないといけない。
>>665
>微分した後のLの中の添え字がμ、νとはまったく別物になってるんですが
>どうしてですか?
の理由も同じ。

>たびたび質問で申し訳ありませんが
あのですね。測地線とかやる前に、曲がった空間での幾何学の基礎をきちんと
身につけてからでないと何をやっても効率悪すぎです。
702ご冗談でしょう?名無しさん:02/10/17 11:22 ID:KC47YMOR
マジな質問です。
>ウィーンにいた私は、北朝鮮の核開発はきわめて初期の段階で、核保有など
>「夢の夢」であることを知っていました。プルトニウムをいくら再処理、生
>産しても核弾頭にはなりません。核実験が不可欠です。核実験をせずに核保
>有国になった国は、世界にありません。私はそういうことを解説し、主張し
>てきたのです。今では、米国も「核弾頭1個か2個分のプルトニウムを貯
>蔵、隠匿している可能性がある」と推測しているにすぎません。

これは極東版にあるある大学教授の発言についてのスレの一部です。
プルトニウムというのは自分で核分裂するので、原爆をつくるのに核実験が必要とは間違っても言えないと思うのですが、物理板なら詳しいかたもいるかと思います。
本当に原爆をつくるのに核実験は必要なのでしょうか。
703ご冗談でしょう?名無しさん:02/10/17 11:34 ID:???
必要無いよ、今はね。
なのにバンバンやってる。目で見ないと理解できない人のために。
704ご冗談でしょう?名無しさん:02/10/17 12:26 ID:???
>>702
北朝鮮に限っては、未臨界は無理だと思う。
プレステを軍事に転用せざるをえないレベルの科学力だし
705狗王苦:02/10/17 12:27 ID:UNywZJlA
>>686
いや、だからクォークと反クォークの入れ換えです。
706ご冗談でしょう?名無しさん:02/10/17 12:31 ID:9o5cSiZm
数学板にこういう寝ぼけたこと言ってるボケナスがいますが,
どう思いますか?

文化としての数学の危機
http://science.2ch.net/test/read.cgi/math/1034669683/
707ご冗談でしょう?名無しさん:02/10/17 13:13 ID:???
小柴さんは、どこからカミオカンデの予算を取ってきたの?
うん億円、、、キモン
708ご冗談でしょう?名無しさん:02/10/17 16:25 ID:???
>原爆をつくるのに核実験が必要とは間違っても言えないと

まあ、実験しなくても作れるとしても、それはシミュレーション技術の向上に
よるものであって、

>プルトニウムというのは自分で核分裂するので

と言う理由からではなかろう。
709ご冗談でしょう?名無しさん:02/10/17 17:10 ID:???
>>702
きわめて初期の段階ということだと、プルトニウムが自発核分裂するので
実験が必要というのはあながち嘘ではない。これが自発核分裂しないウラン
爆弾だと実験なしでもOK。実際、マンハッタン計画のときのトリニティ実験で
試されたのはプルトニウム爆弾のほうで、ウラン爆弾はぶっつけ本番で
広島に投下されました。

ウラン爆弾は構造が簡単なガンタイプの起爆装置でOKなのに対し、
プルトニウム爆弾の起爆装置は複雑な爆縮タイプで、起爆装置の
動作チェックに実験が必要であった。で、プルトニウム爆弾が
ガンタイプにできないのは、まさにプルトニウムが自発核分裂するから。

もちろん、>>708のようにシミュレーション技術の向上で、爆縮タイプの
起爆装置も実験しなくても開発可能になってきているかもしれないけど、
北朝鮮にそこまでの技術があるかどうか
710よっちゃん:02/10/17 18:31 ID:???
エネルギー準位3sへ励起される。とか、エネルギー準位と2pの準位間
で起こり・・・とかで使われるsやpはいったいなんの事ですか?
711ご冗談でしょう?名無しさん:02/10/17 18:34 ID:???
>>700
ヒントとして十分な697の方が教育的。
712のんちゃん ◆YJw/wwHt/g :02/10/17 19:06 ID:???
>>684
できましたら、なぜそれで間違っていると思うのかを書いてもらえますでしょうか。
713狗王苦:02/10/17 19:30 ID:xW+fWdSa
>>712
自分が調べた中ではバリオンについて詳しく説明して対称化している本でも、
メソンについてはそうでないものが全てだったからです。
714ご冗談でしょう?名無しさん:02/10/17 19:55 ID:6YRv1NCD
勉強不足で申し訳ないのだけど、

時間(T)は流れているが、空間(X,Y,Z)方向は流れていないのか?
数学的には同じような扱いしてるのに。

この疑問、どの変の教科書(論文)読めばいい?
715ご冗談でしょう?名無しさん:02/10/17 19:59 ID:???
>>709
アラモゴルドで最初の核実験に使われたのは
プルトニウム×インプロージョン型という説と
ウラニウム×インプロージョン型という説の両方があるね。
(江畑謙介『兵器と戦略』朝日選書,1994)

ウラニウム×ガン型
ウラニウム×インプロージョン型
プルトニウム×インプロージョン型
の3パターンが考えられるわけか。

今は事実上全部、プルトニウム×インプロージョン型だけど。
716ひよこ:02/10/17 20:14 ID:Z1WPRIgg
>>710
あんまり詳しくはないのだが、
3s のように書いた時には主量子数 n = 3 で角運動量量子数 l = 0。
2p と書いた時には n = 2 で、l = 1 を表すようです。
エネルギー準位はこれらの数の組み合わせできまるので、
(もちろん磁気量子数 m も関与するでしょうが…)
同じ主量子数であっても角運動量量子数の違いで
異なるエネルギ準位を取ります。
K 殻電子は n = 1 なので s 軌道しかとれずに 1s のみで、
スピンが↑↓の2種類あるから電子は2個しか入らない。
L 殻は n = 2 なので s と p の両方が取れる。
p に対しては m = 0, ±1 の 3 種類が存在するので
n = 2 に対しては合計 4 つの組み合わせがある。
それぞれに対してスピン↑↓なので、電子は 8 個入る。
ってな具合のようですね。
                  (7 年前の記憶より)
717のんちゃん ◆YJw/wwHt/g :02/10/17 20:17 ID:???
>>713
なるほど・・・
バリオンの波動函数は(フレーバー)×(スピン)の部分では任意のクォー
ク入れ替えに対して対称化してあってカラーも合わせると反対称化され
ることがちゃんと説明してあるのに、π^+の波動函数はそんな説明がな
されていないということですね?手元に「クォークとレプトン」があっ
たので見てみたのですが、そうなっていました。でも反対称になって
いなければならないのは間違いないですから、置換については記述を
省略しているだけなのではないでしょうか?3つクォークがある場合
と違って、2つしかクォークがない場合は対称化の操作って簡単です
よね?だからいちいち書くことをしなかったのかも・・・こんな答で申
し訳ないです。
718ご冗談でしょう?名無しさん:02/10/17 20:49 ID:oX3tyeTN
>716
sとかpとかの意味じゃないのかぁ
詳しくは忘れたけどsharp principal diffuse fundamentalだっけ?
719ひよこ:02/10/17 21:09 ID:Z1WPRIgg
>>718
え?そなの?
いっしょけんめー思い出して損したよ…。
720よっちゃん:02/10/17 21:30 ID:???
>>716
>>718
ありがとうございました。
あと、読む時はs(エス)、p(ピー)って読むのが普通なんですか?
721ご冗談でしょう?名無しさん:02/10/17 22:49 ID:Mk/VcUmt
トムソン散乱は周波数が変わらないのはなぜなのですか?
722ご冗談でしょう?名無しさん:02/10/18 08:51 ID:???
>720 そそ
723HAL ◆2eOaL2U.Gg :02/10/18 09:30 ID:???
>>721
外場の振動数ωに、害鳩しか作用しない粒子置いたときの
解の振動の振動数がωと変わらないのが本質かな

724物理初心者:02/10/18 11:36 ID:9x/Rhtnk
質問させてください

ある試験の問題文
「人体表面電位から心機能を推定したり、過電流から物体中の欠陥の大きさを推定す方法を逆解析手法という」
で、この一文が○か×かなんですが、どちらですか?
また、逆解析手法はどういう手法で、どういったところで使われているんですか?

よろしくお願いします。
725ご冗談でしょう?名無しさん:02/10/18 12:36 ID:NANFRVOU
>>695
回転を止めるものが無いから。

地球上で物体の回転が止まるのは、地面などとの摩擦力によるもの。
726ご冗談でしょう?名無しさん:02/10/18 12:48 ID:NANFRVOU
>>714
流れている。
時間の流れに対応するのは、(時間経過を伴わない)位置の移動。

もちろん、時間の流れはいくつかの点で特殊。

まず、「時間だけの移動は可能なのに対し、空間だけの移動は不可能」。
そのことは数式にある。
cΔt^2-Δx^2≧0

次に、「時間には向きがある」。
これは簡単には数式から導けない。
教科書などではあまり踏み込まないので、一般の書籍を読むことを勧める。
727狗王苦:02/10/18 14:28 ID:Gqu1Xx7Y
>>717
あ、僕が読んでいるのも「クォークとレプトン」です。
バリオンの波動関数を全体で反対称化した舌の根も乾かぬうちにπ+の
波動関数を作っておきながら反対称化しないでこれでどうだ、みたいな書き方(俺にはそう見える)
をするもんですから変だと思ったんです。

でもq\bar{q}系も反対称化しなければならないというのが正しいことが分かりました。
ありがとうございました。
728ご冗談でしょう?名無しさん:02/10/18 15:50 ID:NIwsjtI+
核磁気共鳴の実験で、プロトンのNMRを測定するのですが、
プロトンの場合なぜ静磁場をかけなければいけないのでしょうか?
3d原子のあるなしが関係しているようですが、いまいちよくわかりません。
どなたか教えて下さい。
お願いします。
729質問おやぢ:02/10/18 15:54 ID:NOWiVy/U
振動計測機にて数値が(m/(s2))にて表示されます。
振動速度でしょうが、一般的にわかり易い
振動値として他の単位に換算できないでしょうか?
730ご冗談でしょう?名無しさん:02/10/18 16:19 ID:???
>>729
(最大)加速度が一般的に最も分かりやすいと思います。

まあ、振動が正弦的で振動数が分かればそこから、
速度や振幅は計算できますが...
731ご冗談でしょう?名無しさん:02/10/18 17:35 ID:???
さあ、みんなもどんどん投票してね!

2ちゃんねらが選ぶ最強の売国奴は誰だコンテスト
http://dempa.2ch.net/seroon/soc/x3d8d714c5a9ab.html
732質問おやぢ:02/10/18 17:47 ID:NOWiVy/U
>>730 レス早速ありがとうございます。

データロガーにて数値は、ばあーっと羅列されます。
グラフにすれば見た目はわかり易いんです。
ただ、普通のおっさん達が理解できる縦軸の数値が、ちと。
1バイブとかの単位、ないですわな。(笑
733質問おやぢ:02/10/18 17:51 ID:NOWiVy/U
>>730 加速度で検索してみました。
ttp://www.tacmina.co.jp/convert/acceleration.html
こんな換算ページを発見しました。
再度、ありがとうでございます。
734ご冗談でしょう?名無しさん:02/10/18 19:06 ID:um/HWCLN
ttp://www.hamq.jp/i.cfm?i=DEHIO
このサイトどうよ?
735ご冗談でしょう?名無しさん:02/10/19 00:48 ID:???

I=∫[-1,1](1-z^2)^(-0.5)dzを留数を使って計算しろという問題なんですが
−1と1の切断の周りをクルリと回って2iI=2πiΣResになると思うのだけど
肝心の留数が分かりません。どうやって計算するか教えてください。
736紅乙女:02/10/19 14:50 ID:nwSy1Irg
ブラックホールに関して質問させてください。
BH に落ち込む物質を外から観測すると、シュバルツシルト半径に達したという
情報を得るには無限の時間がかかりますよね。
では、BH が星を飲み込んで太る描写を考えたとき、その星はいつまでも
BH に付随するように見えるのでしょうか?
737ご冗談でしょう?名無しさん:02/10/19 15:02 ID:???
>>736
原理的にはそう。でも実際は急速に赤方偏移して見えなくなる。
738紅乙女:02/10/19 15:12 ID:nwSy1Irg
>>737
すばやいレス、さんきゅ。
落ちていく星にとってみれば、有限の時間でBHの中心に達することが
できますよね。つまりは有限の時間でBHの質量は増え、シュバルツ半径も大きくなる。
じゃあ、「シュバルツ半径に達したよ!」という情報は、頑張って重力場を逃れようとするけれど、BHが太ることによって飲み込まれてしまう
のでしょうか?
736 に自分で書いた「付随するように見える」というのは、
飲み込まれた途端に消えて、やはり太ったBHだけが残るのでしょうか…?
739ご冗談でしょう?名無しさん:02/10/19 17:11 ID:???
>>738
だから、飲み込まれる瞬間を見るには無限の時間が必要なんだってば。

実際は赤方偏移で波長がBH程度に長くなった段階で、何を見てるやら
わからなくなる。波長より細かい物は見えないので。
さらに、それ以前に測定器の検出限界よりエネルギーが小さくなって
事実上見えなくなる。



740質問おやぢ:02/10/19 17:26 ID:PuH6DzIp
>>734 見ました。
長らく理解不可能でひた。
ようっく見たら違う意味でサンクスです。お気に追加です(笑
741主婦のサークルです。:02/10/19 19:17 ID:wWXV6mtk
ほんの少し刺激を求めてる主婦のサークルを作りました!
私たち主婦が楽しめる安全でちょっとトキメキのある出会を求めて
只今男性会員を募集中です!! 女性会員も募集(^○^)/"
742よっちゃん:02/10/19 19:31 ID:???
遷移っていったい何の事ですか?
あと、光共振器、光検出器、照射の読み方がわからない
ので教えて下さい。
743ご冗談でしょう?名無しさん:02/10/19 21:19 ID:IBcgLafO
なんでLCAO近似を考えたのか,その思考回路が分かんねぇ〜。
744ご冗談でしょう?名無しさん:02/10/19 22:25 ID:???
>>743
原子が集まって固体ができるからじゃないの?
745ボルダ:02/10/19 23:24 ID:NePl0wy7
ボルダの振り子の慣性モーメントの式が
I=m(r+l)~2+2mr~2/5 で与えられるわけを教えてください。

l:ナイフエッジから球の上端までの距離
r:球の半径
m:振り子の球の質量
746ご冗談でしょう?名無しさん:02/10/19 23:43 ID:???
東京横浜で物理の参考書沢山売っている場所おしえてください
あと、もし理系専門の古本やがあったら教えてください
747ご冗談でしょう?名無しさん:02/10/19 23:49 ID:???
神保町の明倫館。品揃えはいいが高い。
748つばめさん ◆dpLvO.T.cY :02/10/19 23:53 ID:fAT0AIzk
なんで、掃除のアルバイトで、
周りの3流の私立大学とか専門学校の人は言われずに、
東大卒のわたしだけ分数もコンピューターも使えないだろうと
嫌味を言われたんでしょうか?
749つばめさん ◆dpLvO.T.cY :02/10/19 23:55 ID:fAT0AIzk
いわゆる医者の息子で、医者になりたくもないのに理数系の素質がまったくないのに、
5浪くらいして医学部に行ったような状況と思われたんでしょうか?
750ご冗談でしょう?名無しさん:02/10/19 23:57 ID:???
751ご冗談でしょう?名無しさん:02/10/19 23:57 ID:???
電磁気とか解析力学で、
f(x+δx,y) - f(x,y) = (∂f/∂x)δx
と言う式が度々出てきますが、微積はちゃんとやったはずなのにこれがわかりません。
δxを左辺に移行したら偏微分の定義になってそれでいいかと思ってたけど、
ファインマン物理には「2次以上は無視する」と書いてあります。
ということはこれはなにかの展開なのでしょうか。
あと、yも微小変位した場合はさっきの考え方ができなくなります。どうゆうことなのか
教えてください。説明するのが難しい場合はそれが分かりそうな本を教えてください。
微積の教科書読み漁っても載ってません。
752つばめさん ◆dpLvO.T.cY :02/10/19 23:58 ID:fAT0AIzk
大学では、2次方程式も理解できないと思われました。
753ご冗談でしょう?名無しさん:02/10/20 00:10 ID:???
>微積はちゃんとやったはずなのにこれがわかりません。
(・∀・)ニヤニヤ
754ご冗談でしょう?名無しさん:02/10/20 00:14 ID:???
>微積の教科書読み漁っても載ってません。
(・∀・)ニヤニヤ
755ご冗談でしょう?名無しさん:02/10/20 00:14 ID:mRae01uZ
ちんこ演算子とまんこ演算子の交換関係を教えて下さい
756ご冗談でしょう?名無しさん:02/10/20 00:17 ID:???
>>751
高校の微積?
757ご冗談でしょう?名無しさん:02/10/20 00:18 ID:???
>>751
>微積の教科書読み漁っても載ってません。
読み足りなすぎ
758ご冗談でしょう?名無しさん :02/10/20 00:19 ID:6sHmRAoF
ヤングの実験で、2つのスリットのうち一方のスクリーン側の面を厚さa[m]
屈折率nの薄膜で覆ったときの光路差の求め方を教えてください。
759751:02/10/20 00:23 ID:???
わかった、無勉強は謝るから教えてください!
760ご冗談でしょう?名無しさん:02/10/20 00:25 ID:???
光路長は屈折率×距離でしょ。
覆わないほうはa[m]のところを覆うとna[m]
なのでこの差。
761ご冗談でしょう?名無しさん:02/10/20 00:25 ID:???
>>751
仕立屋シリーズ。
762ご冗談でしょう?名無しさん:02/10/20 00:26 ID:???
テイラー・・・・・
763751:02/10/20 00:28 ID:???
えっ?テーラー展開なんですか?
764ご冗談でしょう?名無しさん:02/10/20 00:31 ID:???
>>763
はい、じゃあ微積の教科書みなおそうね。
765ご冗談でしょう?名無しさん:02/10/20 00:33 ID:6sHmRAoF
02年北海道大
http://www.yozemi.ac.jp/nyushi/sokuho/hokkaido/zenki/butsuri/mon1.html
の(10)の問題で
地表すれすれを円運動する人工衛星の速さをv=√(gR),
打ち出す速さをuとして、エネルギー保存則で
{(v+u)^2}/2 + (-GmM/R^2)=0+0を計算すると√(2gR)-Rωとなって、
答えの√(gR)-Rωと違ってしまいます。どこが間違っているのか教えてください。
766751:02/10/20 00:37 ID:???
>>764
あ、なんとなくわかりました。今から計算します
767ご冗談でしょう?名無しさん:02/10/20 00:38 ID:???
素直にu=v-Rωでいいんじゃない。
768758:02/10/20 00:43 ID:6sHmRAoF
>>760
図に書いた場合、光が幅a[m]の薄膜を斜めに横切るので良く分かりません。
(光路差)=S2P-{S1P*(a/l)*n+S1P*(l-a)/l} (lはスリットからスクリーンまでの距離)
という式を立てたのですが求めることが出来ません。
「覆わないほうはa[m]のところを覆うとna[m]
なのでこの差」というのをもう少しくわしくを教えてください


769765:02/10/20 00:45 ID:6sHmRAoF
>>767
なぜですか。
770751:02/10/20 00:49 ID:???
確認しました。
ただまだちょっとわからないところもあるので自分で詰めてきます。
あー・・・勉強不足だなあ。っとに。
771ご冗談でしょう?名無しさん:02/10/20 00:50 ID:???
>>765
vの速度だと地表すれすれで衛星になれるんだよね。でいま地球が
Rωの速度をすでに出しているんだから、我々は(地表に対して)
あとどれだけ速度を出せばいいか。それだけのことです。
772765:02/10/20 00:53 ID:6sHmRAoF
>>771
ありがとうございました。ちなみに>>765の式は何が間違っているんですか。
773よっちゃん:02/10/20 00:53 ID:???
だれか光共振器、光検出器、照射の読み方がわからない
ので教えて下さい。
マジで困ってます。プレゼンしなきゃいけないんで・・・
774ご冗談でしょう?名無しさん:02/10/20 00:54 ID:???
>>758
式の中に出てくる変数が何を表してるのか分からんが
その斜めに入射するって言うのは本当に計算結果に効いてくるのかな?
大概この手の計算ってsinθ≒θにするでしょ。同様にcosθも1次まで残して
テーラー展開すると1になるので角度が多少変わっても効いてこないんでは
ないかねえ。
775ご冗談でしょう?名無しさん:02/10/20 00:54 ID:???
ひかりきょうしんき、ひかりけんしゅつき、しょうしゃ
776ご冗談でしょう?名無しさん:02/10/20 00:56 ID:???
>>765
たぶん左辺第二項(-GmM/R^2)に1/2を掛けるのを忘れているんでは?
777758:02/10/20 00:58 ID:6sHmRAoF
>>774
ありがとうございました。
778よっちゃん:02/10/20 01:18 ID:???
>>775
ありがとうございました。
779ご冗談でしょう?名無しさん:02/10/20 02:38 ID:???
東大物理の教授で、退官前に他の国立大学に移った人がいたのですが、
なんでなのでしょう?移籍先の国立大学も十分レベルの高い大学であること
には違いないのですけど、わざわざ移籍する理由が分からない…。
780ご冗談でしょう?名無しさん:02/10/20 04:57 ID:9pMkePvZ
東工大に移った安藤先生ですか?
781657:02/10/20 07:30 ID:???
どうして曲がった時空での微小ベクトルAが
Γ^μ_αβ(x^α+dx^α)A^α(x^α+dx^α)dx^β
になるんですか?
どうしてもわかりません
誰か教えてください
782ご冗談でしょう?名無しさん:02/10/20 12:50 ID:QGb3sLvD
電車の中にラジコンヘリをホバリングさせてたらどうなりますか?
http://ex.2ch.net/test/read.cgi/news/1035084647/l50

この答えどなたか分かりますか。
・発車前にホバリングさせていた場合
・走行中の電車でホバリングさせた場合
783ご冗談でしょう?名無しさん:02/10/20 12:52 ID:p9J38F3n
http://gooo.jp
無料掲示板
無料レンタル掲示板
784ご冗談でしょう?名無しさん:02/10/20 15:44 ID:???
>>782
電車が加速するとき体が後ろに傾くだろ?
F-m・a=0だよ
785☆Qのイタズラ☆:02/10/20 17:48 ID:LCgnesid
フェルミオンの自由度を求める時、7/8を
かけるのは、どうしてですか?
786ご冗談でしょう?名無しさん:02/10/20 19:15 ID:2BaTY3yS
ディラック方程式を導くとき、反交換関係から4次元行列α、βを求めてますけど、
もっと次元が高い行列を選ぶことも出来るんですか?
だとしたらそのときの方程式は何を表しているんですか?
787七資産:02/10/20 19:16 ID:G7CBm754
偏光ゴーグルってどうしてまぶしさが減るの?
788   :02/10/20 19:16 ID:SwP1ToND
>>786
 知っている癖に、試すように聞くな!
789ご冗談でしょう?名無しさん:02/10/20 19:19 ID:961bzF/W
すいません。eVとVの間の関係式を教えてください。
790ご冗談でしょう?名無しさん:02/10/20 19:31 ID:???
>>787
雪に反射した光は横偏光している。偏光板の軸方向を縦向き
にしておけば縦偏光しか通らないので反射光をカットできる。
791七資産:02/10/20 19:34 ID:G7CBm754
>>790
ありがとう!!

792ご冗談でしょう?名無しさん:02/10/20 19:36 ID:???
>>789
eVは電子一個を1ボルトで加速した時のエネルギー。
Vは電圧(電位差)なのでVに電子一個の電荷をかけたものがeV
eV=1.6E-19・V
793ぐっさん:02/10/20 19:45 ID:G7CBm754
@電子レンジの周波数はいくら?
A内寸法が機種によらず同じなのはなぜ?
Bどうしてものがあたたまるの?

おしえてください。Bの答えは物体と電磁波が衝突する際に熱が発生するから?
794ご冗談でしょう?名無しさん:02/10/20 20:06 ID:???
2.45GHz
同じかどうか知らんが同じならば中で定常波を作ってるんだろう
マイクロ波を照射するということは急激に電場の向きを変えることに
なる。水分子のような極性を持った分子が電場が変わるごとに向きを
変えるので電磁波のエネルギーが水分子の熱エネルギーになる。
795ぐっさん:02/10/20 20:24 ID:G7CBm754
>>794
ありがとう!!
796ぐっさん:02/10/20 20:34 ID:G7CBm754
携帯電話が異なる周波数帯の機器に影響を与えるのはどうしてですか?
797ご冗談でしょう?名無しさん:02/10/20 20:37 ID:???
シグナルにノイズを乗らせるからとか、
普段、微小電流しか流れない回路にでかい
電流を流しちゃうんじゃない?
798ご冗談でしょう?名無しさん:02/10/21 00:38 ID:???
>>794
Bは、あの周波数の電磁波が
水分子の回転励起エネルギーに対応してるからであって、
極性は関係ないと思うが。
799磁界のエネルギー:02/10/21 01:11 ID:DVZ6XR3Y
 コイルを2つ並べたトランスについてお聞きしたいのですが、
1次コイル側でI1の電流が流れたことにより2次コイル側に
I2の電流が発生したとします。
 このときの1次コイル側と2次コイル側のエネルギーの関係が分かりま
せん。
 エネルギー保存の法則より、1次コイルで発生したエネルギーP1
の一部が磁界(エネルギー)P10(P1>P10)となり、P10のエネルギーの一部
が2次コイルのエネルギーP2(P10>P2)のエネルギーを発生しているのでしょうか?

1次コイルに加えたエネルギー=磁界エネルギー+2次コイルのエネルギー?
(すいません、1次コイルにエネルギーP1を加えると、どういう関係式で
エネルギー保存の法則が成り立つのでしょうか?)
800ご冗談でしょう?名無しさん:02/10/21 01:20 ID:1f+5ZfpP
801ご冗談でしょう?名無しさん:02/10/21 02:02 ID:???
>>794,>>798
電子レンジの場合は、水分子の集合の回転緩和による吸収でしょう。
分子の集合が極性をもっている様です。
だいたい25GHzに誘電損失のピークがあります。
といってもものすごくブロードなピーク。
よって2.45GHzでもOK。

分子単体の振動モードはずーっと高いエネルギ(赤外とか)の話です。
802高1:02/10/21 02:45 ID:WQLR/XSL
すみません。リアル工房ですが、どなたかこの問題を教えて頂けますか?
素人質問ですみません。

               A
               。
           |  。。
           ↓ 。 。
            。  。
           。   。
        B  。。。。。。C
------------------------------------------------------

             床

のような三角形(正確には、こちらに三角形の面を向けた三角柱)
の物体が床に置いてあります。この三角形の斜辺に矢印の様に下向きに
力 F を加えます。このとき三角形は右向きに力を受け右に進み始め
ますが、その時の三角形の加速度を求めなさい。
尚、三角形の質量は M 。角∠ABCはθ。 三角形と床の間の摩擦は
無視できるものとする。
803132人目の素数さん:02/10/21 03:21 ID:7uDYROwc
torrって単位は何て読むのですか?
教えてください。
804ご冗談でしょう?名無しさん:02/10/21 04:48 ID:???
>>803
トール
805ご冗談でしょう?名無しさん:02/10/21 04:52 ID:???
>>802
面に垂直方向に押す力だけが推進力になるわけだよね。
柱の斜面、地面の両方についてこの考え方を適用すると...。
806805:02/10/21 04:54 ID:???
>>805
って、地面の方は違うな。
地面を動かすんじゃないから。
まぁ、どの方向の力が推進力になるかは考えれば分かると思う。
807☆Qのイタズラ☆:02/10/21 11:32 ID:6NaSvfqn
フェルミオンの数を求める時、自由度に7/8を
かけるのは、どうしてですか?

808ご冗談でしょう?名無しさん:02/10/21 14:33 ID:XswjmzcS
偏微分てどういう意味かわからんのです誰か教えて
809ご冗談でしょう?名無しさん:02/10/21 15:10 ID:???
素粒子物理における「現象論」というのはなんのことですか?
810ご冗談でしょう?名無しさん:02/10/21 15:52 ID:???
QCDなんかのこと。
811ご冗談でしょう?名無しさん:02/10/21 17:09 ID:LRv+gTZl
>>802
まず辺abに対する垂直成分を考えます。この成分の大きさは、力FがfNだとすると平行四辺形作ってfsinθ。次にfsinθを地面に対する垂直成分と水平成分に分けます。水平成分は右方向に
(fsinθ)sinθ=fsin2θ。この力で質量Mの物質を押してるので、fsin2θ/M(m/s2)

812ご冗談でしょう?名無しさん:02/10/21 18:39 ID:???
>>811
答え丸教え(・A・)カコワルイ
813ご冗談でしょう?名無しさん:02/10/21 18:44 ID:???
>>808
複数のパラメータを持つ関数で、
そのうち1つのパラメータのみに注目して微分すること。
他のパラメータはただの定数のようにみなすわけね。
例えば、
 f(x,y) = x^2 + xy + 3y^2  (^n は n 乗を表す)
の時、x での偏微分は
 ∂f/∂x = 2x + y
で、y での偏微分は
 ∂f/∂y = x + 6y
になるわけね。
814ご冗談でしょう?名無しさん:02/10/21 19:05 ID:hr0961bN
大統一理論が完成すれば重力制御が可能になるとムーに書いてあったのですが本当ですか?
815ご冗談でしょう?名無しさん:02/10/21 19:13 ID:???
ムーってまだあったのかYO(w
816ご冗談でしょう?名無しさん:02/10/21 19:44 ID:???
そういや、同じ学研の似非科学雑誌UTANってのがあって、最後にはほとんど
ムー化(エロ本化)して廃刊になったな。
編集長でもムーから流れてきたのだろうか?
817ご冗談でしょう?名無しさん:02/10/21 19:57 ID:cZd1ubHT
811ですけどsinではなくcosでしたどうもすいません。
818809:02/10/21 20:48 ID:???
>>810
なるほど、QCDとかですか・・・

というか、現象論というのはそもそも
どういうものを言うのか、
何をもって現象論とそうでないものを分けるのか
わかりません。(馬鹿な文系でスイマセン・・)

GUTは現象論、SUTは非現象論なのでしょうか。
819ご冗談でしょう?名無しさん:02/10/21 21:07 ID:???
820ご冗談でしょう?名無しさん:02/10/21 21:09 ID:???
>>819
ヴァギナ
821ご冗談でしょう?名無しさん:02/10/21 21:26 ID:sWFo2yH+
(1)OPアンプの原理、およびその使用法(例えば加算器、積分器、フィルタなど)について
(2)方形波に含まれる周波数成分と方形波応答が意味するものについて
この二つについて調べたいのですが、載ってるサイト知っていれば教えてください。お願いします。
822ご冗談でしょう?名無しさん:02/10/21 22:30 ID:???
>>818
現象論は、観測された実験の結果にたいする考察に基づいて、
これから観測されるであろう現象などを吟味するようです。
具体的には、実験結果と矛盾しないように標準理論を拡張
したモデルを作り,新粒子なんかを予言したりします。
「現象論」という言葉には、「なぜ」とか「どういうしくみに
なっているのだろう」とかいった、細部の詳しいところまでは
追求しないといったニュアンスがあるのかもしれません。
それで、素粒子論には、もう一方の「場の理論」とよばれる、
より厳密に理論的、数学的側面を研究する分野があります。
GUTとSUTについては何と言えばよいのかわかりません。
823809:02/10/21 23:52 ID:???
>>822
ありがとうです!

なんとなくわかったような気がしてきました。
824ご冗談でしょう?名無しさん:02/10/21 23:54 ID:???
>>816
UTAN読んでた。
ムーは今でも見かけりゃよんどる。
ムーってエロ本か?
825ご冗談でしょう?名無しさん:02/10/22 01:18 ID:???
単純立方格子、体心立方格子、面心立方格子、
ダイヤモンド型格子、六方稠密格子の結晶構造因子の数値を教えてください。

求め方も教えていただけるとうれしいです。
宜しくお願いします。
826ご冗談でしょう?名無しさん:02/10/22 01:36 ID:???
>>824
あ、ごめん。ムーがエロ本じゃなくって、UTANの廃刊間際では毎号のように
「SEXの科学」とか特集してた。それと、ムー的な古代遺跡の謎とかが内容。
一度、そういうのを特集したらちょっと売れたので味を占めたと思われる。
でも、結局あぼーん。
日本ではまともな一般向け科学雑誌なんて存続できないと感じた。
827ご冗談でしょう?名無しさん:02/10/22 01:51 ID:???
光が回折して2dsinθ=nλとなるときに強めあうっていう式なんですけど、
どこからどうやって導き出してくればいいんですか?
828ご冗談でしょう?名無しさん:02/10/22 03:11 ID:ICtgxNuL
海岸線って何でフラクタルなんでしょうか。フラクタルになる要因を知りたい。
樹がフラクタルになるのは納得がゆきますが。
829ご冗談でしょう?名無しさん:02/10/22 06:13 ID:yCA4IyDK
今JJAPのホームページにアクセス出来ないよね?
830ご冗談でしょう?名無しさん:02/10/22 12:14 ID:???
>>781
少なくとも A と dx には比例するからでしょ。
その比例係数が Γ てことで。
831ご冗談でしょう?名無しさん:02/10/22 12:55 ID:vulSdIck
九州大学のエネルギー科は、他大学でいう電気科みたいなものなのでしょうか
832ご冗談でしょう?名無しさん:02/10/22 13:13 ID:bfMAGJ2W
夜空に見える星は恒星ですよね。

こうせい 【恒星】
天球上の互いの位置をほとんど変えず、
それ自体の重力により一塊となり、光や熱
などを放射している星。星座をつくってい
る星や太陽はこれに当たる。

太陽以外の恒星も太陽と同じように
燃えてるから光るんですか?
833ご冗談でしょう?名無しさん:02/10/22 15:11 ID:???
>>827
その式の上に書いてなかった?
834ご冗談でしょう?名無しさん:02/10/22 15:13 ID:???
>>832
そうです

ただ 夜空に輝く星の中には 
月 や 火星,木星,金星など,自分で光ってはいないけど
太陽に照らされて輝いている星もあります。
835832:02/10/22 15:22 ID:???
>>834
ありがとうございました。
836qwer:02/10/22 19:46 ID:t0glHzD2
水平で滑らかな氷の上で、質量が等しい二人が質量の無視できる長さ2aの綱
の両端につかまって、全体の重心のまわりに円運動している。
(1)このとき円運動の角速度をωとして重心のまわりの全角運動量および運
   動エネルギーを求めよ。
(2)二人が綱を手繰り寄せた後、綱の長さが2bとなり円運動を続けた。
   このとき重心のまわりの全角運動量および運動エネルギーを求めよ。

この問題が分かりません。誰か教えて下さい。

837ご冗談でしょう?名無しさん:02/10/22 21:08 ID:OM/S929Z
下の問題ですが、2とおりの答えができてしまいました。
解答1が正しいように思うのですが、解答2のどこが間違いであるのかが
わかりません。おしえてください。

<問題>
円形(面積A=0.01m^2)の排出口から速度S=10m/secで
液体が排出されている。排出口の近くで排出液を完全にさえぎるように平板を置いた。
平板の受ける力F[N]を求めよ。ただし液体の質量密度ρ=1000kg/m^3とする。

<解答1>
時間t[sec]の間に、質量m[kg]の水が速度S[m/sec]で平板に衝突し、
力F[N]を与え続けたとすると運動量保存の法則より、
m×S=F×t
F=m×S÷t=ρ×(V÷t)×S=ρ×(A×S)×S=ρ×S^2×A
=1000×10^2×0.01=1000[N]・・・(答え)

<解答2>
平板は液流の全圧を受けるので、
F=「全圧」×A
「全圧」=「静圧」+「動圧」、「静圧」=0(大気圧)、「動圧」=ρ×S^2÷2より、
F=(ρ×S^2÷2)×0.01=1000×10^2÷2×0.01=500[N]
838☆Qのイタズラ☆:02/10/22 21:16 ID:fJ2zNzq+
828>
なぜ樹のほうは納得行くんです?それが知りたい。
有機・無機の違いはあるが、どっちも自然物なのに。
839ご冗談でしょう?名無しさん:02/10/22 21:17 ID:mIjHzuYD
垂直力と縦分布力との関係を導き出せといわれたんですが、どういうことかわかりません。
840スター10%:02/10/22 21:20 ID:WM96Nhm5
タグをつけない/つけるには、
どうするのですか?
841ご冗談でしょう?名無しさん:02/10/22 22:50 ID:???
>>837
衝突後の液体の運動量はゼロになっちまうのですか?
842ご冗談でしょう?名無しさん:02/10/22 23:29 ID:???
漏れもUTAN読んでた。おかげで複素熱力学とかヒランヤとかいろいろ信じ込まされた。
でも志水一夫氏のこんにゃく問答は面白かった。
843ご冗談でしょう?名無しさん:02/10/23 00:10 ID:Rz6VG+KE
こんなところでする質問にしては少し稚拙すぎるかもしれません。ごめんなさい。
ただいま試験勉強中ですが、一つわからない事があるので質問させてください。
仕事についてですが、ばね定数kのばねを自然長XoからX1まで引き伸ばした時のこの力がした仕事というのは
どのようにして求めればいいのでしょうか?
単純にk(X1-Xo)が仕事になるのですか?
844ご冗談でしょう?名無しさん:02/10/23 00:17 ID:Rz6VG+KE
って・・・・
1/2k(X1-Xo)^2でしたね・・・・。
あぁ俺何やってるんだろう。・゚・(ノД`)・゚・。
絶対理学部物理学科うかってやる。
845ご冗談でしょう?名無しさん:02/10/23 00:35 ID:???
>>843-844
 なんかどこまで分かってるのか、どこをつっこむべきか・・・。
 とにかく、もう一度教科書だか参考書だかを見直せ。
846828:02/10/23 01:50 ID:???
>>838
レスサンクス。
ある程度伸びると分岐する樹があったら、それはフラクタルになりそうだなぁ、と。
成長してゆく有機物にくらべて、擦り減ってゆく一方の無機物はフラクタルになるのか?

…と、思ったんだが、海岸線は海側から(とも限らんが)削れてゆくのと陸側から成長してゆく
ののせめぎあいだから、フラクタルになるのか。
ダムの水位とか株価チャートもフラクタルだしね。

…って、何でこの辺ってフラクタルになるのかなぁ?

847828:02/10/23 01:58 ID:???
>846
読み返して見ると電波な文章だ。前半部分は無かった事にして。スマソ
848ご冗談でしょう?名無しさん:02/10/23 02:21 ID:75Xv0cOY
いまさら人に聞けない質問!レポートってどのように書けばいいのですか?実験・考察・感想などの書き方の手順を詳しく教えてください。
849837:02/10/23 06:40 ID:Eokb8F68
>>841
>衝突後の液体の運動量はゼロになっちまうのですか?
はい。そういうことでよいと思います。
850質問です:02/10/23 08:01 ID:jAYX3Rw+
水などの液体の電気伝導理論を詳しく解説してある
良書があったら教えて下さい。
851ご冗談でしょう?名無しさん:02/10/23 10:58 ID:FwtQSRz6
814ですが、、、もう一度、、
大統一理論が完成すれば重力制御が可能になるとムーに書いてあったのですが本当ですか?
852ご冗談でしょう?名無しさん:02/10/23 11:16 ID:???
ムーってまだあったのかYO(w
853ご冗談でしょう?名無しさん:02/10/23 13:07 ID:???
>>851
大統一理論は「電磁力+弱い力+強い力」をアレする理論。
重力は放置。
854ご冗談でしょう?名無しさん:02/10/23 18:20 ID:FwtQSRz6
>>853
なるほど、、そうでしたか、やはりムーはアレですね、、、、、
855     :02/10/23 18:49 ID:GZKoIzH4
>>853
 じゃ、統一理論は?
856統一理論のいろいろ:02/10/23 22:08 ID:???
Einsteinの統一場理論=重力理論に電磁場を取り込もうという理論。
そもそもはWeylの初期ゲージ理論に始まり、
射影相対論や波動幾何学などに発展するが、行き詰まる。
Eddingtonの基礎理論なんかもこの系統だったような気がする。
Kaluza-Kleinの高次元アプローチだけ後に復活。

電弱統一理論=電磁気力と弱い核力の統合。重力は無視。
Weinberg-Salamが60年代に完成。

大統一理論=電弱力と強い核力の統一理論。重力は無視。
50年代におけるHeisenbergの非線形スピノール場や
湯川の非局所場などの研究が先駆と言えるか。
Georgi-Glashowが70年代に完成させたが、観測と不一致。
超対称性大統一理論(SUSY GUT)に発展。

超統一理論=すべての力の統一。60年代におけるDiracの泡模型や
湯川の素領域理論などが先駆。1976年、N=8超重力が登場。
80年代、Superstringに発展。PenroseのTwistorも登場するがこちらは行き詰まる。
現在はD=11のM理論や、Wilson-Polyakov Loopの量子重力への適応など
主にStringアプローチの延長上での研究が継続されている。
857ご冗談でしょう?名無しさん:02/10/24 03:26 ID:0Tz1PAs4
ttp://www.colorado.edu/physics/2000/applets/satellites.html

こういう動きをFLASHで作りたいんですが、計算式がぜんぜんわかりません。
こういうのに詳しいHPで初心者でもわかりやすく解説してあるところ誰か知りませんか?
858ご冗談でしょう?名無しさん:02/10/24 04:58 ID:ZY8gPnRS
だぁ。
レポート書いてるんだけど片対数グラフの使い方がいまいち分かりまソン。
縦軸は0にならないってことで0.01とすると最初の1メモリは0.01?0.001?
859ご冗談でしょう?名無しさん:02/10/24 05:01 ID:e14Fn04u
0.02
860ご冗談でしょう?名無しさん:02/10/24 05:03 ID:???
もしくは0.011
861ご冗談でしょう?名無しさん:02/10/24 06:17 ID:07Imhb1g
ここの名無しさんの由来は何なんでしょうか?
862ご冗談でしょう?名無しさん:02/10/24 08:54 ID:???
だぁ。
レポート書いてるんだけど線型グラフの使い方がいまいち分かりまソン。
縦軸は横軸が交わるところを0とすると最初の1メモリは0.01?0.001?

























...ってゆーくらいにオマヌケ

好 き な よ う に 定 義 し て 使 え
863ご冗談でしょう?名無しさん:02/10/24 14:31 ID:gYcMdbqR
>>861 そんなことも知らない厨房の退出希望!
ttp://www.iwanami.co.jp/.BOOKS/60/3/6030050.html
864ご冗談でしょう?名無しさん:02/10/24 16:32 ID:wrAu++pD
磁石は磁力でくっついてるかの確認実験でホワイトボードにせっけん水
をつけると磁石がすべりおちる実験レポートをかくのですが、その時に
必要な重要事項を教えて頂けませんか?
865ご冗談でしょう?名無しさん:02/10/24 16:36 ID:yRthrx6l
化学板でも聞いたのですが、物理板で聞いた方が良いと言われたので、聞きます。
核融合は何と何が融合するのでしょうか?
866ご冗談でしょう?名無しさん:02/10/24 16:46 ID:LLZDskCQ
>>865
原子核と原子核。

例えば
太陽の反応:4H→⁴He
水爆の反応:D+T→⁴He+n
見慣れない記号があるかもしれないが、ただの反応式だ。
H: 水素、D: 重水素、T: 三重水素、⁴He: ヘリウム4、n: 中性子
867ご冗談でしょう?名無しさん:02/10/24 16:53 ID:???
>>866さん、有難う御座います。
868ご冗談でしょう?名無しさん:02/10/24 18:47 ID:???
核融合で思い出したのですが、核融合って何度までいきますか?
6000度くらいですか?
869ご冗談でしょう?名無しさん:02/10/24 21:01 ID:JHdt4svq
チェルノブイリ原発事故。
オゾン層破壊についての問題があるとおもいますが、
関係あると思いますか?
チェルノブイリのオゾン層はすくないですよね?
870釜山:02/10/25 02:04 ID:HmdRTK49
固体の比熱を考えるには格子振動について考えればよい。と書かれていたんですが、どうして格子振動による運動エネルギーが比熱に関係してくるんですか?
871ご冗談でしょう?名無しさん:02/10/25 02:14 ID:NaBNWkZh
>>868
核融合で何度まで行くか知らないが、太陽の中心は1600万度と言われている
ので何億度は軽く行くでしょう。
872ご冗談でしょう?名無しさん:02/10/25 02:18 ID:NaBNWkZh
>>870
比熱を計算する時に、格子のポテンシャルが関係してくるでしょ。
格子振動について考えろってのはそのポテンシャルを考ろということでは?
873ご冗談でしょう?名無しさん:02/10/25 02:27 ID:???
>870
格子振動以外の何が比熱に寄与すると思うわけ?
まー極低温では伝導電子の比熱が効いてくるけど。
874ご冗談でしょう?名無しさん:02/10/25 07:48 ID:DHnVrc/K
bicontinuous cubic phase(QU相)
inverted hexagonal phase (HU相) って何ですか?
何が何だかさっぱりわかりません。
どなたかお教えください!!!
ほんと切羽詰ってます!!!(TT)
875ご冗談でしょう?名無しさん:02/10/25 11:06 ID:QeTkgTAS
>>869
地球には東西方向の風が流れているので、
チェルノブイリだけオゾン層が薄いってことはありえないよ
876ご冗談でしょう?名無しさん:02/10/25 13:54 ID:???
放射能とオゾンって関係あんの?
どっちかっつぅとオゾンができちゃいそうな気さえするんだが。
877ご冗談でしょう?名無しさん:02/10/25 15:11 ID:hXzDgEgt
天文・気象板でも同様の質問をしているのですが、
物理板の方が適切だったかなと思ってこちらに質問し直します。

(月の自転と公転の周期が同じという説明に関して)
例えば、ボールに紐を付けて振り回したとき、
ボールと紐の接着点は常に中心の振り回す手の方向にあるわけで、
月と地球との関係と似た状態になるんですよね?
この紐の付いたボールも、自転と公転の周期が一致していると
説明していいのでしょうか?
むしろ、月が常に地球に同じ面を向けているのは、
「円運動系で静止しているため」みたいに説明した方がわかりやすくないですか?
(素人のため用語の使用は不正確だと思いますが、そこのところはおゆるしを。)

あと、円運動する物体の常態としては、
宇宙空間(絶対空間)に対して自転しない円運動と、
公転と自転を一致させる円運動、どちらが自然なのでしょうか。
私は普通、後者を自然体・基準とするべきと思うのですが。

では、どなたかお答え、よろしくお願いします。
878    :02/10/25 15:35 ID:???
>>877
 月は自転している。それは、自転による遠心力が月面でも生じるからだ。
879ご冗談でしょう?名無しさん:02/10/25 16:03 ID:qDadIjqX
自転車に乗ると何故歩くのと同じエネルギーを使ってあんなに速くなるの?
880ご冗談でしょう?名無しさん:02/10/25 16:25 ID:lr0Pa2zy
>>873
スピンあるいは分子の内部自由度
881sage:02/10/25 16:29 ID:lr0Pa2zy
>>875
風っておまえオゾン層の高度知ってるか?
882ご冗談でしょう?名無しさん:02/10/25 16:46 ID:nmJSZodv
>>879
エネルギー効率の問題
883ご冗談でしょう?名無しさん:02/10/25 17:57 ID:TGt8YdTd
音波の振動ってある定点を観測した場合、時間による3次元の関数ですか?
式で表すと、空気の変位Pとして

P=( x(t) , y(t) , z(t) )

であってます?
884熱力学初学者:02/10/25 18:08 ID:YAofsLoV
熱力学第1法則と内部エネルギーの全微分を使って定積比熱を求めるという問題に関して質問があります。

熱力学第1法則は
δQ=dU+PdV (1)
内部エネルギー(T,Vの関数です。)は、
dU=(∂U/∂T)dT+(∂U/∂V)dV (2)
ですよね。
で、(2)を(1)に代入して、
δQ=(∂U/∂T)dT+{P+(∂U/∂V)}dV
定積より、dV=0なので
δQ=(∂U/∂T)dT

ここで、
δQ/dT=∂U/∂T (3)
の左辺の処理というか、意味というか、これって何?って疑問に思います。
δ→dとすれば(3)が定積比熱となるわけですが・・・
勝手にそんなことをして良いのでしょうか???
熱力学を良く理解されている方からの明快な回答をお待ちしてます。
885ご冗談でしょう?名無しさん:02/10/25 18:11 ID:???
回路図で「○」の中に「=・」があるんですが何の記号ですか?
886ご冗談でしょう?名無しさん:02/10/25 18:24 ID:???
>884
第一法則をなぜ
dU=dQ+dW+dN
と書いてはいけないのかを調べればよいのれす。
887ご冗談でしょう?名無しさん:02/10/25 19:01 ID:qDadIjqX
自転車はどの要素でエネルギー効率が良くなるのですか?
888主婦のサークル:02/10/25 19:34 ID:6O3EOryQ
ほんの少し刺激を求めてる主婦のサークルを作りました!
私たち主婦が楽しめる安全でちょっとトキメキのある出会を求めて
只今男性会員を募集中です!! 女性会員も募集(^○^)/"
http://f-cc.com/~tokimail/
889ご冗談でしょう?名無しさん:02/10/25 19:58 ID:???
>887
その質問は結構難しいな。
車輪がはずみ車として働くので、片足を踏み込むときの力がもう片方の足を
持ち上げ、前進させるのに効率よく使えるとか?
890ご冗談でしょう?名無しさん:02/10/25 20:24 ID:???
歩行の場合、地面からの反作用を効率的にエネルギーに
転化できないからムダになっちまうんじゃない?
サイバネティクス+生理学に詳しいヒトならズバっと答えてくれそう。
891ご冗談でしょう?名無しさん:02/10/25 21:28 ID:???
>>887
地面との摩擦を減らすからでしょ。
892879=887:02/10/25 22:35 ID:q5TFvc9c
>>889-891
ご回答ありがとうございます。なんとなくしか私の頭では理解できませんけれども。
しかし摩擦を減らすだけで何十キロものスピードが出るもんでしょうか?
893ご冗談でしょう?名無しさん:02/10/25 22:47 ID:???
>>892
走るのと違って、体重を支えるのはサドルで、足を前に進むためだけに使えるし
エネルギーを失いにくいように構造が工夫されているからとかじゃないのか。
894ご冗談でしょう?名無しさん:02/10/25 23:34 ID:???
>>891
地面との摩擦が減るとスリップするよ(W
895861:02/10/25 23:38 ID:G2K4pGzO
>>863
ご親切に感謝します。
なにぶん物理とは縁遠い者なので厨房なのはご容赦ください。
896ご冗談でしょう?名無しさん:02/10/25 23:39 ID:TihG+aJ9
いろいんな説明を考えてみたが、めんどくさくなってきた。

とりあえず得た結論(感覚的)は、
歩くときには適度な傾斜の階段を登るとき
もっともエネルギー効率がいい。
897877:02/10/26 00:31 ID:2zHZL2DF
>>878さんへ
回答ありがとうございました。

遠心力ですか…。
素人なので今ひとつ実感が湧かないのですが、(すみません)
しっかり自転をしている状態なのだ、と言うことですね。
ということは、円運動する物体の向きは基本的には、
 ←
↓・↑
 →
と言う風に向きを変えるのではなく、
 ←
←・←
 ←
と常に同方向を向きながら円軌道を動くと考えるべき
ということでいいのでしょうか。
898ご冗談でしょう?名無しさん:02/10/26 00:47 ID:???
タイヤがある時
摩擦力を受ける→タイヤが転がる→進む→(゚д゚)ウマー

ない時
摩擦力を受ける→転がらない→止まる→(゚д゚)マズー
899HAL ◆2eOaL2U.Gg :02/10/26 11:39 ID:???
質問スレは常時age
900つばめさん ◆dpLvO.T.cY :02/10/26 12:38 ID:lhYxq8Le
>>864
磁力と重力と抗力と摩擦力の関係。どういう条件で板から離れるか、または滑るか。
そして、形式的に考えると、摩擦係数を石鹸で変えるほかに、
磁石と板の距離を離したり、板を傾けたり(無理か?)して、
力の板に水平な成分と垂直な成分をうまく変化させてみる。ことが試せる。
なんにしても、磁力の大きさ、摩擦係数まで分ればいいかな。
901ご冗談でしょう?名無しさん:02/10/26 12:44 ID:???
test
902ご冗談でしょう?名無しさん:02/10/26 13:09 ID:JBYa8Jvt
ベクトルを向きと大きさではなく、それを一般化しては、
具体的にどういったものと考えれば良いんでしょうか??
903ご冗談でしょう?名無しさん:02/10/26 14:46 ID:???
>>902
各次元に対する成分を羅列するってのが普通だと思うけど。
極座標表示ってのは次元が増えるとよく分からなくなる。
904ご冗談でしょう?名無しさん:02/10/26 14:53 ID:z4Oq8uA3
>>902
座標変換に対してノルムが不変なものと考えれ。
905ご冗談でしょう?名無しさん:02/10/26 15:51 ID:???
>>902
「一般化」と「具体的」は両立するかどうか分かんねえなぁ。
ベクトル空間の公理を満たす集合、としか・・・
>>904
ノルムが定義されてないベクトル空間だってあるよん。
906HAL ◆2eOaL2U.Gg :02/10/26 15:59 ID:???
>>902
・基底がとれる。つまり、dimV個のパラメータで表現される
集合
・一次変換が定義されて、行列と同一視可能

なんてところでどうじゃ。
907904:02/10/26 16:13 ID:???
>>905
「ベクトルを向きと大きさではなく」といってる人に
「ノルムが定義されてないベクトル空間」とかいったって
消化不良を起こすだけでしょ。
908ご冗談でしょう?名無しさん:02/10/26 16:31 ID:g2Yd0DWq
今日これから死ぬほど勉強します。明日は朝から晩までバイトですが、
そのままバイトに行きます。やるぞおー
909つばめさん ◆dpLvO.T.cY :02/10/26 16:53 ID:???
>>902-906
同じことだけど、まず、線形空間(=ベクトル空間)の公理とは、
空間内に要素 X、Yがあったとすれば、
(λを数として)、X+Y、λX も同じ空間内の要素になる。
基底というのは、空間のすべてのベクトルを一意的に表すベクトルの組。

それで、このようなXをベクトルという。
成分の羅列は、数ベクトル(ベクトルの一例)といい、
   また任意のベクトルの、ある基底に対する成分でもある。
また、局所的に線形変換が定義できるのもベクトル。
910つばめさん ◆dpLvO.T.cY :02/10/26 16:58 ID:???
上の説明の4行〜6行目は、
>それで、このようなXをベクトルという。
>基底というのは、空間のすべてのベクトルを一意的に表すベクトルの組。
の順がまとも。
911ご冗談でしょう?名無しさん:02/10/26 17:07 ID:+78BSBnz
磁化率の単位は無次元ですか?
912ご冗談でしょう?名無しさん:02/10/26 17:54 ID:aCv3fquY
こんにちは。物理板の皆さん。
地球科学板で質問したらこちらの方がいいということでやってきました。

今高校の、地震のところをやっているのですが、波は密度の小さいところから、
密度の大きいところにいくと遅くなると思っていたのですが、
なぜ、地震波は密度の小さい地殻から、密度の大きいマントルへいくと速くなるのでしょうか?
参考書などではそういうところに触れてないので、自分で考えてみたのですが、
混乱してしまってよくわからなくなりました。
わかる方がいたらよろしくお願いします。
913実験屋の精子:02/10/26 18:22 ID:0x93uVor
実験の測定結果の扱いで質問があります。
相対誤差 |凅/x|×100
で、
凅=|理論値−実験値|
なのはわかるんですが、xには理論値と実験値のどちらを入れたらいいんでしょうか?
914ご冗談でしょう?名無しさん:02/10/26 18:26 ID:+78BSBnz
>913
理論値
915ご冗談でしょう?名無しさん:02/10/26 18:44 ID:???
>>912
弾性波速度は,密度の平方根の逆数に比例だろ?
そんなアホなと思ったらこんなページがあった。

http://www.gec.gifu.gifu.jp/club/rs-shidou/8.pdf

いろんな岩石採ってきて弾性波速度の密度依存を調べたら密度に線形で上昇してたんだと。
フ〜ン。

弾性率が、密度の(1+α)乗で比例してるからでは?と言ってる。
詳しい事は、やっぱり岩石屋さんの領域の話しではなかろうか。

916実験屋の精子:02/10/26 20:31 ID:0x93uVor
>914
サンクス
917864:02/10/26 20:58 ID:sjNGL3w0
>>900
ありがとうございました!
918ご冗談でしょう?名無しさん:02/10/26 21:15 ID:mGC8FjJe
>>912
波の速さは密度だけに依存するのではないよ。
919昨日の自転車質問男:02/10/26 22:40 ID:K6NrtTxG
たまたま出来た自転車がエネルギー効率のもっとも良いものだったのですか?
それともエネルギー効率を追い求めた究極形が自転車なのですか?
今から人間工学を研究し尽くし、もっとエネルギー効率の良い
人力稼動移動装置(自転車、ローラースルーゴーゴー、キックボードなど)
は創り出せないものでしょうか?
920ご冗談でしょう?名無しさん:02/10/27 00:16 ID:???
ニュートリノの衝突後の軌跡はどんな感じですか?
衝突した相手(水電子?)の軌跡も教えてください。

飛んでくる方向がわかるってことは、
2筋の光が観測できるんでしょうか?
921912:02/10/27 00:30 ID:7WfwFCAS
>>915>>918
地球の内部では
密度だけでなく、剛性率(弾性率?)が大きく変化しているから、
こういう不思議なことになるみたいですね。
疑問だったことがかなりすっきりしました。
さらに詳しいことも調べていこうと思います。
どうもありがとうございました。
922ご冗談でしょう?名無しさん:02/10/27 00:32 ID:???
金属と音の関係についての質問です。
僕はフルートを吹いているのですが、銀のフルートと金のフルートとでは、
音色が全然違います。これはなぜですか?金属の密度とかが影響するのですか?
923ご冗談でしょう?名無しさん:02/10/27 00:37 ID:???
>>912
圧力が高いからだと思うよ
波の速度が密度と圧力に依存すると思うと
次元解析から

 速度=√(圧力/密度)

になるから、圧力が大きいと早くなるね!
924ご冗談でしょう?名無しさん:02/10/27 00:52 ID:llNcy45W
質問です

物体が同時に複数の物体に衝突した場合
それぞれの物体の衝突後の向きと速度はどうすればわかるのでしょうか?
925ご冗談でしょう?名無しさん:02/10/27 00:57 ID:fCiLzffU
質問!
僕は止まっています。例えば質量10kgの物体Aが20m/sで
動いているとすると運動エネルギーはE=mv^2/2=2000ですよね。
僕が同じ向きに10m/sで動くと僕から見て物体は10m/sで動いて
いるからE=500になるでしょ。ってことは運動エネルギーは
相対的なものなんですか?
厨房ですいません。
926ご冗談でしょう?名無しさん:02/10/27 00:58 ID:???
>>924
衝突時の物体の接点と重心との位置関係
927ご冗談でしょう?名無しさん:02/10/27 01:05 ID:???
>>925
他に影響する物体が無ければそれでよし

928ご冗談でしょう?名無しさん:02/10/27 01:09 ID:fCiLzffU
>>927
>他に影響する物体
ってなんですか?
929ご冗談でしょう?名無しさん:02/10/27 01:33 ID:???
他に影響する物体が無ければ

他に考える物体が無ければ
930ご冗談でしょう?名無しさん:02/10/27 01:49 ID:fCiLzffU
よくわからないけどそういうものなのですね。
どうも。
931ご冗談でしょう?名無しさん:02/10/27 01:56 ID:???
>925
運動エネルギーはガリレイ変換に対して不変量ではないという事です。
932うんもっこ:02/10/27 02:00 ID:Thp5mC0R
もし地磁気が存在しないと生体はどうなるのですか?
933ご冗談でしょう?名無しさん:02/10/27 02:01 ID:???
>>927
他に考える物体があってもいいと思うが?

>>925
普段、投げたボールの運動エネルギーを考える時、
地球が動いていることなんて考えてないよね。
太陽を中心にそのボールの運動エネルギーを考えるのと
地球を中心にそのボールの運動エネルギーを考えるのとでは
違ってくるよね。
それと同じこと。
このことに自分で気が付いたのはなかなかエライ。

そこで、光の速度が変わらないと仮定できるなら、
一体どういうことが起こるのか考えてみよう。
934ご冗談でしょう?名無しさん:02/10/27 02:15 ID:fCiLzffU
光のエネルギーは観測者によらず一定ってことですか?
935ご冗談でしょう?名無しさん:02/10/27 02:25 ID:???
>>923
次元解析は、依存する物理量が確定している時に使わないと危険。

弾性論から出てくるように、速度 ∝(弾性率/密度)^0.5で、
弾性率と密度がそれぞれ温度と圧力の関数になる形で速度が決まっているのだと思う。
(915の言うような弾性率の密度依存効果も考慮する必要あり。)
936ご冗談でしょう?名無しさん:02/10/27 02:58 ID:guNy2xqw
>>934
ところが光は質量を持たない。
じゃ、光の運動エネルギーってどうなるのかというと、
光の振動数に比例する形で得られる。
でも、ドップラー効果(厳密には音の場合とちと違うけど)のせいで
動いている視点から見ると振動数は変わる。
つまり、エネルギーも変わる。

あー、いろいろ考えたら前提知識かなり要るから
相対論まで誘導するのは無理っぽいかな。
質量エネルギー E=mc^2 と、
x << 1 の時 √(1+x) は約 1 + x/2 ってのを教えるから、
あとは自分で頭ひねってみると面白いと思うよん。
937ご冗談でしょう?名無しさん:02/10/27 03:10 ID:???
>>936
mは教えないのね。イジワル
まぁ、一番の意地悪は、初心者に相対論的質量を吹き込むことだが

938924:02/10/27 03:17 ID:llNcy45W
>>926

検索で衝突+重心にするとそれらしいサイトが発見できたので勉強してみます
(衝突+角度、速度では上手くヒットしませんでしたので・・・)

ありがとうございます
939 :02/10/27 03:30 ID:???
日本での地磁気の水平分力、垂直分力、伏角を教えてください。
940ぶちゅり:02/10/27 11:13 ID:gD576tqL
もし地磁気が存在しないと生体はどうなるのですか?


941ぶちゅり:02/10/27 12:31 ID:gD576tqL
おしえてよん♪
942ご冗談でしょう?名無しさん:02/10/27 13:33 ID:x4CzZqMG
重力というのは地球の引力と自転による遠心力との合力だと認識していますが、公転による遠心力は関係ないのでしょうか?
もし公転の遠心力が影響するというのであれば昼と夜では体重が変わってきますか?
回転している物体の重心の内側と外側でどのように遠心力が作用するか知らないので、ちょっとそこから先がわかりません。
943ご冗談でしょう?名無しさん:02/10/27 15:02 ID:???
物質の塊である人間および脳が
物質を観測するという物理現象が不思議なのですが
だれか説明してちょ
944ご冗談でしょう?名無しさん:02/10/27 15:22 ID:RNEsGPXF
いまTVで関取どもがパンチ力測定してるんだが、アノ手の機械って何測定してんの?
200kgとか言ってるから瞬間的な力なんだろうけどバッグ自体は怪我を防ぐ配慮か暖簾に腕押しな感じになってて、なんともすっきりしない。
衝撃のピークは怪我覚悟でガツンとやらんと出ないだろ。
力積計った方がおもろいと思うんだがナー
945ゆうた:02/10/27 16:59 ID:5FsqXPbm
現在高校一年の男なんですけど。物理の成績が最悪なんです・・・
なんだかいまいち勉強方法がわかんないです。
等加速度直線運動とか自由落下の公式ありますよね。問題を解くとき、
公式を覚えておいた方がいいんですか?グラフを書いてやったほうがい
いんですか?
946ご冗談でしょう?名無しさん:02/10/27 17:16 ID:JmCUyxFA
947ご冗談でしょう?名無しさん:02/10/27 17:59 ID:RNEsGPXF
大学以上まで物理やるような人間は高1物理あたりだと無意識に通過してたりするだろうからどうアドバイスしていいか困るだろうなあ。
現象と数式の関係が直観できるようになってないうちは飽きるほどグラフと戯れといたた方がいいんでないかな。
948ご冗談でしょう?名無しさん:02/10/27 23:37 ID:+GILQYXZ
グルーオンって何で8種類になるんですか?
3^2−1=8 って書いてあったけど
具体的にその8種類を挙げてみてください。
949ぺリクルス:02/10/27 23:45 ID:chLhWnwS
経済学者に、ワルラス均衡理論の競り人は存在しないじゃないか、そんな曖昧な価格理論で世の中論じられるのか?といった。
そうすると、その経済学者は物理学だって真空を仮定して理論を組み立てている、と言った。
俺がここで疑問に思ったのは、「真空」はこの世に存在するけど、ワルラスの競り人はこの世に存在しないということだ。
しかし、その経済学者によるとこの世に「完全な真空」は存在しないそうである。物理学をやっているひとから考えても「完全な真空」は存在しないのであろうか?
950ペガサス:02/10/28 00:49 ID:???
当然。
真空には、膨張力がある。
宇宙は、「加速度」を、速めて膨張している。
951ご冗談でしょう?名無しさん:02/10/28 01:00 ID:CqosJh2Y
重力加速度と重力場は同じものなのだしょうか?
違うのでしたら重力場とはなんですか?
952ご冗談でしょう?名無しさん:02/10/28 01:11 ID:???
質点に重力加速度を与えるものが重力場です。
953ミねこ猫彡☆(゚Д゚)ゴルァ ◆hXvyVozAPo :02/10/28 01:13 ID:31S13kQM
簡単に言っちゃうと
重力場とは重力における加速が生じる系の事。
重力加速度は重力場における重力加速の大きさ。
954951:02/10/28 01:15 ID:CqosJh2Y
ではもし式で表そうと思ったらどうなります?あらわせますか?
例えば電場はE=kq/r^2みたいに書きますよね。
955ご冗談でしょう?名無しさん:02/10/28 01:25 ID:???
>>948
グルーオンはカラーを交換します。
そのためにはカラーと反カラーからなる必要があります。
群は
 3(x)3~=8(+)1
 (x) :直積
 (+) :直和
 ~  :「反」
と既約分解でき、そのうち1はカラー1重項で
カラーの交換に寄与しません。

その他の8つは、
 RG~, GR~, RB~, BR~, GB~, BG~
と、RR~, GG~, BB~ から作られた
1重項にならない、上の6つと直交するものになります。
1重項は (RR~ + GG~ + BB~)/√3(1/√3 は規格化定数)になります。
他の2つは (RR~ - GG~)/√2 と
(RR~ + GG~ - 2BB~)/√6 になります。

D 波とよく似ているので、
それと比べると分かりやすいかも。
D 波が6成分じゃなくて5成分になるってのも、
結局 S 波成分が出てくるからですよね。
-1 ってのはそれを除外するものです。

D 波では xy と yx が区別できませんが、
カラーでは ~ が付いてるので RG~ と GR~ は区別できます。
違いはそれだけです。
956ご冗談でしょう?名無しさん:02/10/28 01:33 ID:???
>>951
重力場を決定するのが G = k T
その重力場中での運動を決定するのが (d^2 x/ dt^2) = Γ(dx/dt)(dx/dt)
重力加速度はこの運動から求められる。

メンドイので添字は省略.
957951:02/10/28 01:36 ID:CqosJh2Y
なるほどなんとなく理解できるようになりました。
一見簡単そうなのは意外と難しかったり・・
大変ありがとうございました。
958ご冗談でしょう?名無しさん:02/10/28 05:39 ID:???
>>954
つーかニュートン力学の範囲だったら
重力場も電場と同じ形だろ何が疑問なんだ?
959ご冗談でしょう?名無しさん:02/10/28 12:17 ID:x6FsQkpK
地球自体にかかる遠心力を忘れてる。

(太陽)       (地球)
         → →→ →→→

地球から見ると
(太陽)       (地球)
         ←  ・  →

つまり、昼と夜とで公転遠心力の影響は同じ。

ただし、[昼と夜]と[朝と夕]には差がある。
この差は、太陽重力の差とともに、大潮と小潮の違いを作る。

>回転している物体の重心の内側と外側でどのように遠心力が作用するか
遠心力の公式GMm/r²をそのまま使うだけですが何か?
960954:02/10/28 12:18 ID:x6FsQkpK
>>954>>942へのレス
961ご冗談でしょう?名無しさん:02/10/28 12:20 ID:x6FsQkpK
スマソ間違い。
× >>954>>942へのレス
○ >>959>>942へのレス
962もも:02/10/28 15:35 ID:gTQGeR77
キッテル固体物理学入門やってたら、突然、

「せん断定数」

という単語が出てきました。誰か、意味教えて下さい・・。解き方は分かったのですが、意味が知りたいです・・。
963ご冗談でしょう?名無しさん:02/10/28 15:43 ID:???
>>962
俺も昔、多分、同じ本やったが意味も求め方も分からなかった。
今、手元にあるが何のことやら。未だに求め方ワカラン。
964ご冗談でしょう?名無しさん:02/10/28 15:52 ID:fJxUhPjY
点A,Bに同じ電荷5×10の−6乗Cをもつ点電荷がある。ABの中点の
電位を求めよ。AB間の距離は0.4m。

こういう問題なんですが求め方からしてわかりません。どなたか説明よろしくお願いします。
965ご冗談でしょう?名無しさん:02/10/28 15:54 ID:fJxUhPjY
あと、100Vの電位差で加速された電子の速さの求め方もわからないです。
966ご冗談でしょう?名無しさん:02/10/28 15:59 ID:???
>>959
>地球自体にかかる遠心力を忘れてる。
結局、潮汐力と一緒なんだね。
ってことは、月が今どこにあるかの方が重要?
967ご冗談でしょう?名無しさん:02/10/28 16:03 ID:???
>>962-963
せん断応力と変形の間の比例定数じゃないの?
知らんけど。
968964,965:02/10/28 16:32 ID:fJxUhPjY
あと1時間ほどで試験なんでそれまでにどなたかお願いします!
969ご冗談でしょう?名無しさん:02/10/28 17:01 ID:xFWmuefX
>>968
V=44.9*10000
ぐらいか。簡単すぎるから、説明省く。ってこれで俺が間違ってたら、アホだな。
970965:02/10/28 17:27 ID:fJxUhPjY
どなたかあと30分以内に解き方教えてください。

>>965プラス次の問題の解き方も。

あるコンデンサーに電気容量が20μFのコンデンサーを並列につなぎ、これを10V
の電池につなぐと、全体で1×10の−3乗Cの電荷がたくわえられた。ある
コンデンサーの電気容量はいくらか?また、それぞれのコンデンサーに蓄えらえた
電荷はいくらか?
971ご冗談でしょう?名無しさん:02/10/28 17:31 ID:???
>>970
>>1の【質問する前に】を100回くらい読み直すこと
972ご冗談でしょう?名無しさん:02/10/28 17:49 ID:AubNEjkb
>>965
1/2mv^2=eV  より
1/2mev^2=100e

v^2=200e/me=200×1.76×10^11

v =5.93×10^6[m/s]
973ご冗談でしょう?名無しさん:02/10/28 17:56 ID:AubNEjkb
>>970
C=C1+C2
Q=C*V
から求める、たぶん
974権兵衛:02/10/28 20:50 ID:Mp/ZGiUd
水平な台の上に質量mの質点がおいてある。台が周期T,振幅Aで上下に単
振動するとき、質点が台から離れないための条件と、そのときの質点の
運動方程式を求めよ。
975ご冗談でしょう?名無しさん:02/10/28 20:57 ID:???
ワラタ
976ご冗談でしょう?名無しさん:02/10/29 12:44 ID:EORgmaBa
>>566
というか、潮汐力だし。
遠心力、つまり「加速度運動による見かけの重力」の、潮汐力。
ふつう「太陽の潮汐力」というと、これも含めて言う。

もちろん、月の潮汐力のほうが数倍強い。
977ご冗談でしょう?名無しさん:02/10/29 22:43 ID:???
物理の単位でp.e.ってなんですか?
978ご冗談でしょう?名無しさん:02/10/29 23:03 ID:pWx7QpeB
Q1=1,Q2=1があって試験電荷の位置が(X,Y)=(25,0)
にあることを確認し、これを基準として決めて、その
点での電場と電位の値を記録して、その点を少しずつ
ずらして同じ電位になる点を探していって、いくつか
データを取ったときに、第一象限だけの測定で十分で
ある理由を求めよ。
この問題を解ける方教えてください
979ご冗談でしょう?名無しさん:02/10/29 23:07 ID:pWx7QpeB
978のものですが、Q1=1,Q2=-1になった時、第一、第四象限での測定が
必要で、またこれだけで十分である理由も教えて下さい。お願いします。
980ご冗談でしょう?名無しさん:02/10/29 23:10 ID:???
>>978-979
試験電荷の位置がわからんが、「対称性」が理由でしょう。
ってゆーか、物理量はちゃんと単位を書けよ
ってゆーか、>>1の注意書き読めよ
ってゆーか、次スレまだぁ?


981ご冗談でしょう?名無しさん:02/10/29 23:11 ID:???
なにいってんのかわかんねー
わかりやすい文章の書き方から勉強しろ
982ご冗談でしょう?名無しさん:02/10/29 23:12 ID:wQhSamz4
半径R,高さHの円錐を頂点を下にして、水面からXcm
のところに沈めたときの浮力を、上面にかかる水圧と
下面にかかる水圧との差を計算して求めたいのですが、
うまくいきません。

アルキメデスの原理を用いれば、1/3πR^2H(g重)と
なるのはわかるのですが、上記の方法で解いた計算過程が
知りたいです。どなたか教えてください。
983ご冗談でしょう?名無しさん:02/10/29 23:13 ID:???
っていうか981は978へのレス
984ご冗談でしょう?名無しさん:02/10/29 23:14 ID:pWx7QpeB
>>980
もう少し詳しく教えて下さいませんか?お願いします
985ご冗談でしょう?名無しさん:02/10/29 23:32 ID:dh02sqb+
鉛直軸のまわりに自由に回転できる1辺の長さLの正方形の1巻き
コイルが、永久磁石のつくる磁束密度Bの一様な磁場の中に置かれ
ている。辺BCおよびDEの質量をそれぞれMとし、コイルの他の
部分の質量は無視する。また、コイルの電気抵抗は無視できる。
ただし、鉛直上向きをZ軸の正方向、磁場の向きをX軸の正方向と
して、コイルの中心を座標原点Oとする。コイルが回転するときの
摩擦は考えない。
986ご冗談でしょう?名無しさん:02/10/29 23:46 ID:dh02sqb+
上は2002年度TB・U重要問題集のP112の問題です。
詳しく説明できる方お願いします。図と問題を載せたいので
すが、崩れてしまうので遠慮させていただきます。
簡単に説明すると磁石SNの間に直流電流の通っているコイル
があって
電流Iが通っているとき、コイルが安定して静止するときの
コイルの上部分の座標を求めろということです。そのあと
コイルの両端にかかるFをそれぞれ求めろということだそ
うです。
ちなみに磁石にはさまれたコイルの中心がOです。
987ご冗談でしょう?名無しさん:02/10/30 00:06 ID:i1cGQpYn
P凾u=nRTとU=W+Qで体積の記述がない問題で
Uを体積×圧力で計算してもいいのでしょうか?
回答にのそう記載されているのですが、よく分かりません。
988ご冗談でしょう?名無しさん:02/10/30 00:54 ID:KvEi1y68
>>982
円錐表面の微小面積にかかる力を表面全体にわたって積分しろ
989ご冗談でしょう?名無しさん:02/10/30 01:56 ID:???
 膨張する宇宙の1番外側の、速度を教えてください。
「マジです。」
990ご冗談でしょう?名無しさん:02/10/30 02:05 ID:xxJPKgS8
物質の色ってどうやって決まってるんですか?
991ご冗談でしょう?名無しさん:02/10/30 02:05 ID:???
>>989
まず「膨張する宇宙の1番外側」をはっきりさせれ
9929拍:02/10/30 02:24 ID:dR7W5/VY
>991
考えときます。
993ご冗談でしょう?名無しさん:02/10/30 09:59 ID:???
>>989
閉じた宇宙なら、「膨張する宇宙の1番外側の速度」に対応するものが
あるかもしれない。しかし、「開いた」及び「平坦な」宇宙では
そういうものは、定義できない。
だって、空間の体積が∞だもん。
994ご冗談でしょう?名無しさん:02/10/30 11:28 ID:xjomhf5V
>>977
Pe (Pa gaugE) ならゲージ圧 (絶対圧−大気圧) のPaだな
p.e.とは書かないとは思うが
995つばめさん ◆dpLvO.T.cY :02/10/30 15:59 ID:???
>>985-986
コイル全体に生じる力のモーメントのZ成分=0になるようなコイル面の向きを
求めればいいと思うが、
コイル面の法線がどれだけZ軸から傾いてるか、
四角形がコイル面の法線に対しどんな向きかが分らないです。


ウワーン
996ご冗談でしょう?名無しさん:02/10/30 17:01 ID:???
新スレはこちら
■ちょっとした疑問はここに書いてね15■
http://science.2ch.net/test/read.cgi/sci/1035913464/
997ご冗談でしょう?名無しさん:02/10/30 17:28 ID:Kh6pShCY
>>995
2分のLです。
998ご冗談でしょう?名無しさん:02/10/30 17:29 ID:???
1000
999ご冗談でしょう?名無しさん:02/10/30 18:03 ID:EWOMgj4G
1000ご冗談でしょう?名無しさん:02/10/30 18:04 ID:EWOMgj4G
1000とりたかったんです
10011001
このスレッドは1000を超えました。
もう書けないので、新しいスレッドを立ててくださいです。。。